You are on page 1of 191

C S

GRBNER L G?
Bernd Sturmfels

L VN THIM
CON NGI V S NGHIP
H Huy Khoi

EUCLID V
C S CA HNH HC
Ng Bo Chu, Richard Fitzpatrick

TP TRNH TNG
TRONG NHM
Terence Tao

V HNG S LIN THNG


TRN LI T ONG
Hunh Cng Bng

V CC CHUYN MC KHC

No. 8

Nu hai vt th c c s Euler khc nhau, th chng khng th ci ny


bin thnh ci kia sau mt php bin i thun nghch lin tc (kiu nh
co dn cao su). Ngi ta ni hai vt th khng cng kiu tp.
RNG HP NH TO VA VN C
(GII THIU TP HC)
Nguyn Hu Vit Hng
C s ca hnh hc c coi l mt trong nhng quyn sch c nh
hng nht ti s pht trin ca vn minh nhn loi.
EUCLID V C S CA HNH HC
Ng Bo Chu, Richard Fitzpatrick

CH BIN:
Trn Nam Dng

BIN TP VIN:
V Quc B Cn
Ng Quang Dng
Trn Quang Hng
Nguyn Vn Huyn
Nguyn Tin Lm
L Phc L
Nguyn Tt Thu
ng Nguyn c Tin

13
04

No. 8

LI NG CHO EPSILON S 8
Ban Bin tp Epsilon

Epsilon c g trong s 8 ny? Ban bin tp xin mi bn c hy lt qua phn gii thiu ca
chng ti.
Mng lch s ton hc c bi ca GS H Huy Khoi vit v thn th v s nghip ca GS L Vn
Thim m chng ta va k nim 98 nm ngy sinh thng 3 va qua. Cng mng lch s ton hc
nhng c ni dung chnh l gii thiu v t-p c bi vit rt hm hnh, d hiu nhng cng rt
su sc ca GS Nguyn Hu Vit Hng vi ta "Rng hp nh to va vn c". Cc bn s
thy bt ng khi thy rng ha ra cng vi Leonard Euler v i, c mt nh th Vit Nam cng
tm ra tng to bo ca t-p (c dn trong ta ). Cng ch gii thiu v ton cao cp
c bi ca Matt Baker v nh l hn nhn, bi v c s Grobner ca Bernd Sturmfels. Epsilon
s 8 cng gii thiu bi vit ca Terence Tao trn blog ca anh gii thiu v bi bo ca anh v
V H Vn v tp trnh tng (Sum-free sets). Mc im sch s c bi ca GS Ng Bo Chu v
Richard Fitzpatrick gii thiu v cun Euclid v C s ca hnh hc. Mc vn cc vn c in
v hin i s c thi T ti Php do GS Nguyn Tin Dng gii thiu.
Bn cnh l cc chuyn mc quen thuc v ton s cp, li gii cc s trc, cc bi vit ca
cc tc gi "rut" ca Epsilon, bi ton hay, li gii p, ton gii tr ...
Chng ti c bit mun nhn mnh n s ng h ca cc tc gi dnh cho Epsilon. iu ny
ni ln mt tinh thn "Khi chng ta ng gp v cng ng, cng ng s v ta". GS H Huy
Khoi ch thn gi bi n cho Ban bin tp. GS Nguyn Hu Vit Hng, khi chng ti xin
bi cng vui v gi bn gc chng ti tin bin tp ch vi li nhn "vi iu kin cc cu
khng c sa, d ch mt ch". Cn Terence Tao th tr li ngn gn "This is fine with me.
Best, Terry". Cc tc gi "rut" nh Ng Quang Hng, L Ngc Tu th d bn bu vn lun
gng dnh thi gian vit bi cho chng ti. M cc anh th vit qu chun, BBT ch cn rp vo
l xong. Cc bn tr cng rt hng hi vit bi (s ny s c 3 bi ca cc bn hc sinh l Hong
Cao Phong, Nguyn Trn Hu Thnh v Xun Anh), sn sng nhn dch bi khi c nh (s
ny s c bn dch ca bn Nguyn V Anh).
M khng ch c cc bn tr sung. La trung nin cng sung lm. S ny s c bi ca Trnh
o Chin v anh cng Trn Minh Hin ha s vit 1 bi v nh l Cauchy Davenport cho s 6.
Lo ngoan ng Nguyn V Duy Linh cng hng hi dch bi v c s Grobner ca Sturmfels.
Ban bin tp cm n s ng h ca cc tc gi v s n nhn nng nhit ca cc c gi. Hy
tip tc chung tay st cnh cng chng ti trn con ng y gian nan pha trc.
i nhiu ngi, ta s i rt xa.

MC LC
Ban Bin tp Epsilon
Li ng cho Epsilon s 8 . . . . . . . . . . . . . . . . . . . . . . . . . . . . . . . . . .

L Ngc Tu
Xp x Diophantine trn Rn - Vc t xp x km v Tr chi siu phng tuyt i . . . .

Bernd Sturmfels
C s Grobner l g? . . . . . . . . . . . . . . . . . . . . . . . . . . . . . . . . . . . .

21

Terence Tao
Tp trnh tng trong nhm . . . . . . . . . . . . . . . . . . . . . . . . . . . . . . . . .

25

Matt Baker
Ton hc ca hn nhn . . . . . . . . . . . . . . . . . . . . . . . . . . . . . . . . . . .

29

Hunh Cng Bng


V hng s lin thng trn li t ong . . . . . . . . . . . . . . . . . . . . . . . . . . .

33

Nguyn Hu Vit Hng


Rng hp nh to va vn c (Gii thiu Tp hc) . . . . . . . . . . . . . . . . . . . . .

47

ng Nguyn c Tin
Cc bi ton on bi . . . . . . . . . . . . . . . . . . . . . . . . . . . . . . . . . . . .

55

Nguyn Trn Hu Thnh


Xung quanh nh l Brokard . . . . . . . . . . . . . . . . . . . . . . . . . . . . . . . .

61

Xun Anh
T gic ngoi tip ng trn . . . . . . . . . . . . . . . . . . . . . . . . . . . . . . . .

75

Trn Quang Hng, Nguyn Tin Dng


Mt s ng dng ca cc v i cc . . . . . . . . . . . . . . . . . . . . . . . . . . . .
4

93

Tp ch Epsilon, S 08, 04/2016


Nguyn Tin Lm, Ng Quang Dng
Tnh cht hnh hc ca ng cong bc ba . . . . . . . . . . . . . . . . . . . . . . . . . 119
Hong Cao Phong
Biu din s nguyn dng di dng tng cc s chnh phng . . . . . . . . . . . . . 125
Trnh o Chin
Mt s dng ton v bt phng trnh hm . . . . . . . . . . . . . . . . . . . . . . . . . 133
Trn Nam Dng
Bi ton hay li gii p . . . . . . . . . . . . . . . . . . . . . . . . . . . . . . . . . . 151
Ng Bo Chu, Richard Fitzpatrick
Euclid v C s ca hnh hc . . . . . . . . . . . . . . . . . . . . . . . . . . . . . . . . 155
H Huy Khoi
L Vn Thim: Con ngi v s nghip . . . . . . . . . . . . . . . . . . . . . . . . . . 161
Trn Nam Dng
Cc vn c in v hin i . . . . . . . . . . . . . . . . . . . . . . . . . . . . . . . 173
Ban Bin tp Epsilon
V k thi Vit Nam TST 2016 v danh sch i tuyn Vit Nam . . . . . . . . . . . . . . 189

Tp ch Epsilon, S 08, 04/2016

XP X DIOPHANTINE TRN Rn- VC T


XP X KM V TR CHI SIU PHNG
TUYT I
L Ngc Tu
(i hc Brandeis, Massachusetts, M)

1. Gii thiu
Trong phn 1 v phn 2 ca lot bi v xp x Diophantine [16, 17] chng ta chng minh
nh l Dirichlet trn Rn nh sau:
pE
nh l 1 (Dirichlet). Vi mi vc t v t xE 2 Rn X Qn , tn ti v s vc t hu t
D
q


p1
pn
; :::;
2 Qn vi pE 2 Zn v q 2 Z, q 0 sao cho:
q
q




xE pE < 1 :

q jqj1C n1
Tng qut hn mt t, chng ta gi mt hm s lin tc khng tng W R>0 ! R>0 l mt hm
xp x, v gi mt vc t xE 2 Rn l -xp x c1 nu nh tn ti v s pE 2 Zn , q 2 Z, q 0
sao cho:




p
E
xE
 .jqj/ :

q
jqj
Tp cc vc t -xp x c trn Rn s c k hiu l WAn . /. Nu nh ta s dng k hiu:

, th nh l Dirichlet c th c pht biu li thnh:


W k 7! k
 
WAn 1 D Rn :
n

Hm s s c gi l mt hm Dirichlet (trn Rn ) nu nh WAn . / D Rn .


Cu hi v hm s Dirichlet ti u cho Rn c tr li mt phn bi nh lut 0-1 sau ca
Khintchine:
nh l 2 (Khintchine 1926). K hiu  l o Lebesgue trn Rn :
(i) Nu nh chui

1
X

.k/n hi t th .WAn . // D 0.

kD1

(ii) Nu nh chui

1
X

.k/n phn k th .Rn X WAn . // D 0.

kD1
1

-approximable

Tp ch Epsilon, S 08, 04/2016





Vi  > 0 bt k, theo nh l 2,  WA 1 C
D 0. V vy, 1 C khng phi l hm
n
n


1
Dirichlet, v s m 1 C
trong nh l 1 l ti u. Tuy nhin, vi nhng hm s tin v 0
 n 1



1
1
nhanh hn mt t nh k 7! k n .log k/ n hay k 7! k n .log log k/ 1 , nh l 2 khng th
cho ta bit c rng y c phi l hm Dirichlet hay khng. Tht ra nhng hm ny khng th
l hm Dirichlet c, hay tng qut hn na, nu nh hm s tha mn:
1

lim k n .k/ D 0;

k!1

th khng phi l mt hm Dirichlet trn Rn : WAn . / Rn .


iu ny c th c chng minh bng cch ch ra s tn ti ca cc vc t xp x km c nh
ngha nh sau: xE 2 Rn c gi l xp x km nu nh tn ti mt hng s c > 0 (ty thuc vo
x)
E sao cho vi mi pE 2 Zn , q 2 Z, q 0:




xE pE > c :
(1.1)

q jqj1C n1
Tp cc vc t xp x km trn Rn s c k hiu bi BAn .
Khi n D 1, cc s xp x km tng ng vi cc lin phn s n b chn, v th BA1 khng
rng. Theo nh nh l ca Lagrange, rng mt s thc l mt s i s bc 2 khi v ch khi
m rng lin phn s ca l tun hon, mi s thc i s bc 2 v t u xp x km. Tuy
khng c cng c lin phn s khi n  2, chng ta c th chng minh c trc tip m rng
ca quan st trn cho Rn nh sau:
nh l 3. Nu nh f1; 1 ; :::; n g l mt c s ca mt trng s i s thc2 bc .n C 1/, th
E D .1 ; :::; n / 2 BAn .
Bi tp 4. Chng minh nh l 3.
V d trn ch ra rng c t nht v hn m c cc vc t xp x km trn Rn . Mi n nm
1954, Davenport [5] chng minh rng BA2 l mt tp khng m c, v mt nm sau y,
Cassels [4] chng minh rng BAn l khng m c vi n bt k.
Vy cc tp BAn ln nh th no? Phn tch BAn ra thnh nh sau:


[
n
BAn D
R X WA c 1
n

c>0
1 
[


R X WA k
n


;
1

kD1

v p dng nh l 2, ta c c.
.BAn / D 0:
Ni mt cch khc, theo o Lebesgue th BAn l mt tp nh khng ng k. Hn th na,
cch phn tch nh trn cn ch ra rng BAn thuc phm tr th nht theo Baire, mt hi m
c ca cc tp khng u tr mt.
Mt trong nhng cng c ph bin o kch c cc tp nh nh vy l chiu Hausdorff, k
hiu l dim (xem thm chi tit 2). S dng cch biu din cc s xp x km di dng lin
phn s b chn, Jarnk [11] chng minh rng tp cc s xp x km BA1 c chiu Hausdorff
bng 1. n 1966, Schmidt [21] m rng kt qu ny ra cho cc vc t xp x km:
2

real algebraic number field of degree .n C 1/

Tp ch Epsilon, S 08, 04/2016


nh l 5 (Schmidt 1966). dim BAn D n.
Schmidt chng minh kt qu ny da vo mt phng php hon ton mi m ng ngh ra: s
dng mt tr chi v hn vi thng tin hon ho m sau ny gi l tr chi Schmidt (xem [19,
Phn 4]). p dng tr chi ny, Schmidt chng minh rng nu nh yE1 ; yE2 ; ::: l mt dy cc vc
t trn Rn , th giao ca cc tnh tin ca BAn bi yE1 ; yE2 ; ::: vn c chiu Hausdorff bng n:
!
1
\

dimH
BAn CyEk D n:
kD1

Tng qut hn, Schmidt chng minh rng:


nh l 6 (Schmidt 1966). Gi U l mt tp m bt k trn Rn , ffi W U ! Vi g1
i D1 l mt h
3
n
m c cc hm t U vo cc tp m Vi  R .
!
1
\
dim
.fi / 1 .BAn / D n:
i D1

Da trn tng ca Schmidt, McMullen [20] gii thiu mt bin th ca tr chi Schmidt, gi
l tr chi tuyt i4 , v chng minh rng tp BA1 l mt tp thng cuc i vi tr chi ny
(thng cuc tuyt i5 ). Tuy nhin, khi n  2, BAn khng phi l mt tp thng cuc tuyt i.
V th Broderick, Fishman, Kleinbock, Reich, v Weiss [1] m rng tng ca McMullen ra
v gii thiu tr chi siu phng tuyt i, trong y tp thng cuc c gi l thng cuc siu
phng tuyt i6 , vit tt l HAW. p dng tr chi ny, BFKRW lm mnh hn nh l 6
ca Schmidt nh sau:
nh l 7 (BFKRW 2012). Gi U l mt tp m bt k trn Rn , ffi W U ! Vi g1
i D1 l mt h
7
1
n
m c cc vi phi C t U vo cc tp m Vi  R .
!
1
\
dim
.fi / 1 .BAn / D n:
i D1

nh l 5 v nh l 6 u l h qu ca nh l sau:
nh l 8 (BFKRW 2012). BAn l mt tp thng cuc siu phng tuyt i.
Trong phn cn li ca bi ny, chng ti s gii thiu chi tit hn v chiu Hausdorff, v tr
chi siu phng tuyt i, v chng minh nh l 8.

2. Chiu Hausdorff
Mt s ti liu tham kho cho chiu v o Hausdorff: Fractal Geometry: Mathematical
Foundations and Applications [6] v The Geometry of Fractal Sets [7] ca K. J. Falconer.
3

uniformly bi-Lipschitz
absolute game
5
absolute winning
6
hyperplane absolute winning
7 1
C diffeomorphism
4

Tp ch Epsilon, S 08, 04/2016


Vi mi tp con khng rng U  Rn , ng knh ca U c nh ngha l khong cch ln
nht gia 2 im bt k trong U :



E yE 2 U :
diam U WD sup xE yE W x;
Nu nh fUi g l mt h m c cc tp c ng knh khng qu v E 

1
[

Ui , ta s gi

iD1

fUi g l mt -ph8 ca E.
Vi mi tp E  Rn , v vi mi s; > 0, o ngoi .; s/ Hausdorff ca E c nh ngha
l:
(1
)
X
.diam Ui /s W fUi g l mt -ph ca E :
Hs .E/ WD inf
i D1

Bi tp 9. Chng minh rng Hs l mt o ngoi, ngha l tha mn 3 tnh cht sau:


(i) Hs .;/ D 0.
(ii) A  B H) Hs .A/  Hs .B/.
!
1
1
[
X
s
Ai 
(iii) H
Hs .Ai /.
i D1

i D1

Khi gim dn v 0, lp cc -ph ca E nh i, vy nn Hs .E/ tng dn v gii hn ca


Hs .E/ khi ! 0 tn ti (c th l C1). Ta gi gii hn ny l o ngoi Hausdorff vi
chiu s 9 ca E:
H s .E/ WD lim Hs .E/:
&0

Theo l thuyt o tng qut, khi ta gii hn vo cc tp H s -o c, H s tr thnh o.


Hn th na, cc tp Borel trn Rn u l H s -o c vi mi s > 0.
o Hausdorff c mt s tnh cht nh sau:
B 10. Cho E  Rn .
(i) Khi s D n, o Hausdorff vi chiu n tng ng vi o Lebesgue trn Rn : Tn ti
mt hng s c > 0 sao cho vi mi tp Borel E,
H n .E/ D c.E/:


(ii) Vi > 0, k hiu E D xE W xE 2 E . o Hausdorff vi chiu s ca E tha mn:
H s .E/ D s H s .E/:
(iii) Tng qut hn, nu nh f W E ! Rm l mt hm sao cho tn ti hng s c; > 0 vi
mi x;
E yE 2 E:




f .x/
E
f .y/
E  c xE yE ;
th vi mi s > 0:
H s= .f .E//  c s= H s .E/:
8
9

-cover
s-dimensional Hausdorff outer measure

10

Tp ch Epsilon, S 08, 04/2016


(iv) Nu nh H s .E/ < 1, th vi mi t > s, H t .E/ D 0.
(v) Nu nh H s .E/ > 0, th vi mi 0 < t < s, H t .E/ D 1.
Bi tp 11. Chng minh B 10.
Tnh cht (iv) v (v) trong B 10 cho thy c 1 thi im s D s0 m H s .E/ nhy t 1
xung 0, gi l chiu Hausdorff ca E:
dimH .E/ WD supfs > 0 W H s .E/ D 1g D inffs > 0 W H s .E/ D 0g:
Mt s tnh cht c bn ca chiu Hausdorff nh sau:
B 12. Cho E  Rn .
(i) Nu 0 < H s .E/ < 1, th dimH .E/ D s.
(ii) Nu E l mt tp m ca Rn th dimH .E/ D n.
(iii) Nu E  F th dimH .E/  dimH .F /.
(iv) Nu E l mt a tp m-chiu trong Rn th dimH .E/ D m.
(v) Vi mi dy fEi g:
dimH

1
[

!
Ei

D sup dimH .Ei /:


i

i D1

Bi tp 13. Chng minh B 12.


Chng ta s thy chiu Hausdorff l mt cng c quan trng m t cc tp c o Lebesgue
khng ng k thng qua mt v d ni ting v tp Cantor.
V d 14. Tp Cantor10 C c th c
[ nh ngha theo cc bc nh sau. t C0 D 0; 1.
bc th k  1, nu nh tp Ck D
Ik;i l hp ca cc on khng giao nhau tng cp, th
i

CkC1 s c bng cch b i cc on m gia cc Ik;i c di dng bng 1/3 di ca


Ik;i . C th hn, ta s c c:
C0 D 0; 1

 

1
2
C1 D 0;
[ ;1
3
3
 
 
 


1
2 1
2 7
8
[ ;
[ ;
[ ;1
C2 D 0;
9
9 3
3 8
9
::
:
Tp Cantor C l giao ca tt c cc tp Ck .
CD

1
\
kD0

10

Cantor middle third set

11

Ck :

Tp ch Epsilon, S 08, 04/2016

C0
C1
C2

1=3

1=9 2=9 1=3

2=3

2=3 7=9 8=9

C th thy c rng Ck bao gm 2k cc on thng c di 3

, v

C0  C1  C2      C:
Vi mi k  0:

 k
2
:
.C/  .Ck / D
3

T ta suy ra c rng C c di ( o Lebesgue trn R) bng 0.


V Ck l cc tp compact, C cng l mt tp compact v khng rng. Ta c th m t cc phn t
ca C nh sau. Vi mi s thc 0  x  1, vit x bng h c s 3:
x D .0:a1 a2 :::/3 D

1
X

ai 3 i ;

a1 ; a2 ; ::: 2 f0; 1; 2g:

i D1

Khi y:
x 2 C () a1 ; a2 ; ::: 2 f0; 2g:
log 2
Chng ta s chng minh rng vi s D
, H s .C/ D 1. V vy tp C c chiu Hausdorff bng
log 3
log 2
.
log 3
2k
[
t Ck D
Ik;i , trong y Ik;i l cc on ng c di 3 k , vi mi > 0, chn k ln
sao cho 3

i D1
k

 . Khi y fIk;i g l mt -ph ca C, v ta c c:


k

Hs .C/

2
2 
X
X
.diam.Ik;i //s D

3
i D1

Ly gii hn khi & 0:

log 2
 log
3

D 1:

i D1

H s .C/ D lim Hs .C/  1:


&0

chng minh chiu ngc li, gi fU g l mt ph bt k ca C. Khng mt tnh tng qut,


chng ta c th gi s rng U l cc
on thng ng. V C l mt tp compact, ta c th tm
c mt s hu hn cc on Uj 1j m ph C.

Gi k l s nguyn dng nh nht sao cho vi mi 1  i  2k v vi mi 1  j  m, nu nh


phn trong ca Ik;i giao vi Uj th Ik;i  Uj . Gi Ij l tp cc on Ik;i nm trong Uj :


Ij WD Ik;i W Ik;i  Uj ;
12

Tp ch Epsilon, S 08, 04/2016


v Uj0 l on ng nh nht cha mi on Ik;i trong Ij .

Ta c th d dng kim tra c rng Uj0 1j m cng l mt ph ca C, v:
m
X

diam Uj

s

j D1

m
X

s
diam Uj0 :

j D1

Nu nh Uj0 ch cha 1 on Ik;i th hin nhin: Uj0 D Ik;i . Cn khi:


2l < #Ij  2lC1 ;
ta c th tm c mt on ng K  Uj0 sao cho:
(i) K o \ Ck D ;,
(ii) diam K 

1
diam Uj0 ,
3

(iii) Uj0 X K o bao gm 2 on ng J v J 0 , mi on cha nhiu nht 2l on con Ik;i trong


Ij .
T ta c c:
diam Uj0

s

s
D diam J C diam K C diam J 0


 s
3
0

diam J C diam J
2

s
1
1
0
D2
diam J C diam J
2
2



1
1
s
0 s
 2 .diam J / C diam J
2
2
s
s
D .diam J / C diam J 0

log 2
sD
log 3

.0 < s < 1/

Quy np theo l, ta c c:
diam Uj0

s

.diam Ik;i /s :

Ik;i 2Ij


V Uj0 l mt ph ca C:
1
X

.diam.Ui // 

i D1

Vy H s .C/ D 1 v dimH .C/ D

m
X

diam Uij

j D1

s

2
X
.diam Ik;i /s D 1:

i D1

log 2
.
log 3
13

Tp ch Epsilon, S 08, 04/2016

3. Tr chi siu phng tuyt i


Trong phn ny, chng ta s k hiu B.x;
E r/ l qu bng11 ng trong Rn vi tm xE v bn
knh r:


B.x;
E r/ WD yE 2 Rn W xE yE  r :
Mt siu phng12 L trong Rn l mt tp hp cc nghim ca mt hm tuyn tnh n n khc 0.
Khong cch t mt im xE n L c nh ngha l:




dist x;
E L WD inf xE yE W yE 2 L :
Tp hp cc im c khong cch n L khng qu r c gi l mt r-ln cn ca L, v k
hiu l:



L.r/ WD xE W dist x;
E L r :
1
v mt tp i c S  Rn , tr chi -siu phng tuyt
3
i gia An v Bnh din ra nh sau:
Cho trc mt hng s 0 < <

1. An v Bnh ln lt thay phin nhau i, v Bnh l ngi i trc.


2. u tin Bnh chn mt qu bng bt k B1 D B.xE1 ; r1 / vi bn knh r1 > 0.
3. bc th i  1, An chn si -ln cn ca mt siu phng Li sao cho 0 < si  ri .
4. bc th i C 1, Bnh chn mt qu bng Bi C1 D B.xEi C1 ; ri C1 / sao cho ri C1  ri v:
i/
B.xEi C1 ; ri C1 /  B.xEi ; ri / X L.s
i :

B1

xE2

L1.s1 /

B2

r2
xE1

r1

An s thng nu nh:
S\

1
\

B.xEi ; ri / ;;

i D1
11
12

v chng ta dng sup-norm, nn tht ra B.x;


E r/ l hnh hp vung trong Rn
hyperplane

14

Tp ch Epsilon, S 08, 04/2016


cn nu khng th Bnh thng. Tp S c gi l mt tp -thng cuc siu phng tuyt i
nu nh An c chin lc lun lun thng trong tr chi -siu phng tuyt i (vit tt
l -HAW) bt k Bnh c i nh th no i na. S c gi l thng cuc siu phng tuyt
1
i (vit tt l HAW) nu nh S -thng cuc siu phng tuyt i vi mi 0 < < .
3
Lu 15. Khi n D 1, cc siu phng L trn R n gin l cc im. Khi y, tr chi siu
phng tuyt i cn c gi l tr chi tuyt i c gii thiu bi McMullen trong [20].
Lu 16. Tr chi siu phng tuyt i c th c chi trn mt khng gian metric tng
qut .X; dist/ m trong y cc siu phng L c th c thay th bi cc tp ng cho trc
trong X. Tr chi tng qut ny gi l tr chi H-tuyt i13 c gii thiu bi Fishman,
Simmons, v Urbanski [9], pht trin v p dng trong [14].
Lu 17. iu kin < 1=3 l d cho An c chn nh th no i na, Bnh cng lun c
la chn hp l cho bc i tip theo. Ta c th chi tr chi siu phng tuyt i trn mt tp
con X  Rn vi mi la chn ca Bnh u c tm nm trong X nu nh iu kin sau c
tha mn: Tn ti ; r0 > 0 nh sao cho vi mi qu bng B.x;
E r/ c tm xE 2 X v bn knh
0 < r < r0 v vi mi siu phng L,


X \ B.x;
E r/ X L. r/ ;:
iu kin trn m bo khi nh, tr chi -siu phng tuyt i trn X s ko di v hn.
Nhng tp X tha mn iu kin ny s c gi l -siu phng phn tn14 . X s c gi l
siu phng phn tn nu nh tn ti > 0 sao cho X l -siu phng phn tn. V d: tp Rn l
1
-siu phng phn tn, nhng mt ng thng trong khng gian 3 chiu R3 khng phi l mt
3
tp siu phng phn tn.
Lu 18. Nu nh X  R l mt tp siu phng phn tn, ta s chi tr chi siu phng tuyt
i trn X bng cch bt Bnh phi chn cc qu bng c tm nm trong X . Khi y cc tp
thng cuc s c gi l HAW trn X .
Mt s tnh cht quan trng ca cc tp thng cuc trong tr chi siu phng tuyt i nh sau:
nh l 19 ([1]). Gi s nh X  R l mt tp siu phng phn tn.
(i) Nu S  R l mt tp HAW th dimH .S/ D n.
(ii) Nu S l mt tp HAW trn X , v Y  X l mt tp siu phng phn tn, th S HAW trn
Y.
(iii) Nu S1 ; S2 ; ::: l cc tp HAW trn X th

1
\

Si cng l mt tp HAW trn X.

i D1

(iv) Gi s nh f W Rn ! Rn l mt vi phi C 1 , v S l mt tp HAW, th f .S/ cng l mt


tp HAW.
13
14

H-absolute game
-hyperplane diffuse

15

Tp ch Epsilon, S 08, 04/2016


tng chnh ca chng minh phn (i) ca nh l 19 l xy dng trong S mt tp con ging
nh tp Cantor15 nh sau: mi bc, ta chia nh qu bng B.xEi ; ri / thnh cc qu bng con
c phn trong i mt khng giao nhau vi bn knh ri v b i cc qu bng giao vi .ri /-ln
cn ca siu phng Li trong chin lc thng cuc ca An. Tp ging Cantor nm trong tp
HAW S ny cho chng ta mt chn di ca chiu Hausdorff ca S, v chn di ny s tin v
n khi tin v 0. Bn c c th xem thm chng minh y [3, nh l 2.2].
Bi tp 20.

1
(a) Chng minh rng tp Cantor trn R l -siu phng phn tn.
9

(b) C th thay s

1
bng mt s khc ln hn hay k0?
9

4. Vc t xp x km
p dng nh l 19, ta d dng c c nh l 8 suy ra cc nh l 5 v 7. chng minh
nh l 8, chng ta s dng B n hnh16 .
p
p0
B n hnh l m rng ca quan st sau trn R: Cho k > 1, nu nh v 0 l 2 s hu t
q
q
i
0
i C1
khc nhau vi mu s k  q; q < k , th:

p p 0 pq 0 p 0 q

D
 1 > k 2i 2 :
q

qq 0
q0
qq 0
1
Nh vy mi on thng trn R c bn knh 0 < r < k 2i 2 c cha nhiu nht mt s hu t
2
p
vi k i  q; q 0 < k i C1 .
q
Cho .n C 1/ im xE0 ; xE1 ; xE2 ; :::; xEn, sao cho n vc
t .xE1 xE0 /; .xE2 xE0 /; :::; .xEn xE0 / c lp
tuyn tnh. n hnh vi cc nh xE0 ; xE1 ; :::; xEn c nh ngha l:


.xE0 ; :::; xEn / WD xE0 C a1 .xE1 xE0 / C ::: C an .xEn xE0 / W a1 ; :::; an  0; a1 C ::: C an  1 :
Khi n D 1, .x0 ; x1 / l on thng ni x0 v x1 . Khi n D 2, .xE0 ; xE1 ; xE2 / l hnh tam gic vi
cc nh xE0 ; xE1 ; xE2 . Khi n D 3, .xE0 ; :::; xE3 / l t din vi cc nh xE0 ; :::; xE3 .
Th tch ca n hnh c th c tnh n gin nh sau:

1
 .xE0 ; :::; xEn / D det xE1
n

xE0 ; xE2

xE0 ; :::; xEn


xE0 :

1
B 21 (B n hnh [15, B 4]). Cho 0 < < . Vi mi k 2 N, gi Uk l tp cc
3
n
n
vc t hu t c mu nm gia nC1 .k 1/ v nC1 k :


pE
n
n
.k 1/
k
n
nC1
nC1
Uk WD
q<
:
W pE 2 Z ;
q
15
16

Cantor-like set
Simplex Lemma

16

Tp ch Epsilon, S 08, 04/2016


t Vn l th tch ca qu bng n v trong Rn . Vi mi:
0 < r < .nVn /

1
n

v vi mi xE 2 Rn , tn ti mt siu phng Lk sao cho:




Uk \ B x;
E k 1 r  Lk :


pEn
pE1 pE0
pE0 pE1
; ; :::;
;
Bi tp 22. Gi
l .n C 1/ im hu t trong Uk sao cho n vc t
q1
q0


q0 q1
qn
 
pE2 pE0
pE2 pE0
pEn
pE0 pE1
; ...,
; ; :::;
c lp tuyn tnh. Tm mt cn di ca 
.
q2
q0
q2
q0
q0 q1
qn
Bi tp 23. Chng minh B n hnh 21.
1
c nh bt k. Xt tr chi -siu phng tuyt i trn
3
R vi BAn l tp i tng ca An. Lu rng BAn tr mt, nn nu nh bn knh ca cc
la chn ca Bnh khng hi t v 0, An s thng. An c th i bt k cho n khi bn knh
1
ca qu bng Bnh chn nh hn .nVn / n . V th ta c th gi s rng B1 D B.xE1 ; r1 / vi
1
r1 < .nVn / n , v lim ri D 0.
Chng minh nh l 8. . Cho 0 < <

i !1

t c D 2 r1 . Vi mi k 2 N, gi ik l lt i m:
k 1 r1  rik > k r1 :
Vi mi lt khng nm trong dy fik g1
kD1 , An c th i bt k. bc i th ik , theo B
n hnh 21, tn ti siu phng Lk sao cho:
Uk \ B.xEk ; rik /  Lk :

Bc i lt th ik ca An s l kC1 r1 -ln cn ca Lk . V


. kC1 r1 /
Bik C1  Bij X Lk
;
pE
2 Uk :
q




yE pE  kC1 r1 D c k

q

vi mi yE 2 Bik C1 v vi mi

V:

1
[

c
1

q 1C n

Uk D Qn ;

kD1

theo nh ngha ca BAn (1.1),


1
\

B.xEi ; ri / D

i D1

V vy, An c chin lc cho kt qu

1
\

B.xEik ; rik / 2 BAn :

kD1
1
\

B.xEi ; ri / lun giao vi BAn .

i D1

17

Tp ch Epsilon, S 08, 04/2016


Lu 24. Mt h qu th v ca cc kt qu trn l 2 tp rt nh l tp Cantor C v tp cc s
xp x km BA1 vn giao nhau, v phn giao cng khng nh:
dimH .BA1 \C/ D dimH .C/ D

log 2
:
log 3

Khng nhng th, vi mi dy s a1 ; a2 ; :::, cc dch chuyn ca BA1 bi ai vn giao vi tp


Cantor:
!
1
\
log 2
.BA1 Cai / D
:
dimH C \
log
3
i D1
Kt qu ny c chng minh bi Fishman [8] s dng tr chi ca Schmidt.

Ti liu tham kho


[1] R. Broderick, L. Fishman, D. Kleinbock, A. Reich, v B. Weiss, The set of badly approximable vectors is strongly C 1 -incompressible, Math. Proc. Cambridge Philos. Soc. 153, no.
2 (2012), pp. 211253.
[2] R. Broderick, L. Fishman, v D. Simmons, Badly approximable systems of affine forms and
incompressibiblity on fractals, J. Number Theory 133 (2013), pp. 21862205.
[3] R. Broderick v D. Kleinbock, Dimension estimates for sets of uniformly badly approximable systems of linear forms, preprint (2013), arXiv:1311.5474.
[4] J. W. S. Cassels, Simultaneous Diophantine approximation II, Proc. Lon. Math. Soc. 3
(1955), pp.435448.
[5] H. Davenport, Simultaneous Diophantine approximation, Mathematika 1 (1954), pp. 5172.
[6] K. J. Falconer, Fractal Geometry: Mathematical Foundations and Applications (1990),
John Wiley & Sons.
[7] K. J. Falconer, The geometry of fractal sets, Cambridge Tracts in Math. 85 (1986), Cambridge Univ. Press.
[8] L. Fishman, Schmidts game on fractals, Israel J. Math. 171 (2009), pp. 7792.
[9] L. Fishman, D. Simmons, v M. Urbanski, Diophantine approximation and the geometry of
limit sets in Gromov hyperbolic metric spaces, sp c ng ti Mem. Amer. Math. Soc.,
arXiv:1301.5630.
[10] D. Gale v F. M. Stewart, Infinite games with perfect information, in: Contribution to the
theory of games, Vol. II, Annals of Math. Studies 28 (1953), pp. 245266.
[11] V. Jarnk, Diophantische approximationen und hausdorffsches mass, Recueil Math. Moscow
36 (1929), pp. 371382.
[12] A. Y. Khintchine, Einige Satze u ber Kettenbruche, mit Anwendungen auf die Theorie der
Diophantischen Approximationen, Math. Ann. 92 (1924), pp. 115125.
18

Tp ch Epsilon, S 08, 04/2016


[13] A. Y. Khintchine, Zur metrischen Theorie der Diophantischen Approximationen, Math.
Zeitschrift 24 (1926), pp. 706713.
[14] D. Kleinbock v T. Ly, Badly approximable S -numbers and absolute Schmidt games, J.
Number Theory 164 (2016), pp. 1342.
[15] S. Kristensen, R. Thorn, v S. Velani, Diophantine approximation and badly approximable
sets, Advances in Math. 203 (2006), pp.132169.
[16] L Ngc Tu, Xp x Diophantine trn R v Lin phn s, Epsilon 4 (2015).
[17] L Ngc Tu, Xp x Diophantine trn Rn - Quy tc Dirichlet v Hnh hc ca s, Epsilon
5 (2015).
[18] L Ngc Tu, Xp x Diophantine vi o - nh l Khintchine, Epsilon 6 (2015).
[19] L Ngc Tu, Tr chi v hn vi thng tin hon ho, Epsilon 7 (2016).
[20] C. McMullen, Winning sets, quasiconformal maps and Diophantine approximation, Geom.
Funct. Anal. 20, no. 3, (2010), pp. 726740.
[21] W. M. Schmidt, On badly approximable numbers and certain games, Trans. Amer. Math.
Soc. 123 (1966), pp. 178199.
[22] W. M. Schmidt, Badly approximable systems of linear forms, J. Number Theory 1 (1969),
pp. 139154.

19

Tp ch Epsilon, S 08, 04/2016

20


C S GROBNER
L G?
Bernd Sturmfels - i Hc California, Berkeley, M

Bi vit c Nguyn V Duy Linh dch t bi bo What is Groebner basis ca


gio s Bernd Sturmfels ng trn tp ch Notice of AMS, volume 52, number
10; 2005:
Mt c s Grobner l mt tp hp cc a thc nhiu bin c cc tnh cht mong mun v gii
thut. Mi tp hp cc a thc c th bin i thnh mt c s Grobner. Qu trnh bin i ny
tng qut ha ba k thut quen thuc: Php kh Gauss gii h phng trnh tuyn tnh, thut
ton Euclide tnh c chung ln nht ca hai a thc mt bin v thut ton n hnh trong
qui hoch tuyn tnh (xem 3) Chng hn, u vo ca php kh Gauss l tp cc dng tuyn
tnh sau
 D f2x C 3y C 4z 5; 3x C 4y C 5z 2g;
V thut ton bin i  thnh c s Grobner
% D fx

z C 14; y C 2z

11g:

Gi K l mt trng bt k, chng hn nh trng s thc K D R; trng s phc K D C;


trng s hu t K D Q; hay l trng hu hn K D Fp : Ta k hiu Kx1 ; : : : ; xn l vnh cc
a thc n bin xi vi h s trong trng K: Nu F l mt tp hp bt k cc a thc th ideal
sinh bi  l tp hp hi bao gm mi t hp tuyn tnh cc a thc ca 
hi D fp1 f1 C    C pr fr W f1 ; : : : ; fr 2  v p1 ; : : : ; pr 2 Kx1 ; : : : ; xn g:
Trong v d ca chng ta, tp  v c s Grobner % ca n sinh ra cng mt ideal h%i D hi:
Theo nh l Hilbert v c s, mi mt ideal trong Kx1 ; x2 ; : : : ; xn c dng I D hi; ngha l
n c sinh ra bi mt tp hu hn  cc a thc.
Mt th t n thc trn Kx1 ; x2 ; : : : ; xn l mt th t ton phn  trn tp hp tt c cc
n thc x a D x1a1 x2a2    xnan vi hai tnh cht sau
.1/ N l nhn tnh, ngha l x a  x b ko theo x aCc  x bCc vi mi a; b; c 2 Nn :
.2/ n thc hng l nh nht, c ngha l 1  x a vi mi a 2 Nn f0g:
Mt v d ca th t n thc (vi n D 2/ l th t t in phn bc
1  x1  x2  x12  x1 x2  x22  x32  x12 x2    
Nu chng ta c nh th t n thc ; khi mi a thc c duy nht mt s hng khi u
trong  .f / D x a : l n thc  cc i x a xut hin trong khai trin ca f vi h s
khc khng. Chng ta vit cc s hng ca f theo th t  gim dn, v thng thng chng
ta gch di s hng khi u. Chng hn, mt a thc bc hai c th c vit nh sau:
f D 3x22 C 5x1 x2 C 7x12 C 11x1 C 13x2 C 17:
21

Tp ch Epsilon, S 08, 04/2016


Gi s I l mt ideal ca Kx1 ; : : : ; xn : Khi ideal khi u ca n trong  .I / l ideal sinh
ra bi s hng khi u ca tt c a thc trong I W
in  .I / D hin  .f / W f 2 I i:
Mt tp con hu hn % ca I l mt c s Grobner tng ng vi th t theo s hng  nu nh
cc s hng khi u ca cc phn t trong % sinh ra ideal khi u:
in  .I / D hin  .g/ W g 2 %i:
Khng c yu cu v tnh ti tiu tr thnh mt c s Grobner. Nu % l mt c s Grobner
ca I th mt tp con hu hn bt k ca I cha % cng l mt c s Grobner. khc phc tnh
phi ti tiu , chng ta gi % l mt c s Grobner rt gn nu:
.1/ Vi mi g 2 % h s ca in  .g/ trong g l 1:
.2/ Tp hp fin  .g/ W g 2 %g l tp nh nht sinh ra in  .I /; v
.3/ khng c s hng theo sau vi mi g 2 % nm trong in  .I /:
Vi nh ngha ny, ta c nh l sau y: Nu nh c nh th t n thc  th mi ideal I
trong Kx1 ; : : : ; xn c mt c s Grobner rt gn duy nht.
C s Grobner rt gn % c th c tnh t mi tp sinh ca I theo phng php c gii
thiu trong lun n ca Bruno Buchberger nm 1965: Buchberger gi phng php ca mnh
theo tn ca ngi hng dn Wolfgang Grobner. Sau ngi ta nhn ra rng, tng v c s
Grobner c trc chng hn trong mt bi vit ca Paul Gordan, mt nh nghin cu v
bt bin. Tuy nhin Buchberger l ngi u tin ra mt gii thut tnh c s Grobner.
C s Grobner rt tin dng gii h phng trnh a thc. Gi s K  C; v  l mt tp hu
hn cc a thc trong Kx1 ; : : : ; xn : a tp ca  l tp tt c cc khng im phc chung
./ D f.z1 ; : : : ; zn / 2 Cn W f .z1 ; : : : ; zn / D 0 vi mi f 2 g:
a tp khng thay i nu ta thay  bi mt tp cc a thc sinh ra cng mt ideal trong
Kx1 ; : : : ; xn : Ni ring, c s Grobner gii hn % ca ideal hi c cng mt a tp:
./ D .hi/ D .h%i/ D .%/:
u im ca % l n cho bit nhng c tnh hnh hc ca a tp, nhng c tnh ny khng hin
l t : Cu hi u tin c t ra l liu a tp ./ c th rng hay khng. nh l khng
im Hilbert ng rng a tp ./ rng khi v ch khi % bng f1g:
Lm th no m s khng im ca mt h thng cc phng trnh cho? tr li cho cu
hi ny, chng ta cn thm mt nh ngha. Cho mt ideal c nh I trong Kx1 ; : : : ; xn v mt
th t n thc ; mt n thc x a D x1a1 x2a2    xnan c gi l chun nu nh n trong trong
ideal khi u in  .I /: S lng cc n thc chun l hu hn nu v ch nu mi
xi xut
3 bin

4
5
hin trong mt ly tha no ca ideal khi u. Chng hn,
nu
in

.I
/
D
x
;
x
;
x
1
2
3 th
3 4

4
s c su mi n thc chun, nhng trong in  .I / D x1 ; x2 ; x1 x3 tp hp cc n thc
chun l v hn.
a tp .I / l hu hn nu v ch nu tp hp cc n thc chun l hu hn v s lng cc n
thc chun bng vi lc lng ca .I / trong cc khng im bi cp k s c m k ln.
22

Tp ch Epsilon, S 08, 04/2016


Khi n D 1 y chnh l nh l c bn ca i s, nh l ny pht biu rng a tp .f / ca
a thc mt bin f 2 Kx vi bc d bao gm d s phc. y tp hp c phn t duy nht
ff g l c s Grobner, v cc n thc chun l 1; x; x 2 ; : : : ; x d 1 :
Tiu chun ca chng ta trong vic quyt nh liu mt a tp c hu hn hay khng tng qut
ha cng thc sau cho s chiu ca mt a tp. Xt mt tp con S ca tp cc bin fx1 ; : : : ; xn g
th no cho khng c n thc no trong S xut hin trong in  .I /; v gi s rng S c lc
lng ln nht trong s cc tp con c cng tnh cht. Khi lc lng ti i jS j bng vi s
chiu ca .I /:
Tp hp cc n thc chun l mt c s trong khng gian vector trn trng s K cho vnh
cc thng d Kx1 ; : : : ; xn =I: nh ca mt a thc p modulo I c th c biu din mt
cch duy nht nh l mt K t hp tuyn tnh ca cc n thc chun. Biu thc ny l dng
chun ca p: Qu trnh tnh dng chun l thut ton chia. Trong trng hp quen thuc vi mt
bin x; khi m I D hf i v f c bc d; thut ton chia biu din mt a thc ty p 2 Kx
di dng mt K t hp tuyn tnh ca 1; x; x 2 ; : : : ; x d 1 : Tuy nhin thut ton chia p dng
c cho mt c s Grobner ty % vi s lng bin ty .
Lm th no chng ta c th th nghim liu mt tp cc a thc cho trc l mt c s Grobner
0
0
0
hay khng? Xt hai a thc ty g v g trong %: Thnh lp S a thc m g mg : y m
0
0
0
v m l hai a thc bc nh nht c th th no cho m  in  .g/ D m  in  .g /: S a thc
0
0
m g mg nm trong ideal h%i: Chng ta p dng thut ton chia i vi c s Grobner tm
0
0
thi % cho m g mg : Dng chun thu nhn c l mt K t hp tuyn tnh ca cc n thc
trong khng c n thc no chia ht cho mt n thc khi u t %: iu kin cn % l
mt c s Grobner l
0

normalform% .m g

mg / D 0 vi mi g; g 2 %:

Tiu chun Buchberger pht biu rng iu kin cn cng chnh l iu kin : Mt tp hp
% cc a thc l mt c s Grobner nu v ch nu mi S a thc ca n c dng chun zero.
T tiu chun ny, ngi ta ra thut ton Buchberger 1 tnh c s Grobner rt gn % t
mt tp hp u ra bt k.
Tm li, cc c s Grobner v thut ton Buchberger tm chng l nhng khi nim cn bn
ca i s. Chng cung cp nhng cch tnh ton tin tin trong hnh hc i s, chng hn nh
l thuyt kh, tnh i ng iu, gii quyt tnh k d, ... Cng nh cc m hnh a thc c mt
khp ni t khoa hc n k thut, cc c s Grobner c cc nh nghin cu s dng trong ti
u ha, lp trnh, robotics, l thuyt iu khin, thng k, sinh hc phn t v nhiu ngnh khc.
Chng ti mi bn c th dng qua mt trong cc ci t ca thut ton Buchberger (chng hn
nh CoCoA, Macaulay2, Magma, Maple, Mathematica, hay Singular).

Ti liu tham kho


[1] DAVID COX, JOHN LITTLE, and DONAL O SHEA, Ideals, Varieties and Algorithms. An
Introduction to Computational Algebraic Geometry and Commutative Algebra, second ed.
Undergraduate Texts in Mathematics, Springer-Verlag, New York, 1997:
[2] NIELS LAURITZEN, Concrete Abstract Algebra: From Numbers to Grobner Bases, Cambridge University Press, 2003:
23

Tp ch Epsilon, S 08, 04/2016


[3] BERND STURMFELS, Two Lectures on Grobner Bases, New Horizons in Undergraduate
Mathematics, VMath Lecture Series, Mathematical Sciences Research Institute, Berkeley,
California, 2005; http://www.msri.org/communications/vmath/special_
productions/.

24

TP TRNH TNG TRONG NHM


Terence Tao - UCLA, California, M

Trong s ny, Epsilon trn trng gii thiu vi c gi mt bi vit ca nh ton


hc Terence Tao, qua bn dch ca Trn Nam Dng.
Ti v V H Vn va ng ln arXiv bi bo tp trnh tng trong nhm [1] (gi ng n tp
ch Discrete Analysis), cng vi bi bo tng quan nh km [2] (gi ng n J. Comb.). Cho
tp con A ca nhm cng tnh G D .G; C/, ta nh ngha .A/ l lc lng (s phn t) ca tp
con B ln nht ca A trnh tng trong A theo ngha mi tng b1 C b2 vi b1 ; b2 l cc phn t
phn bit ca B nm ngoi A. V d, nu A l chnh nhm th .A/ D 1, v khng c hai
phn t no ca A li c th c tng l mt ci g nm ngoi A. Mt cch tng qut, nu A l
hp ca k nhm, th .A/ ti a l bng A theo nguyn l chung v th.
Nu G l tp hp cc s nguyn, th ta khng c nhm con thc s no, v ta c th d on
rng .A/ s tng cng vi A. V d ta c kt qu n gin sau:
Mnh 1. Nu A l tp hp gm 2k s t nhin th .A/ > k.
Chng minh: Ta s dng l lun ca Ruzsa [3] vn c da trn mt l lun c ca Choi [4].
Gi x1 l phn t ln nht ca A, v sau mt cch quy, khi x1 ; x2 ; : : : ; xi c chn,
gi xi C1 l phn t ln nht trong A khng bng x1 ; x2 ; : : : ; xi sao cho xi C1 C xj A vi mi
j D 1; : : : ; i , v kt thc qu trnh xy dng ny khi khng tm c mt xi C1 nh vy. Qu
trnh ny s cho chng ta dy x1 > x2 >    > xm cc phn t trong A, trnh tng trong A v c
tnh cht vi mi y 2 A, hoc y bng vi mt trong cc xi , hoc y C xi 2 A vi i no vi
xi > y. Lp li qu trnh ny, ta thy mi y 2 A u c dng xi1 xi2    xij vi j  1 no
v 1  i1  i2     < ij  m. S cc biu thc xi1 xi2    xij ti a l 2m 1, do
2k  2m 1, t m  k C 1. V .A/  m nn ta suy ra iu phi chng minh.
c bit ta c .A/ >> logjAj i vi cc tp con A ca tp hp cc s nguyn. Ta c th ci
thin cn di n gin ny, nhng s rt tt cng sc. Cn di tt nht cho n thi im hin
nay l
.A/  logjAj.loglogjAj/1=2 o.1/
mt kt qu ca Shao [5] (xy dng da trn cng trnh trc ca Sudakov, Szemeredi v Vu
[6] v ca Doussep[4]). Trong chiu ngc li Ruzsa [3] xy dng v d mt tp hp A ln vi
.A/  exp.O. logjAj/).
S dng cng c chun ca ng cu Freiman, cc kt qu trn y cho s nguyn c th m
rng cho cc nhm abel khng xon G. Trong bi bo ca mnh chng ti nghin cu trng hp
ngc li khi G l nhm hu hn (nhng vn abel). Trong bi bo ny ca Erdos [7] (m trong
i lng .A/ ln u tin c gii thiu), cu hi sau y c t ra: nu A l ln so
vi .A/ th c suy ra tn ti hai phn t x; y thuc A sao cho x C y D 0 hay khng? Chng ti
tm c nhng phn v d n gin cho mnh ny. V d, nu H l mt nhm hu hn
25

Tp ch Epsilon, S 08, 04/2016


cng tnh ty , khi tp hp: A WD 1 mod 7; 2 mod 7; 4 mod 7  H  .Z=7Z/  H c
.A/ D 3 nhng khng c hai phn t x; y 2 A no c tng bng 0; dng v d ny vn ng
nu thay 7 bng mt s nguyn t Mersenne ln hn, v ta c phn v d trong Z=2n Z vi n ln
ty . Tuy nhin, theo hng khng nh, ta c th chng minh rng cu tr li cho cu hi ca
Erdos l khng nh nu jGj khng c c nguyn t nh. R hn l
Mnh 2. Vi mi k  1 tn ti C  1 sao cho nu G l nhm abel hu hn c bc khng c
c nguyn t nh hn hay bng C v A l mt tp con ca G c bc t ln l C v .A/  k
th tn ti x; y 2 A sao cho x C y D 0.
C hai cng c chnh s dng chng minh kt qu ny. Mt l b loi b s hc c
chng minh mi Kral, Serra v Vena [8]. Ch rng iu kin .A/  k c ngha l vi mi cc
phn t phn bit x1 ; : : : ; xkC1 2 A, tn ti mt trong cc phn t xi C xj ; 1  i < j  k C 1,
cng nm trong A. Mt cch nm na, b loi b s hc cho php ta gn nh c th b i
yu cu x1 ; : : : ; xkC1 phn bit, v iu ny c ngha l t x 2 A gn nh c th suy ra 2x 2 A.
Php i xng gn-v t ny khi kt hp vi gi thit rng jGj khng c c nguyn t nh s
cho rt nhiu s phn tn trong h s Fourier ca 1A m c th c khai thc chng minh
nh l.
Cng c th hai l nh l cu trc di y, l kt qu chnh ca bi bo ca chng ti, v i
ng theo hng phn loi cc tp con A vi .A/ nh:
Mnh 3. Cho A l tp con hu hn ca mt nhm cng tnh G bt k vi .A/  k. Khi
tm c cc nhm con hu hn H1 ; H2 ; : : : ; Hm vi m  k sao cho jA \ Hi j >>k jHi v
jA n .H1 [    [ Hm /j <<k 1. Hn na, nu m D k th tp ngoi l A n .H1 [    [ Hm / bng
rng.
Mt cch nm na, nh l ny ch ra rng v d v hp ca k nhm con ni trc dng nh
l v d duy nht ca cc tp hp A vi .A/  k theo m-u-l b sung vi tp con ngoi l
nh v vi s lm tt ca cc nhm con cho cc tp con dy.
nh l ny c mu sc ca cc nh l o khc trong t hp cng tnh nh nh l Freiman
v qu tht l ta c th s dng nh l Freiman (v cc cng c lin quan nh nh l BalogSzemeredi) thu c mt cch d dng phng n yu ca nh l ny. Tht vy, nu khng
c tp trnh tng ca A bc k C 1 th t l >>k 1 ca tt c cc cp a; b trong A phi c tng
cng nm trong A (ngc li ta c th ly k C 1 phn t ngu nhin ca A v chng s trnh
tng trong A vi xc sut dng). T iu ny v nh l Balog-Szemeredi v nh l Freiman
(trong mi nhm abel, c chng minh bi Green v Ruzsa [10]), ta thy rng A phi cn
xng vi i tp cp s (coset progression) H C P ca rank chn trn. Sau ta c th
kh thnh phn khng xon P ca i tp cp s ny bng mt s phng php (v d s dng
cc phng n ca l lun trong mnh 1), vi kt qu cui cng m ngi ta c th xc nh
v tr nhm H1 hu hn c phn giao ln vi A.
T im ny s hp dn nu loi b H1 khi A v lp li. Nhng ta s gp mt kh khn k thut
l loi b mt tp hp nh H1 khi A c th lm thay i i lng .A/ theo mt cch khng
th d on c, v th ta vn cn phi gi H1 li khi phn tch tp thng d A n H1 . Kh khn
th hai l tp hp A n H1 c th l nh hn ng k so vi A v H1 nhng vn cn ln theo
ngha tuyt i, do ni ring th mi s hng sai s c kch thc chn trn bi jAj vi  nh
26

Tp ch Epsilon, S 08, 04/2016


c th vn ln so vi tp thng d A n H1 , v do s hng sai s l khng chp nhn c.
Ta c th vt qua c nhng kh khn ny nu trc tin ta thc hin bc tin chun ha
ca nhm H1 , cho tp thng d khng giao vi mi i tp ca H1 qu nhiu. Cc l lun cn
tr nn phc tp hn khi ta bt u loi b nhiu hn mt trong cc nhm H1 ; : : : ; Hi t A v
phn tch tp thng d A n .H1 [    [ Hm /; tht vy qu trnh qun l epsilon lin quan s tr
nn phc tp n ni m chng ta buc phi s dng cch pht biu phn tch khng tiu chun
ca bi ton gi phc tp ca cc l lun mc chp nhn c (xem bi vit blog trc
ca ti v ch ny [11]). Mt nhc im ca cch lm nh vy l chng ta khng c gii hn
hiu qu cho cc hng s trong nh l chnh ca chng ti; s th v c c mt n s c
mt chng minh trc tip hn cho nh l ca chng ti m c th s dn n gii hn hiu qu.

Ti liu tham kho


[1] http://arxiv.org/abs/1603.03068
[2] http://arxiv.org/abs/1603.03071
[3] Imre Z. Ruzsa, Sum-Avoiding Subsets, The Ramanujan Journal, March 2005, Volume 9,
Issue 1, pp 77-82.
[4] Choi, S. L. G. On a combinatorial problem in number theory. Proc. London Math. Soc. (3)
23 (1971), 629642.
[5] Shao, Xuancheng Finding linear patterns of complexity one. Int. Math. Res. Not. IMRN
2015, no. 9, 23112327.
[6] Sudakov, B.; Szemerdi, E.; Vu, V. H. On a question of Erdos and Moser. Duke Math. J. 129
(2005), no. 1, 129155.
[7] Erdos, P. Extremal problems in number theory. 1965 Proc. Sympos. Pure Math., Vol. VIII
pp. 181189 Amer. Math. Soc., Providence, R.I.
[8] Krl, Daniel; Serra, Oriol; Vena, Llus On the removal lemma for linear systems over abelian
groups. European J. Combin. 34 (2013), no. 2, 248259.
[9] Freiman theorem
[10] Green, Ben; Ruzsa, Imre Z. Freimans theorem in an arbitrary abelian group. J. Lond. Math.
Soc. (2) 75 (2007), no. 1, 163175.
[11] https://terrytao.wordpress.com/2007/06/25/ultrafilters-nonstandard-analysis-and-epsilonmanagement/

27

Tp ch Epsilon, S 08, 04/2016

28

TON HC CA HN NHN
Matt Baker Vin cng ngh Georgia, M

Nguyn V Anh dch t blog ca Matt Baker Trn Nam Dng hiu nh.
Thn tng V Quc B Cn, bin tp vin u tin ca Epsilon nhn ngy ci ca
anh v ch Nguyn Th Kim Anh vo ngy 27 / 3 / 2016.
Ngy 25 thng 6 nm 2014
Cng kh lu k t bi blog gn nht ca ti mt trong nhng l do l gn y ti kt
hn. k nim cho s kin ny, cng nh nh du s tr li ca ti trn blog ton hc ti s
ng bi v nh l Hall v hn nhn.
Hy xt tr chi solitaire (game xp bi ca Windows): Bn c mt b bi c chia lm 13 cc,
mi cc 4 l, v bn phi chn 1 l t mi cc sao cho khng c gi tr no (t A n K) lp li.
y l mt s kin ton hc v cng p l tr chi lun c th hon thnh, bt chp cc qun
bi c chia ra sao.
Chng ta s suy lun iu ny t mt kt qu tng qut hn ca Philip Hall, thng c gi
l nh l Hall v hn nhn. Cho hu hn tp hp A1 ; A2 ; : : : ; An v bn phi tm cc phn
t ring bit x1 2 A1 ; x2 2 A2 ; : : : ; xn 2 An : (Trong v d tr solitaire, cho Aj cha cc gi
tr ca cc l bi nm trong cc th j:) S la chn ny c gi l SDR (system of distict
representatives-h i din ring bit). Di iu kin no th iu ny kh thi? h i din
ring bit ny tn ti th iu kin cn thit l vi mi tp con J 2 .1; 2; : : : ; n/; tp hp
AJ D Uj 2J Aj cha t nht jJ j phn t. nh l ca Hall khng nh rng nhng iu kin trn
cng chnh l iu kin .
Trong v d v tr bi bi, ta chn k cc bt k, cha 4k l bi. Bi v ch c 4 l bi mi cc vi
mi gi tr, khi chng ta kt hp nhng cc bi chng ta chn, s c t nht k gi tr khc nhau.
Do , nhng iu kin ca nh l Hall u tha v nh l hn nhn s khng nh rng chng
ta c mt cch i thng.
Mt v d khc, xt bi ton sau c li gii n gin hn nh l Hall v hn hn:
[K thi Putnam, bi ton B-3] Mt gii u vng trn 2n i ko di 2n 1 ngy. Mi ngy, tt
c cc i u vi mt i khc, trong c 1 i thng v 1 i thua trong mi n trn. Xuyn
sut gii u, 2 i bt k gp nhau ng 1 ln. Liu c lun chn c mi ngy mt i thng
m khng c i no c chn nhiu hn 1 ln?
Kt qu ca Hall c bit ti l nh l hn nhn v n c th pht biu li theo cch sau. Gi s
mi mt tp hp hu hn chng trai quen vi mt tp hp hu hn cc c gi. iu kin l g
mi chng trai u kt hn vi mt trong nhng ngi h quen bit? iu kin cn v l
mi tp hp k chng trai, c chn mt cch ngu nhin, quen vi t nht k c gi. Rt d dng
nhn thy s tng ng ca hai cch pht biu (cho Aj l tp hp cc c gi m chng trai th
j quen).
29

Tp ch Epsilon, S 08, 04/2016


Cch chng minh n gin nht cho nh l ca Hall m ti bit xut hin u tin trong bi bo
2 trang ca Halmos v Vaughan, c thc hin php quy np theo s chng trai n: Ta trch dn
trc tip chng minh ca Halmos v Vaughan:
Vi n D 1; kt qu l hin nhin. Vi n > 1; nu vi mi tp hp k bn nam 1 6 k 6 n; c
t nht k C 1 ngi quen, 1 bn nam s ci mt trong nhng ngi quen bt k ca anh ta
v a nhng ngi cn li vo gi thuyt quy np. Nu ngc li, mt s nhm k chng trai,
1 6 k 6 n; c chnh xc k ngi quen, th tp hp ny s c th kt hn theo gi thit quy np,
n k chng trai cn li cng tha mn iu kin cn vi nhng c gi cha kt hn. Tht vy,
nu 1 6 h 6 n k; v nu mt s tp hp h chng trai cha c v t hn h c gi cha c chng,
th tp hp h chng trai cha c v cng vi k chng trai kt hn s quen t hn k C h c gi.
p dng gi thit quy np vo n k chng trai cha c v, ta thu c kt lun nh l ng vi
n C 1 chng trai.
C mt chng minh p cho nh l ca Hall, c ra bi Jack Edmonds, da trn i s
tuyn tnh. y l mt v d s dng k thut i s gii quyt cc vn t hp. Trc khi
xem xt chng minh ca Edmonds, chng ta cn 2 nh ngha:
nh ngha 1. Cho B l mt ma trn m  n vi cc s hng trong trng F c s 0: Chng ta
gi B l generic nu cc s hng khc 0 ca n c lp i s (chng khng tha mn mt a
thc khng ng nht 0; vi h s trong Q/:
nh ngha 2. Cho B l mt ma trn m  n c hng r < n: Ta ni B l dng Edmonds bnh
thng (Edmonds Normal Form, hay ENF) nu B c th vit di dng mt ma trn c ma trn
B1 l ma trn r  .r C 1/ pha trn bn tri m:
.a/ r C 1 ct u tin ca B to thnh mt tp hp ph thuc tuyn tnh nh nht.
.b/ Mi dng ca ma trn B2 pha di bn tri l mt t hp tuyn tnh cc dng ca B1 :

D nhn thy, mi ma trn B bc m  n c hng r < n c th chuyn v dng ENF bng cch
hon v cc ct v dng. Tht vy, nh vo vic hon v cc ct, chng ta c th gi s r C 1 ct
0
u tin to thnh mt tp hp ph thuc tuyn tnh nh nht. Ma trn hon v B xc nh bi
r C 1 ct u tin c hng r; nn nh vo hon v cc dng chng ta c th gi s r dng u
0
tin to thnh mt c s cho khng gian dng ca B : Ma trn kt qu l mt ENF.
By gi chng ta cng xem php chng minh ca Edmonds cho nh l ca Hall:
Chng minh. Cho n l s lng chng trai v m l s lng c gi. Cho ma trn generic m  n
vi Bij 0; nu c gi th j quen chng trai th j: S dng cng thc chun cho nh thc ca
ma trn bng tng cc hon v, cng vi generic ca B; ta thy rng s tn ti mt cch t chc
ci hnh phc khi v ch khi nh thc ca mt ma trn nh n  n no ca B khc 0; v iu
ny xy ra khi v ch khi B c hng > n: Khng mt tnh tng qut, chng ta c th gi s (bng
cch hon v cc ct v dng nu cn) rng B l mt ENF: V cc ct ca B1 ph thuc tuyn
tnh, tn ti mt vc-t v khc 0 trong nhn ca B: Bng php kh Gauss, chng ta c th gi s
ta ca v l nhng a thc c s hng khc 0 ca B1 : Nh vo tnh cht .a/ ca ENF, khng
30

Tp ch Epsilon, S 08, 04/2016


mt s hng no ca v bng 0: V da vo tnh cht .b/; mi dng ca B2 u trc giao vi v:
V B2 l geniric, nn B2 D 0: Nhng nh vy th tp hp cc chng trai tng ng vi r C 1 ct
u tin ch quen vi cc c gi tng ng vi r dng u tin, mu thun.
Cc chng minh bng i s tuyn tnh xem ra l qu mc cn thit so vi phng php t hp
n gin. Nhng n li c vi tnh nng kh hp dn: V d, vic lm sng t mt cch r rng
dn ti mt thut ton xc sut hiu qu kim tra liu c mt h i din ring bit (SDR).
(S thun tin nu xem cc s hng ca B l cc n s). Chng minh trn a bi ton v vic
xt xem mt a thc c khc 0 hay khng. V mt tnh ton l khng kh thi tnh cc nh
thc con ca B nh cc a thc, nhng nu chng ta thay th cc gi tr c th vo cc n s,
chng ta c th xc nh mt cch hiu qu cc gi tr tng ng c khc 0 theo mt m-un
nguyn t ln hay khng. Nu dng thut ton nhn nhanh ma trn tnh nh thc th ta s
c thut ton tim cn nhanh nht xc nh bi ton hn nhn c nghim hay khng (trong
thc t, c nhng thut ton tt nh chy nhanh hn, v cn tm ra cc cp ghp).
Cui cng, chng ti mun cp n nh l Hall c th m rng cho cc trng hp c v s
chng trai (nhng s c gi quen tng chng trai vn l hu hn). S tng qut ny do Everett v
Whaples xut. Mt ln na ta c cch no tt hn l trch dn Chng minh t Sch (Proof
from the Book) ca Halmos v Vaughn, s dng cu trc im - tp hp topo gim bt trng
hp hu hn: Nu tp hp B cc chng trai l v hn, xt mi b 2 B v tp G.b/ l nhng ngi
b quen, topo ha cc tp topo ri rc, G.b/ l khng gian compact Hausdorff. Vit G theo dng
tch -cc topo cho mi G; theo nh l Tikhonov, G l compact. Nu fb1 ; b2 ; : : : ; bn g l mt
tp hu hn chng trai bt k, xt tp hp H cha cc phn cha tt c cc phn t g D g.b/ ca
g sao cho gi gj vi bi bj ; j D 1; ; 2; : : : ; n: H l tp con ng ca G; v da vo kt
qu trong trng hp hu hn, H khc rng. V hp hu hn cc tp hp hu hn nn lp ca tt
c tp hp nh H c tnh cht giao hu hn, v do c giao khc rng. V phn t g D g.b/
0
0
trong phn giao ny tha mn tnh cht g.b/ g.b / vi mi b b ; php chng minh hon tt.

Ghi ch
1: Bi bo cng b nm 1950 ca Halmos v Vaughan, c tn l The Marriage Problem,
c ng li trong cun sch Classic Papers in Combinatorics (bin son bi by Gessel
and Rota), cng vi bi bo ban u ca Hall. Bi bo ca Edmonds nm 1967 c tn l
Systems of Distinct Representatives and Linear Algebra. on chng minh ca Edmonds
m ti ni n trn c ly t bi bo ny 1: Cc bnh lun trn y v nh thc
v kha cnh thut ton ca bi ton hn nhn c trch t bun sch Thirty - three
miniatures ca Jiri Matousek. Cun sch ny cha mt s v d th v v chng minh cc
kt qu t hp bng i s tuyn tnh.
2: Li gii ca bi ton Putnam c th xem 2: Nu bn thch bi ton ny, y c mt
bi ton vui khc da trn nh l Hall: Nu G l mt nhm hu hn v H l mt nhm
con ch s n: Chng minh rng tn ti cc phn t g1 ; g2 ; : : : ; gn thuc G sao cho tn ti
ng mt gi trong mi lp k tri ca H v ng mt gi trng mi lp k phi.
3: Richard Rado chng minh m rng quan trng sau y ca nh l Hall cho matroid: Gi
s Ai l h cc tp con ca tp hu hn S c nh s bi tp I v M l matroid trn S:
Khi tn ti mt h i din ring bit gm cc phn t c lp nu v ch nu AJ c
hng t nht l jJ j vi mi J  I: Nh mt h qu v d, nu A1 ; A2 ; : : : ; An l cc tp
31

Tp ch Epsilon, S 08, 04/2016


con ca mt khng gian vc-t V c n chiu sao cho dim span AJ > jJ j vi mi tp con
J 2 f1; 2; : : : ; ng; th V c c s fa1 ; a2 ; : : : ; an g vi ai 2 Ai vi mi i:
4: C mt phin bn ca nh l Hall v hn nhn cho siu th c pht biu bi Aharoni
v Haxell trong c a ra mt chng minh mi ca nh l Hall da trn b Sperner
(b ny li tng ng vi nh l im bt ng Brouwer trong t-p). Xem bi blog
3 ca Gil Kalai bit r hn.
5: C mt s kt qu ni ting trong t hp m u tng ng nhau, theo ngha l chng
c th suy ra c t nhau mt cch kh n gin. l nh l Hall v hn nhn, nh l
Dilworth, nh l lung cc t lt ct cc tiu v nh l Menger. Mt c im chung
ca tt c cc nh l ny l mt vi iu kin cn hin nhin thy li chnh l iu kin
. C th xem y 4:
6: Mt s bi ton yu thch ca ti lin quan n bi ton hn nhn l bi ton hn nhn n
nh v bi ton th k.
7: nh l Hall ng vai tr ch o trong bi bo sau 5 ca Zur Izhakian v Louis Rowen
v supertropical matrix algebra.
8: nh l Hall v hn nhn c th c hiu nh iu kin cn v tn ti cp ghp
y cho mt th hai phe. Tutte chng minh m rng sau y cho mt th bt
k, khng nht thit phi l hai phe: Mt th G c cp ghp y khi v ch khi vi
mi tp con U cc nh, th cm sinh trn phn b ca U c tt a jU j thnh phn lin
thng vi s nh l. T nh l Tutte ta c th ngh n mt nh ngha thong hn v
hn nhn.

Ti liu tham kho


[1] https://www.ima.umn.edu/preprints/Feb92Series/918.pdf
[2] http://kskedlaya.org/putnam-archive/2012s.pdf
[3] https://gilkalai.wordpress.com/2012/11/25/
happy-birthday-ron-aharoni/
[4] http://robertborgersen.info/Presentations/GS-05R-1.pdf
[5] http://arxiv.org/abs/0806.1178

32

V HNG S LIN THNG TRN


LI T ONG
Hunh Cng Bng - Ecole Normale Suprieure de Lyon

Hng s lin thng trn mt mng li l i lng lin quan n s lng ng i khng t ct
trn li (li c th l vung, hay lc gic (li t ng), ...) Vo nm 1982, mt lp lun
da trn
q gas coulomb ca Nienhuis d on rng trn mng li lc gic, hng s lin thng
p
bng 2 C 2. iu ny c chng minh chi tit bi Duminil-Copin v Smirnov bng cch
s dng phng php "parafermionic".

1. Gii thiu
K hiu cn l s ng i khng t ct (SAW) trn mng li lc gic H vi di n v bt u
t O. Ta c:
p
cn  . 2/n
iu ny c c bng cch m s ng i ln pha trn v i xung pha di bc th
2k C 1 v s ng i ngang bc th 2k C 2 vi k 2 N.
Ta c th ct mt ng i SAW c di m C n thnh hai phn SAW c di n v m. Do
cmCn  cm  cn :
Theo b subadditivity, ta c
1

lim cnn D  2
1

p


2; 2 and cn  n ; 8n

v  D lim cnn D inf.cnn /.


n

q
nh l 1. i vi mng li lc gic, ta c  D
33

2C

2.

Tp ch Epsilon, S 08, 04/2016


Mt vi k hiu: Ta s lm vic trn nhng trung im ca cc cnh ca H . Tp hp tt c nhng
im l H .
Ta vit a 2 H ; W a ! E ngha l bt u ti a v kt thc ti mt im trong E  H .
l. / D #fa 2 H W a 2 g l di ca (V l s cc nh thuc ).
Ta nh ngha hm s sau y:
X

z.x/ D

x l. / for x > 0:

Wa!H

Ta co
z.x/ D

x l. / D

cn  x l. / D

Wa!H

cn  x n 

.x/n :

Do ,
 If x <

1
then z.x/ < C1:


 If x >

1
then z.x/ D C1.


Khi , ta ch cn chng minh


1
z.x/ < C1 if x < p
p
2C 2
va
z.x/ D C1 if x D p

1
2C

p :
2

1
2i
ta dat xc D p
p voi j D e 3 .
2C 2

2. Cu
Trong mc ny, ta nh ngha mt lp ca SAW l cu v ta s chng minh rng s cu s tng t
l vi s SAW ( n ) v t ta c th chng minh rng
q
p
b .H / D 2 C 2:

nh ngha 1. Mt cu n-bc l mt SAW c n-bc sao cho


1 .0/ < 1 .i/  1 .n/ 8i D 1; 2; 3; : : : ; n:
trong 1 .i/ l ta u tin .i/.
34

Tp ch Epsilon, S 08, 04/2016


K hiu bn mt cu n-bc vi .0/ D 0. Ta t b0 D 1:
Ta c bmCn  bm  bn , do
1

b D lim bn n D sup bnn :


n!C1

Hn na, bn n  nb  n .
nh ngha 2. Mt na mt phng n-bc l mt SAW c n bc vi 1 .0/ < 1 .i/; 8i:
Ta t hn l s lng na mt phng n bc vi .0/ D 0:
nh ngha 3. dy ca na mt phng n bc l
max 1 .i/

min 1 .i/

0i n

0i n

vi bn;A l s n-bc cu vi di A:
Ta c bn D

n
X

bn;A .

AD1

nh l 2. (Hardy-Ramanujan): Cho n 2 N, gi PD .n/ l s cch vit n D n1 Cn2 C  Cnk


trong n1 > n2 >    > nk  1 cho bt k k, khi , ta c
 n  21
as n ! C1:
ln PD .n/  
3
Mnh 1. hn  PD .n/  bn vi mi n  1.
t b0 D 0, ta nh ngha
Ai C1 D max . 1/i . 1 .j /

1 .ni //

j >ni

va

n
ni C1 D max j > ni . 1/i . 1 .j /

o
1 .ni // D Ai C1 :

Ngha l: n1 l gi tr cc i ca 1 .j / W j > n0 v n2 l gi tr cc tiu ca 1 .j /; j > n1 .


Ta t hn .a1 ; a2 ; : : : ; ak / l s na mt phng n bc vi
K D k; Ai D ai :
Ta c

hn .a1 ; a2 ; : : : ; ak /  hn .a1 C a2 ; a3 ; : : : ; ak /
 ::: 
 hn .a1 C a2 C    C ak / D bn;a1 Ca2 CCak :

Do ,
hn D

hn .a1 ; a2 ; : : : ; ak /

k1 1a1 <a2 <<ak




bn;a1 Ca2 CCak

k1 1a1 <a2 <:::<ak


n
X

n
X

AD1

AD1

PD .A/    bn;A  PD .n/   

bn;A


bn

35

Tp ch Epsilon, S 08, 04/2016


Chng ta c c hn  PD .n/  bn .
Chng minh ny cung cp mt phng php phn tch thnh nhng cu c di gim dn.
Chng ta s chng minh nh l sau y ca Hammersley-Welsh cho mng lc gic.
  21
2
, khi , tn ti mt hng s n0 .B/ sau cho
nh l 3. C nh B > 
3
8n > n0 .B/ W cn  bnC3  e B

p
nC3

Chng minh. Ta s chng minh rng


cn 

n
X

hn

 hmC4 :

mD0

t x1 D max 1 .i/; m D maxfi W .i/ D x1 g. Ta xa i .m/ v thm vo 5 im gia


0i n

a1 ; a2 ; a3 ; a4 ; a5 ca lc gic cha .m/:


ng i ny . .0/; .1/; : : : ; .m 1/; a1 ; a2 ; a3 ; a4 / l mt na mt phng c .m C 3/ bc
v .a5 ; .m C 1/; : : : ; .n// l mt na mt phng c .n m/ bc. Do ,
cn 

n
X

hn

 hmC3

mD0

S dng mnh :
cn 

n
X

hn

mD0

n
X

 hmC3 

n
X

m/  PD .m C 3/  bn

PD .n

 bmC3

mD0

PD .n

m/  PD .m C 3/  bnC3

mD0

 n  21

Theo nh l Hardy th: PD .n/  


3
  21
2
B > B0 > 
.
3
chng ta c c:
PD .n

khi n ! C1, do 9 W PD .n/  e

0
2 B

hp

m/  PD .m C 3/  e

Do :
cn  .n C 1/ 2 e B

n m
2 C

nC3

p mC3 i

9B0 .B/; 8n  B0 .B/ W cn  e B

36

 2eB

 bnC3

H qu 1. .H / D b .H /.

nC3

bnC3 :

nC3

2
B 0 . A
2/

vi

Tp ch Epsilon, S 08, 04/2016


Ta c:
cn  e B

p
nC3

bnC3 ) cnn  e B

nC3
n

1
nC3
bnC3

 nC3
n

)   b

nn  D b .
Ghi ch. Chng ta c cng kt qu cho mng hnh vung Z2 bng cch thay th n C 3 bi n C 1:
  12
p
2
C nh B > 
khi c mt gi tr n0 D n0 .B/ sao cho cn  bnC1 e B n vi mi
3
n  n0 .
X
Chng ta nh ngha B.x/ D
bn x n , nh vy
 Nu x >

1
: B.x/ D C1


 Nu x <

1
: B.x/ < C1.


3. Parafermionic
Mt min  l hp ca tt c cc trung im t vic a ra mt tp hp ca cc nh V ./.
Mt trung im z s thuc v min  nu t nht mt u mt ca cnh lin kt ca trung im
ny thuc v V ./. Trung im ny thuc v @ nu ch c duy nht mt u mt l nm trong
V ./
nh ngha 4. S gc quay W .a; b/ ca mt SAW gia a v b l s ln r sang tri ca tr

i s ln r sang phi ca , ly kt qu nhn vi . khi i t a n b.
3
nh ngha 5. Cho a 2 @; z 2 ; chng ta t
X

F .z/ D F .a; z; x;  / D

i  W .a;z/ l. /

Wa!z

5
B 1. Nu x D xc ;  D , khi F tha mn:
8
8v 2 V ./ W .p

v/F .p/ C .q

v/F .q/ C .r

v/F .r/ D 0

trong p; q; r l 3 cnh lin kt n v:


Chng minh. Ta vit
.p
D.p

v/F .p/ C .q v/F .q/ C .r v/F .r/


X
X
v/
e i  W .a;p/ x l. / C .q v/
e
Wa!p

.r

v/

i  W .a;q/ l. /

Wa!q
i  W .a;r/ l. /

Wa!r

trong p; q; r l 3 trung im theo th t ngc chiu kim ng h xung quanh v:


37

Tp ch Epsilon, S 08, 04/2016


Ta k hiu:
Cp D f   W W a ! pg
Cq D f   W W a ! qg
Cr D f   W W a ! rg
v


Cp3 D 2 Cp W i qua q v r


Cp2 D 2 Cp W ch i qua q v r


Cp1 D 2 Cp W khng i qua q cng khng i qua r

Ta c 3 trng hp:
, c nh ngha bi: e
i qua cng nhng trung im m
1. If 2 Cn3 , ta lin kt n e
i qua v thm vo ng p ! r vo e
. c bit, e
2 Cp3 .
2. nu 2 Cp1 , ta lin kt n Q v QQ i qua 2 trung im (p; q or p; r ) bng cch m
rng ng i thm mt bc.
3. nu 2 Cp2 , n l trong trng hp 2 Cp1 or Cr1 ,
Ta nh ngha: Nu mt ng kt thc ti trung im z, C. / D .z

v/e

i  w .a;z/ l. /

Trong nhng lp lun tip theo, chng ta s xem xt mt vi trng hp:


1. Trng hp u tin, chng ta s chng minh rng
xcl. /  .p

v/e

We have l. / D l. /;
Q .q

i  w .a;p/

v/ D e

2i
3

C .q

.p

i  w .a;q/ l. Q /
xc

v/e

v/ v


w .a; p/ D w .a; r/ C w .r; p/ D w .a; r/ C


w Q .a; p/ D w Q .a; r/ C w Q .r; p/ D w Q .a; r/ C
Do

xcl. /  .p

v/e

i  w .a;p/

C .q v/e
4
2i
3 C e 3 .p

D .p v/e i  w .a;r/
h
5i
2i
D .p v/e 6 C e 3 .p
0
D .p

v/e

5i
8

w .a;r/

D0

v/e
5i
6

 @e

5i
6

4
3

4
4
D w .a; r/ C
3
3

i  w .a;q/ l. Q /
xc

v/e
i

e
1

i  w .a;r/C 4
3

5i
8 w .q;r/

i

Ce 6 AD0

0

2. Trng hp th 2, chng ta s chng minh rng


QQ D 0
c. / C c. /
Q C c. /
Ni cch khc,
xcl. / .p

v/e

i  w .a;p/

C .q

v/e

i  w Q .a;q/ l. Q /
xc

38

C .r

v/e

i  w QQ .a;r/ l. QQ /
xc

D0

Tp ch Epsilon, S 08, 04/2016


Ta c
QQ D l. / C 1
l. /
Q D l. /
v

 
w Q .a; q/ D w Q .a; p/ C w Q .p; q/ D w .a; p/ C
;
3

w QQ .a; r/ D w QQ .a; p/ C w Q .p; r/ D w .a; p/ C
3
2i
2i
q v D .p v/e 3 ; r v D .p v/e 3

Ta vit li nh sau:
xcl. / .p

v/e
2i
3

i  w .a;p/

C .q

i 5
3 8

v/e
2i
3

i  w Q .a;q/ l. Q /
xc

C .r

v/e

i  w QQ .a;r/ l. QQ /
xc

D0

i5
24

, 1 C xc e e
C xc e
e
D0
 7i

7i
, xc e 8 C e 8 C 1 D 0

, 2 cos xC C 1 D 0
8
p
p

1
2C 2
, cos D
D
8
2xC
2
q
p

iu ny l ng bi v 2 C 2 D 2 cos . B c chng minh
8
Ghi ch 1) Chng ta c th thy .1/ ging nh l tch phn ri rc theo ng tam gic trn mt
mng li theo ngha sau y. Gi H  l mng li "dual" ca H . Mt ng:
W f0; 1; 2; : : : ; ng ! H 
(v 2 H  nu v ch nu v l tm ca cc mt ca H ).


I
n 1
X
i C i C1

. i C1
F W H ! C l mt hm trn H: Chng ta nh ngha: F .z/dz WD
F
2
iD0

If .0/ D a; .1/ D b; .2/ D c; .3/ D a v F ging nh trong nh ngha 5, khi ta c








I
aCb
bCc
aCc
F .z/dz D .b a/F
C .c b/F
C .a c/F
2
2
2



D 2e 2 .p
D0

i

v/F .p/ C 2e 2 .q

i

v/F .q/ C 2e 2 .r

v/F .r/

Tng qut hn, nu W f0; 1; 2; : : : ; ng ! H  sao cho .0/ D .n/. Chng ta phn tch thnh
nhng ng tam gic. Ta c:
I
F .z/dz D 0:

2) Nhng mi quan ht ny th khng th xc nh c duy nht hm F nh trong nh ngha,


bi v s bin l ln hn s phng trnh. Chng ta c th a ra hai hm m tha mn mi quan
ht ny:
F .z/ D 0; 8z 2 H
F .z/ D 1; 8z 2 H
39

i /.

Tp ch Epsilon, S 08, 04/2016

4. Chng minh nh l
Chng ta c nh a 2 H ging nh l gc ta ca mt phng phc.Chng ta xem xt mt
min dc  D ST c to thnh t T dy lc gic, v phin bn h hn ST;L ct ti cao L

v gc . Chng ta s xc nh V .ST / v V .ST;L /.
3
nh ngha ; tng ng l nhng bin bn tri, bn phi ST and ;  l nhng bin pha trn
v pha di ca ST;L
Chng ta c
3T C 1
:
2
p
Bin pha trn  thuc v ng thng c phng trnh 3y
8z 2 ; Re z D

1
x D 3L C .
2
p
1
.
Bin pha di  thuc v ng thng c phng trnh 3y x D 3L
2
Do


3T C 1
V .ST / D z 2 V .H / W 0  Re z 
2
p

n
o

V .ST;L / D z 2 V .H / W 3 Im.z/ Re.z/  3L


Chng ta nh ngha
X

AT;L .x/ D

x l. /

ST;L Wa!nfag

BT;L .x/ D

x l. /

ST;L Wa!

ET;L .x/ D

x l. /

ST;L Wa![

5
B 2. For x D xC ;  D , Ta c
8
c AT;L .xc / C BT;L .xc / C c ET;L .xc / D 1
trong c D cos

3

; c D cos .
8
4
5
khi
8
v/F .qv / C .rv v/F .rv / D 0

Chng minh. Chng ta chng minh rng: Nu x D xc ;  D


8v 2 V ./ W .pv

v/F .pv / C .qv

trong pv ; qv ; rv l nhng trung im ca 3 cnh lin kt n v:. Chng ta cng nhng mi


quan h ny trn tt c cch nh v 2 V .ST;L / khi
X
X 2i
X
X 2i
e 3 F .z/ D 0
F .z/ C
e 3 F .z/ C
e i  F .z/ C
z2

z2

X
z2

F .z/ C

z2

2i
3

z2

F .z/ C . 1/

X
z2

z2

c bit,
40

F .z/ C

X
z2

2i
3

F .z/ D 0

Tp ch Epsilon, S 08, 04/2016


 . 1/

F .z/ D

z2

e i  C e
2

i 

AT;L .xc / D

5
AT;L .xc / D
8

cos

1C

3
cos
AT;L .xc /
8
X

F .z/ D e i  0 BT;L D BT;L .
z2


e

2i
3

F .z/ C e

2i
3

z2

De

i
4

F .z/ D e

2i
3

2i
3 

xcl. / C e

Wa!

i
4

xcl. / C e

2i
3

2i
3 

Wa!

z2

xcl. /

Wa!

1  i
e 4 Ce
D
2

xcl. /

Wa!


i
4

ET;L .xc / D cos


ET;L .xc /
4

Do , ta c
cos


3
AT;L .xc / C BT;L .xc / C cos ET;L .xc / D 1:
8
4

Ch . Chng ta nhn thy rng .AT;L .xc //L and .BT;L .xc //L l nhng dy tng, khi
lim AT;L .xc / D AT .xc / and lim BT;L .xc / D BT .xc /:

L!C1

L!C1

ta c
cos

3

AT;L .xc / C BT;L .xc / C cos ET;L .xc / D 1
8
4

v .AT;L .xc //L ; .BT;L .xc //L l nhng dy tng, do ET;L .xc / l mt dy gim, iu ny suy
ra rng
lim ET;L .xc / D ET .xc /:
L!C1

Ta c cos

3

AT .xc / C BT .xc / C cos ET .xc / D 1.
8
4

By gi, chng ta s trnh by phn chng minh nh l.


Trong phn 2, chng ta ch ra rng b .H / D .H /. Do , ch cn chng minh rng
z.xc / D C1 v B.x/ < C1; 8x < xc .
1. Chng minh rng z.xc / D C1:
Chng ta xem xt 2 trng hp:
 If 9T0 W ET0 .xc / > 0 do z.xc / 

C1
X
LD1

.ET0 ;L .xc //L l mt dy gim.


41

ET;L .xc / 

C1
X
LD1

ET0 .xc / D C1. Bi v

Tp ch Epsilon, S 08, 04/2016


 If 8T W ET .xc / D 0, then we have cos

3
AT .xc / C BT .xc / D 1 vi mi T: D
8

dng chng minh rng


AT C1 .xc /

AT .xc /  zc BT C1 .xc /2

do
3
3
AT C1 .xc / C BT C1 .xc / cos
AT .xc / BT .xc / D 0
8
8
3
AT C1 .xc / AT .xc / C BT C1 .xc / BT .xc /
, 0 D cos
8
3
 cos
xc BT C1 .xc /2 C BT C1 .xc / BT .xc /
8

cos

K hiu cos

3
D c , ta c
8
c xc BT C1 .xc /2 C BT C1 .xc /  BT .xc /:

Chng ta s chng minh rng




1
1
BT .xc /  mi n B1 .xc /;
:
c xc T


1
1
Gi s rng tn ti T0 sau cho BT0 .xc / < min B1 .xc /;
, khi chng ta c
c xc T0
c xc BT0 .xc /2 C BT0 .xc /  BT0

1 .xc /

) BT0 1 .xc /  c xc BT0 .xc / C BT0 .xc /






1 2 1
1
1
< min B1 .xc /;
C c xc min B1 .xc /;
c xc T0
c xc T02




1
1
1
1
 min B1 .xc /;
C min B1 .xc /;
c xc T0
c xc T02
0
1
C

B
C
1
1
1 B
B
C
 min B1 .xc /;
B C 2C
c xc B T0
T0 C
@ A
1
0 1

<T

Bng cch tng t, ta c


BT0



1
1
2 .xc / < min B1 .xc /;
c xc T0 2

:::


1
B1 .xc / < min B1 .xc /;
 B1 .xc /
c x c
42

Tp ch Epsilon, S 08, 04/2016


bt ng thc cui cng l mt s mu thun, do


1
1
BT .xc /  min B1 .xc /;
for all T;
c xc T
then

X

C1
1
1
D C1
z.xc / 
BT .xc /  min B1 .xc /;
c
T
xc
T D1
T D1
C1
X

2. Chng minh ca B.x/ < C1; 8x < xc .


Gi s rng x < xc . V BT .x/ l mt cu c di t nht T nn
 T
 T
X
X x
X x
x
x
l. /
l
l. /
T l. /
BT .x/ D
x

. / . /x

. / x

BT .xc / 
xc
xc
xc
xC
bi v BT .xc /  1. Nh vy,
B.x/ D

C1
X

BT .x/ 

T D1


C1 
X
x T
xc

T D1

< C1

bi v x < xc .

5. M hnh vng O.n/ trn mng li lc gic:


Cho  l mt th con ca mng li lc gic, chng ta xem xt cu hnh ca nhng vng n
gin khng t ct . Cho n  0; x > 0: Chng ta nh ngha o trn tp hp X ca cu hnh
nhng vng n khng t ct:
P.!/  n#loop.!/ x #edges.!/ :
K t , mt phn giao gia 2 vng s c thm vo. Trong trng hp ny, cu hnh s c
hp gia cc vng khng t ct v giao din trnh cc vng t a vo b. Chng ta s m rng
nh ngha ca "paraforminoic observable" trong phn 3.
nh ngha 6. Cho u; v 2 @; z 2 ; n 2 0I 2; x > 0, ta nh ngha
P
e i  w .u;z/ x j!j n#loop.!/
F .z/ D F .z; u; v; x; n; / D

!2 .u;z/

i  w .u;v/ x j!j n#loop.!/

!2 .u;v/

trong  .u; v/ l mt tp hp ca nhng cu hnh vi giao din (SAW ) t u n v:


Ghi ch. Cho u; v c nh, chng ta t
X
cD
e

i  w .u;v/ j!j #loop.!/

!2 .u;v/

khi chng ta c
F .z/ D F .z; u; v; x; n; / D

1
c

X
!2 .u;z/

vi quy c 00 D 1 v cho n D 0, ta c:
43

i  ! .u;z/ j!j #loop.!/

Tp ch Epsilon, S 08, 04/2016


 Nu #loops.!/ > 0 then P.!/ D 0.
 Nu #loops.!/ D 0 then P.!/  x #edges.!/ .
Ngha l X D fSAW g.
1
Mnh 2. Gi  l mt min hu hn ca H v a; b 2 @. t x.n/ D p
p
2C 2


3
1
 .n/ D 1
v F l "paraformionic observable" khi
arccos n
4
2
v/F .p/ C .q

.p

v/F .q/ C .r

v
n

v/F .r/ D 0

trong p; q; r l 3 trung im ca nhng cnh lin kt n v:


5
1
Ghi ch: Nu n D 0 then x D p
p ;  D . iu ny c chng minh trong b 1.
8
2C 2
Chng minh. Chng ta c th lin kt cu hnh 1 ; 2 v 3 ( 4 ; 5 ; 6 )

C nh mt trng hp (trng hp 1 ; 2 ; 3 ) Chng ta s chng minh rng:


C. 1 / C C. 2 / C C. 3 / D 0:
Ni cch khc,
xnj!1 j .p
C xnj!3 j .r

x/e

 w

x/e

.u; p/n#loops.!1 / C xnj!2 j .q

 w 3 .u;r/ #loops.!3 /

x/e

 w 2 .u;q/ #loops.!2 /

D0

trong j!2 j D j!3 j D j!1 j C 1 v


#loops.!1 / D #loops.!2 / D #loops.!3 /;
w 2 .a; q/ D w 2 .a; p/ C w 2 .p; q/ D w 1 .a; p/ C

3
2i
v/e 3 ; r v D .p

w 3 .a; r/ D w .a; p/ C
q

v D .p

44

v/e

2i
3

 
;
3

Tp ch Epsilon, S 08, 04/2016


Chng ta vit li
xnj!1 j .p

i  w 1 .u;p/ #loops.!1 /

v/e

C xnj!2 j .q

v/e

i  w 2 .u;q/ #loops.!2 /

C xnj!3 j .r

v/e

i  w 3 .u;r/ #loops.!3 /

D xnj!1 j .p

v/e

C xnj!1 jC1 .p
C

i  w 1 .u;p/ #loops.!1 /

v/e

xnj!1 jC1 .p

2i
3

v/e

i  w 1 .u;q/

2i
3

n#loops.!2 /

i  w 1 .u;p/C 
3
2i

2i
3

xn e

D1Ce
D1
D1
D1

2i
3

i
3

i
3

C xn e
i
4

2i
3

arccos.

n
2

n#loops.!3 /

i
3

N th d chng minh rng 1 C e 3 xn e 3 C xn e 3 e


 n
1
3
and xn bi p
, khi ta c
1
arccos
p
4
2
2C 2 n
1Ce

i

2i


3

D 0. Chng ta thay th  by

i
3

/p

1
p

Ce

2i
3

i
3

e 4 arccos.
i

n
2

/p

1
p

2C 2 n
2C 2 n
1
1
i
n
n
i
e 4 arccos. 2 / p
e 4 arccos. 2 / p
p
p
2C 2 n
2C 2 n


 n 
 n 
1
1
1
1
cos
arccos
cos
arccos
p
p
p
p
4
2
4
2
2C 2 n
2C 2 n

2 cos 14 arccos n2
D0
p
p
2C 2 n

iu ny l ng.

6. Gi thuyt cho m hnh O(n)


nh ngha ca s bin i pha th tng ng n s tn ti ca xc 2 .0I C1/ sao cho
1. Cho x < xc : Xc sut m nh a v b l trn cng mt vng gim nhanh nh ly tha
theo khong cch gia a v b:
2. Cho x > xc : Xc sut m nhng nh a v b l nghch o ca ly tha theo khong
cch gia a v b:
1
Gi thuyt: Cho n 2 2I 2, khi xc .n/ D p
p
2C x

Ti liu tham kho


[1] Hugo Duminil-Copin, Parafermionic observables and their applications to planar statistical
physics models.
45

Tp ch Epsilon, S 08, 04/2016


[2] Hugo
Duminil-Copin, Smirnov: The connective constance of the honeycomb lattice equals
q
p
2 C 2.
[3] Hugo Duminil-Copin, R. Bauerschmidt, J. Goodman, G. Slade, Leture on the self-avoidingwalks.
[4] Hugo Duminil-Copin, N. R. Beaton, Mireille Bousquet-Mlou, Jan de Gier, Anthony J.
Guttmann, The critical
p fugacity for surface adsorption of self avoiding walks on the moneycomb lattice is 1 C 2.

46

RNG HP NH TO VA VN C
(GII THIU TP HC)
Nguyn Hu Vit Hng
(Trng H Khoa hc T nhin, HQG - H Ni)
C nhng vn ca hnh hc, nhng li khng ph thuc vo kch c to nh,
rng hp, di ngn ca cc i tng lin quan. Nhng vn nh th thuc v
mt lnh vc c gi l Tp hc (Topology1 ). Trong nhng vn thuc loi ny,
chuyn mt mnh t rng hay hp, vung hay mo chng quan trng g. (Th c l
khng!) V th, nhng ngi bun t, bun bt ng sn ch nn hc Tp. Nu
nh v t m m h c hc, h th no cng c quyt rng cc nh Tp hc l
nhng k in, hm hp.
Cao m khot lun nh th khng kho d dn n t bin, m m, ri d sinh
ra ni nhm. trnh chuyn , ta hy bt u bng mt vi v d. i khi, vi v
d thc cht c th ra mt l thuyt, c khi cn ra c mt ngnh hc.
Leonhard Euler (1707 - 1783), nh ton hc v i ngi Thu S, c xem l
cha ca ngnh Tp hc, v ng l ngi u tin nghin cu hai bi ton sau y.

1. Bi ton v by chic cu
Konigsberg l mt thnh ph c thuc Vng quc Ph v nc c cho n 1945. Sau i
chin Th gii II, n thuc Lin X (c) ri Nga, v c gi l Kaliningrad. Ch c rt t du
tch ca Konigsberg cn st li ngy nay Kaliningrad.
thnh ph Konigsberg, c 7 chic cu. Chng ni hoc l hai b sng, hoc mt b sng v
mt trong hai c lao, hoc ni hai c lao . Xem bn sau y:

Tt c cc lin kt trong bi u ca Ban Bin tp.

47

Tp ch Epsilon, S 08, 04/2016


T xa, c dn Konigsberg t cu hi: Liu c th i mt ln qua tt c 7 chic cu m
khng c cu no phi lp li hay khng?
Khng cn tm nhiu lm n v tr c th ca 7 chic cu. iu quan trng nht m ngi
ta quan st c t bi ton ny l nh sau: y l mt vn ca hnh hc, nhng khng ph
thuc vo ln ca cc yu t tham d (dng sng rng hay hp; nhng chic cu di hay ngn,
to hay b; cc c lao ln nh th no). Vn ch ph thuc hnh dng v v tr tng i ca cc
yu t.

Khng c bng chng no cn li chng t rng Euler ti Konigsberg. Tuy nhin, nm 1735
ng chng minh rng mong mun tm mt cch i qua c 7 chic cu mt ln, khng lp li
l khng th thc hin c.
Chng ta th tm hiu li gii ca Euler cho bi ton 7 cy cu. Trn bn Konigsberg hy thay
mi b sng, mi c lao bng mt im, gi l nh, thay mi chic cu bng mt ng ni cc
nh, gi l cnh. Hnh thu c gi l mt th. Bi ton v 7 ci cu Konigsberg thc cht
l chuyn c gng v bng mt nt th sau y:

Bi ton ny rt quen thuc vi tr em qua tr chi v hnh bng mt nt. C ai thu thiu thi
li chng tng au u vi cu v ci phong b ch bng mt nt?

48

Tp ch Epsilon, S 08, 04/2016

Hy bt u vi nhn xt n gin sau y: Mi khi ta i qua mt nh, th c 2 cnh (2 cy cu)


xut pht t nh c i qua: Cnh i ti, v cnh i ra khi nh . Nh th, mi ln i
qua mt nh, s cnh ni vi nh m ta cha i qua gim i 2. Cho nn, nu mt nh c s
cnh ni ti l mt s chn (gi tt l nh chn) th mi ln i ti ta lun cn ng thot
ra ngoi. Cn ti mi nh l, chng hn c (2k C 1) ng ni vi nh , th sau k ln i qua,
ti ln (k C 1) ta s ht ng i khi nh .
Nh vy, cc nh l chnh l cc cn tr cho vic i qua m khng phi dng li. Chin
thut ca ta l khng xut pht t cc nh chn (nu vn cn nh l), v nu xut pht t mt
nh chn, khi i khi nh , chng ta s bin nh chn ny thnh mt nh l trong phn tip
theo ca tr chi.
Ta ch cn xt cc th lin thng, ngha l th m gia 2 nh bt k ca n u c t nht
mt ng ni. (Vic v mt th khng lin thng hin nhn qui v v tng thnh phn lin
thng ca n.) Da trn nhng nhn xt v nh chn v nh l ni trn, ta c th chng minh:
(1) Trong mi th, s cc nh l lun l mt s chn,
(2) Mt th lin thng khng c nh l no, cn ti thiu 1 nt v.
(3) Mt th lin thng c 2n nh l (n > 0), cn ti thiu n nt v.
Cch v nh sau: Xut pht t mt nh l bt k (nu c), v tu cho n khi khng v c
na. Khi ta gp mt nh l khc. Nt v va ri kh bt 2 nh l (l im u v im cui
ca nt v). Lp li qu trnh trn cho n khi khng cn nh l no. Trng hp khng c nh
l no, hy xut pht t mt nh chn bt k, v tu cho n khi khng v dc na. Khi ta
gp li nh xut pht.
Chng ti khng i su vo chi tit chng minh nhng khng nh trn.
Ci phong b c 5 nh, trong 4 gc l 4 nh l. V th, khng th v phong b bng 1 nt.
Cn t nht 4=2 D 2 nt v c phong b.
Trong bi ton 7 cy cu Konigsberg, c bn nh, u l cc nh l. Do , khng th v
th bng 1 nt. Ti thiu cn 4=2 D 2 nt. l l do trong sut chiu di lch s, khng
ngi dn no Konigsberg c th i mt ln qua tt c cc cy cu m khng chic cu no b
lp li.
49

Tp ch Epsilon, S 08, 04/2016

2. Bi ton v s mt, s cnh, v s nh ca mt a din


L. Euler chng minh nh l sau y, thot nhn tng nh tr chi tr con: Trong bt c a
din li no, s mt tr i s cnh cng vi s nh u bng 2.
Hy ly vi v d.

Trong mt t din, s mt m D 4, s cnh c D 6, s nh d D 4;


Ta c m c C d D 4 6 C 4 D 2.

Trong mt hnh hp ch nht, s mt m D 6, s cnh c D 12, s nh d D 8;


Ta cng c m c C d D 6 12 C 8 D 2.
V sao li c chuyn lc th ly du cng, lc li ly du tr trong nh l trn? Xin tha:
Mt l mt yu t 2 chiu, nh th 0 chiu, nhng yu t chn chiu th c mang du cng;
cn cnh l mt yu t 1 chiu, tc s chiu l, nn n mang du tr.
Ging nh bi ton v 7 chic cu, bi ton ny cng l mt vn hnh hc, nhng khng ph
thuc vo ln cc yu t. Tht vy, mt a din d b nh ht u hay to nh tri t th s
mt, s cnh, v s nh ca n cng khng thay i. Nhn xt trn gi cho suy lun sau y:
Hy tng tng a din li c lm bng cao su. Ta hy thi phng a din li thnh mt
qu bng hnh cu.
50

Tp ch Epsilon, S 08, 04/2016

Cc mt, cnh, v nh ca a din bin thnh cc mt (cong), cnh (cong), v nh trn mt cu.
Nh th, nh l trn ca Euler v bn cht l mt nh l v mt cu: Trong mi cch phn mt
cu thnh cc hnh a gic cong, s mt tr s cnh cng s nh u bng 2. Hn na, mi
hnh thu c t mt cu bng mt php bin i lin tc (tng t nh co dn mng cao su) u
nghim ng nh l ny.
Chng ta va t c mt bc tin quan trng trong cch ngh: Bi ton ca Euler ban u xt
rt nhiu i tng, l bt c a din li no. Rt cuc, n l mt bi ton v ch mt i tng
duy nht, l mt cu.
t c bc tin l do chng ta s dng lp lun v cc bin i kiu co dn cao su.
Ngi ta gi l cc php bin i tp.

By gi, thay cho mt cu ni trn, hy ly mt xuyn (ci sm t) lm th nghim. C th


phn chia ci sm bng 2 ng (c D 2), mt ng ct theo vt mng-xng, ng kia ct dc
ton b chiu di xm. Hai ng ny ct nhau ti mt im duy nht (d D 1). B ct hai ng
, sm tr thnh mt mt hnh ch nht (m D 1). Vy, s m Euler quan tm ca mt xuyn
(sm) l m c C d D 1 2 C 1 D 0.

51

Tp ch Epsilon, S 08, 04/2016


Tip theo, hy ly mt mt xuyn kp, c c bng cch dnh 2 chic sm t vo nhau. Bn
hy t chn mt cch chia mt xuyn kp thnh cc mt ging nh hnh vung (cong), cc
cnh (cong), v cc nh. Chng hn, ta chn cch chia m t bng hnh v trn. Cc ng
v vng ct nhau 2 im (1 im nhn thy, 1 im l hnh chiu thng ng ca im nhn
thy), vy d D 2. Cc ng v vng c 2 nh y chia lm 4 cnh (4 na ng trn),
cng thm 2 ng mu xanh, vy c D 6. Sau khi ct theo cc ng y mt xuyn kp b chia
thnh ra 2 hnh ch nht, vy m D 2. Ta c m c C d D 2 6 C 2 D 2. Nh th, s Euler
ca xuyn kp l 2.
Ta c th dnh nhiu sm t vi nhau to thnh mt mt xuyn c g l. S m Euler quan
tm i vi mt bng m c C d D 2.g 1/.
Trong tp hin i, nh l Euler c tng qut ho nh sau: Nu chia bt c mt vt th n
chiu no thnh cc phn ging nh a din, th tng s cc phn vi chiu chn tr i tng
s cc phn vi chiu l lun l mt hng s, c gi l c s Euler, ca vt th .
Nh th, mi vt th u l s tng ho nhp nhng ca hai phn m v dng, chn v l ni ti
ca n, khng th thay i. c s Euler, cng cn c gi l c s Euler Poincar (bi v
Poincar (1854-1912) chnh l ngi u tin thc c chuyn ny trng hp s chiu tu
), ca mt vt th chnh l mt loi bn th, mt loi chng minh th, mt ID Card
ca vt th y.
H qu l, nu hai vt th c c s Euler khc nhau, th chng khng th ci ny bin thnh ci
kia sau mt php bin i thun nghch lin tc (kiu nh co dn cao su). Ngi ta ni hai vt
th khng cng kiu tp.
Nh th, mt cu, mt xuyn, v mt xuyn kp khng cng kiu tp, v chng c c s Euler
khc nhau (tng ng bng 2, 0, v 2). V mt trc gic, v sao chng khng cng kiu tp?
L do tht n gin: Mt cu khng c l no; mt xuyn c 1 l (l ci ch ngi ta vn chui
vo bin sm thnh phao bi); cn mt xuyn kp c 2 l. Cc nh tp bo mt xuyn c 1
l, nn c ging (genus) bng 1; mt xuyn kp c 2 l, nn c ging bng 2; mt cu khng c
l no, nn c ging bng 0. ra th, phi c l th ging mi khng b trit tiu. Cc nh
tp tht gii m .
Riemann cn chng minh mt nh l tht thm thu: Mi mt 2 chiu trn (tc mn mng), b
chn (c th gi trong mt cn phng), v c hng (tc l phn bit c pha no l ngoi da,
pha no l trong tht) u xc nh c v mt tp ch bng cch m s l trn n. Ch ch,
phi mi Picasso n y mi c.
Nhng chuyn k trn c th dn chng ta n nhng g? Sau y l mt kt lun tht kh tin.
Khng nh: D c nho nn mt cc bt, hnh ci bnh m, k n mc no, min l hnh ca
cc bt lc u v khi thi nn vn l ci bnh m, nn xong li ci bnh vo ng ch c,
khi lun lun c mt ht bt m khng thay i v tr.
Tht vy, sau mt php bin i lin tc 2 chiu, cc bt hnh ci bnh m c bin thnh mt
hnh cu B. Gi S l mt cu bao quanh B. Khng nh trn c chng minh bng cc bc
suy lun sau y:
52

Tp ch Epsilon, S 08, 04/2016


1) Khng c php bin i no bin B thnh S v vn gi nguyn mi im trn S . ( chng
minh: Gi s tn ti mt php bin i nh th. Trc php bin i, S l bin ca B, sau php
bin i S phi l bin ca chnh S. iu ny v l.)
2) Gi s phn chng, sau nho nn khng c ht bt m no gi nguyn v tr. Gi s ht bt
m x c bin thnh f .x/ sau nho nn. Na ng thng ni f .x/ vi x (ko di) ct mt
cu S ti mt im duy nht, k hiu g.x/. Php bin i x thnh g.x/ chnh l mt php bin
hnh lin tc, bin B thnh S v gi nguyn mi im trn S . (Nu x nm trn S, th na ng
thng ni f .x/ vi x ct S ti chnh x.) iu ny mu thun vi im 1). Mu thun ny bc b
gi thit phn chng.

Hai bi ton c Euler nghin cu ni trn l nhng v d n gin ca cc vn hnh hc


trong kch c khng quan trng, ch c hnh dng v v tr tng i ng vai tr quyt nh.
Ngnh ton hc nghin cu nhng vn nh vy ngy nay c gi l Tp hc (Topology).
Ngm cho k th chuyn kch c khng quan trng c to ho duy tr nh mt trong nhng
nguyn l hng u, ng vai tr m bo an ninh khng ch cho x hi loi ngi, m cho
ton b cc ging loi trong t nhin. Nu mt ngi mua nhm i giy, cht qu hay rng qu,
tc l ngi y gp mt vn v kch c, th anh ta em i. Th nhng, nu ngi y ly v, v
nu nh anh ta cng gp mt vn v kch c, ri i i, th nguy him v cng. V nu rt
nhiu ngi sau khi ly v cng gp vn v kch c nh th, u cn phi i, th x hi chc
chn sinh lon.
B cha th nm H Xun Hng (17721822) chnh l nh Tp hc u tin ca Vit Nam,
ngi bng trc cm tuyt vi pht biu tng minh nhng tng to bo ca tp t hn
200 nm trc. Khng nghin cu bi ton v 7 ci cu hay bi ton v s mt s cnh s nh
trong a din, nhng bng mt tip cn y mn cm, b nhn ra chuyn ny t xa. B vit
tht nhn vn:
Rng hp nh to va vn c
Ngn di khun kh cng nh nhau.
Hai cu th trch trong bi Dt ci ca b:
Thp ngn n ln thy trng phau
Con c* mp my sut canh thu
Hai chn p xung nng nng nhc,
Mt sut m ngang thch thch mau,
53

Tp ch Epsilon, S 08, 04/2016


Rng hp nh to va vn c
Ngn di khun kh cng nh nhau
C no mun tt ngm cho k
Ch n ba thu mi di mu.
Nh th, H Xun Hng (17721822) c lp v gn nh ng thi vi L. Euler (1707-1783),
pht biu tng minh quan im c bn ca tp hc. N s h H qu l khi u y
sinh kh cho m hu sinh lm tp ca Vit Nam, trong c k hc tr vit bi ny:
Mt mt anh hng khi tt gi
Che u qun t lc sa ma.
Theo c H n s qu l kh. May sao,
"Mi gi chn chn vn mun tro".
Vy m li bo cc nh Tp l hm th nghe th qui no c, h gii.
V thanh:
Lo C-nh-t c xong bi ny ci ph ln, m rng: C mnh t cng khng bit n to
hay b, vung hay mo, li bo rng nh nhau tut. Th th ngho sut i l phi. Bn tp ny
xem ra ch lo chuyn ging thi.

54

CC BI TON ON BI
ng Nguyn c Tin
(i hc Trento, Italy)
Sn khu bng sng trong ting h reo vang di.
V nh o thut hin ra, sang sng ni.
Cc bn, cc bn ca ti i, tr o thut bi hay nht m nay ang ch i. y,
hy chn 5 l bi ngu nhin. Chn no, bn ti i, ngu nhin cng c hay suy
t cn trng u cng chng sao.
Chn i no, tht kn o khi mi con mt th gian.
La chn xong hy giao cho ngi tr l ca ti nhng l bi ca bn. Ri anh ta s
a li bn l trong s cho ti.
Xem no, tng l mt:
By bch, m c, tm chung, cui cng l ba r.
Gi trn tay ngi tr l ch cn li mt, duy nht mt l m thi. Mt l bi m ch
anh ta v ngi la chn bit l g.
Nhng con mt ca nh o thut, c c thu tm can.
Bn ti i, chnh l gi bch!

1. Nm l bi ca Fitch Cheney
Tip ni nhng s trc, chuyn mc gii tr k ny trn trng gii thiu vi c gi mt lot cc
bi ton , v k ny l cc bi ton on bi. Nh thng l, chng ti bt u chuyn mc
bng bi ton kinh in nht v bi ton khi u ca ln ny l bi ton nm l bi ca Fitch
Cheney. Bi ton ln u tin c in trong quyn Math Miracle ca tc gi Wallace Lee vo
nm 1950. Trong quyn ny, tc gi ghi nhn tc gi bi ton l William Fitch Cheney (1894
- 1974), vn c gi mt cch thn mt l Fitch - nh o thut. Theo tc gi, tr o thut ny
c Fitch sng to vo khong nhng nm 1920 v chng ti mn chi tit ny vit li
thnh li ta cho chuyn mc k ny. n gin hn cho vic tip cn bi ton, chng ti pht
biu li bi ton dng ton nh sau:

Hai ngi A v B tham gia mt tr chi nh sau: A c nhn ngu nhin 5 l bi t b bi


chun 52 l v B khng bit A nhn nhng l bi no. Sau khi xem xong, A c php gi
li 1 l ty v a ln lt 4 l cn li cho ngi qun l tr chi. Ngi qun l s ln
lt t cc l bi vo cc v tr c nh s cho trc t 1 n 4. B nhn vo 4 l bi
ca A v c yu cu on l bi cn li, nu on ng, h thng tr chi, nu on sai,
h tht bi. Tm chin thut cho 2 ngi h lun lun chin thng.
55

Tp ch Epsilon, S 08, 04/2016


Thot nhn, iu ny l khng th v vi 4 l bi, chng ta ch c th t hp thnh 4 D 24 trng
hp khc nhau, trong khi b bi chun c 52 l. Tuy nhin, vi mt cht my m v kin nhn,
hn bn c s nhanh chng tm ra cch thc khm ph ra b mt on bi ca nh o thut.
y, chng ti gii thiu mt cch gii n gin nh sau:
- V c 5 l bi, nn chc chn tn ti t nht 2 l bi ng cht (cng l c, cng r, cng chun
hoc cng bch). Gi 2 l ny l M v N .
- V mi cht bi c 13 l khc nhau, nn ta lun tm c cch m ti khong cch ca 2
l ti a l 6 (v ti thiu l 1 do M v N cng cht, khng th bng nhau). V d nu M D 7
v N DBi th khong cch l 4 v sau 7 l 8; 9; 10 v bi, 4 l. Nu M D bi v N bng 3 th
khong cch l 5 v sau bi l m, gi, t, hai, ba, 5 l. Khng mt tnh tng qut, ta gn M cho
l u tin v N l l c m ti.
- Chin thut l gi li l N v t l M v tr s 1. 3 l cn li, lun c th t ln nh ngm
nh trc (v d th t chi tin ln nh c ln hn r, r ln hn chun..) nn c th to thnh
6 hon v khc nhau, mi hon v ng vi 1 s cho bit khong cch ca M v N .
- Nh vy, vi cch hon v 3 l cn li v tr 2; 3; 4 v l u tin l l M , ta bit c cht
ca N v khong cch t M n N , nn xc nh c N .
Liu rng ta c th lm tt hn, ngha l c th tm ra l bi bng 4 l cn li trong mt b bi
ln hn, v d 100 l thay v ch 52 l chng hn?
Hy quay li tr o thut vi ci nhn ton hc hn mt cht: chng ta hy xem mt b 4 l nhn
c t tr l l mt thng ip, do vy c 52  51  50  49 thng ip khc nhau. Nh o thut
thy 4 l bi v phi on l cn li, iu
! ny ngha l nh o thut ch cn phi on trong t
52
hp chp 5 ca 52 trng hp. V
D 52  51  50  49  48=5 nn s lng thng ip
5
m nh o thut nhn c l qu d so vi s trng hp, gp 5=48 D 2:5 ln. V tht s l ta
c th gii quyt bi ton vi 124 l bi ch khng phi ch vi 52 l.
Chin thut c th vi 4 l bi c th tm ra l cn li trong s 124 l bi chng ti xem nh
mt bi tp dnh cho c gi. Gi : da trn s d khi chia cho 5.
Tng qut ha bi ton, cho m l bi ly t n, c th chn gi li m
cn l cn li khi v ch khi n  m C m 1.

1 l t xc nh

Tip tc m rng, vi b ba .m; n; k/, n > m > k, vi m l bi ly ra t b bi n l, c th chn


gi li k l sao cho t xc nh c m k l cn li khi v ch khi:
!
n
 n.n 1/.n 2/ : : : .n k C 1/:
m
Vi bi ton 5 l bi ca Fitch Cheney tng ng b 3 .52; 5; 4/:
Vic chng minh chi tit kt qu ny nm ngoi khun kh ca mt bi ton gii tr, chng ti
ch gii thiu vn tt thng qua l thuyt th nh sau: Xt mt th hai pha G vi hai tp
phn bit S v T trong S l tp tt c cc b m l bi ly t n l v T l tp tt c cc cch sp
xp k l. Mt cp ghp trn th G chnh l chin thut cn tm. Theo nh l Konig-Egervry,
56

Tp ch Epsilon, S 08, 04/2016


ln cc i ca mt cp ghp trn th hai pha bng ng kch thc ca tp nh ph nh
nht (c th dn v bi ton lung cc i - lt ct cc tiu), v iu kin ti u s t c khi
jS j D jT j.

2. Bi ton 55 l bi
Trong lot 3 bi ton tip theo, chng ti s trch chn gii thiu t tc gi quen thuc ca chuyn
mc ton hc gii tr, Stan Wagon qua cc bi ton s 922, 1219, v 1223.
V y, bi ton 922, bi ton 55 l bi:
C 55 l bi, c gn cc s khc nhau t 1 n 55 v t ngu nhin, p mt, thnh vng
trn. C mi ln ta s c lt mt l bi v xem s trn . Tm chin thut lt bi t nht
sao cho tm c 1 l bi chc chn c gi tr ln hn l bi bn tri v bn phi ca n.

Bi ton ny c Stan Wagon gii thiu nm 2000, v theo ng n bt ngun t bi ton


Sharygin ng trn trang cut the knot.
p n ca bi ton l cn ti a 10 ln m bi, ta s lun tm c mt l c gi tr ln hn hai
l hai bn.
Chng ti gii thiu y mt li gii nh sau:
- Gi f .x/ l gi tr ca l bi ti v tr x. Khi , xt mt b 3 v tr .m; n; k/ bt k, ta lun tm
c mt l bi ln hn 2 l cn li. Khng mt tnh tng qut, gi s l bi l n, l bn tri l
m v bn phi l k (do cc l bi xp thnh vng trn nn ta lun tm c b 3 tha mn iu
kin ny). Khi f .m/ < f .n/ v f .n/ > f .k/. Ta gi mt b 3 c tnh cht nh vy l tnh
cht S , ngha l l gia c gi tr ln hn 2 l bn tri v phi.
- Gi s jn mj > 1, ngha l 2 l m v n khng nm k nhau. Chn mt l t bt k gia m v
n. C hai trng hp: Nu f .t/ > f .n/, ta c b 3 .m; t; n/ c tnh cht S . Nu f .t/ < f .n/,
ta c b 3 .t; n; k/ c tnh cht S . Nh vy, d trng hp no th b 3 mi to vi t u c cc
ch s gn hn so vi b 3 .m; n; k/. V c lm nh vy, cui cng ta s thu c mt b 3 m
khong cch cc l bng 0, hay ni cch khc, l 3 l lin tc tha mn tnh cht S , cng chnh l
yu cu ca bi.
- Tip tc n gin ha, vi mi b .m; n; k/ ta c khong cch gia cc l l fn m 1; k n 1g,
tc l s l bi gia cc l ca b 3 ny. Nh vy mc tiu ca bi ton l tm ra b 3 c khong
cch f0; 0g.
- rt v c f0; 0g trong tnh hung xu nht th trc s l f0; 1g (chng ta ang xt
trong tnh hung xu nht, khng quan tm n may ri). Ln ngc tip tc, ta c b ln nht
trong tnh hung xu nht c th a v f0; 1g l f1; 2g. Lu , y, khng mt tnh tng qut,
ta thy khong cch tri hay phi l nh nhau, ngha l f1; 2g tng ng vi f2; 1g.
- Tip tc ln ngc ta c: f0; 0g

f0; 1g

f1; 2g

f2; 4g

f4; 7g

f7; 12g

f12; 20g.

- Nh vy, khi im ta c th chn l bi a bt k, sau chn l bi b cch a 12 l bi gia,


v sau chn tip l bi c cch b 20 l. Khi , ta c nu f .a/ hoc f .b/ l l ln nht trong 3
57

Tp ch Epsilon, S 08, 04/2016


l th ta ch m thm ti a 6 l na l s chc chn a v c 0; 0. Nu f .c/ ln nht, ta tn
thm mt ln m a 20; 20 v 12; 20. Trng hp ny ta cn ti a 7 ln m bi.
- Vy trong tnh hung xu nht, ta tn ti a 10 ln m bi.
V, vi cch lm nh trn, nu ta s thy s l bi ti a vi n ln lt bi ta s chc chn tm
c mt l bi c gi tr ln hn hai l bn cnh chnh l s Fibonacci th n.

3. Bi ton 1219
Bi ton ny c Stan Wagon pht biu nh l mt bi ton v cai ngc, y chng ti pht
biu li vi dng tr chi on bi, nh sau:
A v B tham gia mt tr chi vi lut nh sau:
A v B c mi vo mt phng ring ch gi ra tham gia tr chi.
u tin, ngi dn tr s mi A ra v cho A xem mt b bi chun 52 l c xp nga
mt thnh mt hng. Sau khi xem xong, A c quyn chn 2 l bi bt k, nu mun v i
ch 2 l ny. Sau , ngi dn tr lt p li ton b cc l bi (gi nguyn th t) li v mi
B ra. Ngi dn tr s ni tn mt l bi T ty , v cho php B lt cc l bi ln tm ra l
bi T ny. Nu sau ti a 26 ln lt bi B tm c l bi T , h chin thng tr chi. Ngc
li, nu sau 26 ln lt bi m B khng tm ra l T , h tht bi.
A v B trong qu trnh chi khng c c bt c trao i g vi nhau, v h ch c trao
i chin thut vi nhau trc khi chi. Hy ch ra chin thut cho A v B h lun c th
thng tr chi.
S 26 trong bi l mt trong nhng gi quan trng trong vic tm ra chin thut. Trong phn
ny, chng ti gii thiu mt chin thut da trn chu trnh nh sau:
- u tin A v B ngm nh nh s cc l bi t 1 n 52. Khi B c ngi dn tr cho bit l
cn tm l l bi T , B s m l v tr T . Nu l ng l l T , B tm ra v dng. Ngc li,
nu l c gi tr f .T / T , B s m tip l bi v tr f .T / tng ng v tip tc nh vy
cho n khi gp T .
- Mt chui cc l bi nh vy (bt u t mt v tr x, sau m tip v tr f .x/ trn l bi ti x
v tip tc cho n khi gp l x), c gi l mt chu trnh. V v ton b b bi ch c 52 l,
nn ch c th tn ti nhiu nht mt chu trnh c di hn 52=2 D 26. Do vy, chin thut ca
A l tm ra chu trnh di hn 26, nu c, v i ch 2 l bi ct ngn chu trnh ny thnh 2 chu
trnh khng vt qu 26.
- Vic "ct" chu trnh ca A thc hin kh n gin nh sau: khng mt tnh tng qut, gi s
chu trnh di hn 26 l 1 2 3    N (ngha l f .1/ D 2; f .2/ D 3; : : : ; f .N / D 1),
lc ny ta ch n gin hon i v tr l th 26 v l th N th ta s c 2 chu trnh ngn hn l
1 2 3    N v 27 28    26.
Mt cu hi kh hn c chng ti xem nh mt bi tp dnh cho c gi l vi cch lm nh
chin thut trn, k vng B tm ra l bi T ngu nhin bt k trong s 52 l bi E.52/ l
bao nhiu? V tng qut vi n l bi th k vng E.n/ l bao nhiu? V d ta c E.2/ D 1,
E.3/ D 11=9.
58

Tp ch Epsilon, S 08, 04/2016

4. Bi ton 1223
Bi ton cui cng m chng ti gii thiu chuyn mc ln ny l mt pht trin ca bi ton
trc, pht biu nh sau:
A v B tham gia mt tr chi vi lut nh sau:
A v B c mi vo mt phng ring ch gi ra tham gia tr chi.
u tin, ngi dn tr s mi A ra v cho A xem 5 l bi, nh s t 1 n 5 v c xp
nga mt thnh mt hng. Sau khi xem xong, A c quyn chn 2 l bi bt k, nu mun v
i ch 2 l ny. Sau , ngi dn tr lt p li ton b cc l bi (gi nguyn th t) li v
mi B ra. Ngi dn tr s ni ngu nhin mt s nguyn T ty trong khong t 1 n 5
v yu cu B tm l bi c s tng ng trong s cc l bi ang lt p. Nu sau ng 1 ln
lt bi B tm c l bi T , h chin thng tr chi. Ngc li, h tht bi.
A v B trong qu trnh chi khng c c bt c trao i g vi nhau, v h ch c trao
i chin thut vi nhau trc khi chi. Hy ch ra chin thut cho A v B kh nng chin
thng tr chi l cao nht.

Ta c th thy nu B chn ngu nhin 1 l, khng quan tm n A, kh nng h thng tr chi l


1=5. Nu A lun lun i ch l bi sao cho l 1 s nm v tr 1 v chin thut ca B l nu
T D 1 th s m l v tr 1, ngc li m ngu nhin cc l v tr t 2 n 5. Lc ny, xc sut
chin thng l 2=5 D 40%.
Nu A v B s dng chin thut ct chu trnh nh bi ton trc, xc sut chin thng ca h l
bao nhiu? Ta thy, vi chin thut ngy th B s m l bi v tr T khi ngi dn tr cho bit
l cn tm l T . Stan Wagon, do vy, k hiu cho chin thut ny l (12345) ng vi cc v tr
s m bi tng ng. Chin thut ca A nh sau: nu A gp 1 cp b hon i v tr, A s hon
i v tr cp ny li cho ng. Ngc li (ngha l khng c cp b hon i v tr), A s tm
chu trnh c di k  3 v ct thnh chu trnh ny, nu c, thnh chu trnh ngn hn c di
k 1 v mt l ri. D thy, nu khng c chu trnh k, cc l bi nm ng v tr t 1 n 5.
Vi cch lm ny c tt c 284 trng hp B s tm ng l T trong s 5:5 kh nng, do vy xc
284
1
1
sut h thng tr chi l
D 47 %, ln hn 7 % so vi cch lm trn.
5:5
3
3
Stan Wagon a ra cch tm cng thc tng qut tnh s trng hp thnh cng f .n/ vi n l
bi vi chin thut ny l:
f .n/ D 2:n 1 C T .n/
vi T .n/ l s lng cc hon v ca n s nguyn u tin trong c tn ti t nht mt cp b
hon i v tr. Mt vi gi tr T .n/ u tin l 0, 1, 3, 9, 45, 285, 1995. Trong trng hp u
bi n D 5, ta c f .5/ D 2:5 1 C T .5/ D 2:120 1 C 45 D 284. Chng minh chi tit ca
T(n) v f(n) c th xem thm ti trang nh ca Stan Wagon.
Tuy nhin, cng thc tng qut trn khng phi l im th v ca bi ton m l kt qu bt
ng sau y: chin thut (11345) s c kh nng thnh cng cao hn chin thut (12345)! V y
cng chnh l chin thut ti u. Chin thut ny tm tt n gin l sau khi nghe ngi dn tr
ni l bi T , B s lt l bi v tr T tng ng, tr trng hp T D 2 th B s lt l bi v tr 1.
59

Tp ch Epsilon, S 08, 04/2016


2
Tng s kh nng thnh cng theo chin thut ny l 286 v do vy xc sut thng li l 47 %,
3
1
cao hn % so vi chin thut (12345). Chin thut ny c Stan gi l double-door (ca i).
3
Mt cu hi m vn cha c gii p l vi b bi chun 52 l, chin thut no l tt nht? V
vi n tng qut? Hin ti, kt hp kho st bng my tnh, Stan Wagon, tc gi ca bi ton cng
ch mi chng minh v xut c cch lm ti u cho n trong phm vi t 1 n 10.

Tham kho v trch dn


c gi c th tham kho chi tit hn cho cc bi ton gii thiu chuyn mc ln ny cc a
ch v ti liu sau:
1. Cc bi ton ca Stan Wagon ti http://mathforum.org/wagon/
2. Kleber, M., The Best Card Trick. Mathematical Intelligencer, 24 (2002).
3. Simonson, S. and Holm, T., Using a Card Trick to Teach Discrete Mathematics. Primus:
Problems, Resources and Issues in Mathematics Undergraduate Studies, 13 (2003):248-269.

60

XUNG QUANH NH L BROKARD


Nguyn Trn Hu Thnh
(Trng THPT Chuyn L T Trng, Cn Th)
Bi vit ny s ni v cc vn xoay quanh nh l Brokard quen thuc v c
trnh by bng cc cng c hnh hc phng thun ty.

1. M u
nh l Brokard [1] ni v tam gic c c cc nh l giao im ca cc cp ng thng to
bi mt t gic ni tip v nhn tm ngoi tip ca t gic y l trc tm. Bn thn Brokard cng
l mt trong nhng vin ngc qu v c nhiu ng dng trong cc cuc thi hc sinh gii cc
nc. Sau y ti xin trnh by li mt cch chng minh nh l ny thng qua mt b c kh
nhiu ng dng nh sau:
nh l Brokard. Cho t gic li ABCD khng l hnh thang ni tip ng trn tm O.
Gi E; F; G ln lt l giao im ca AB v CD, AD v BC , AC v BD. Khi O l trc
tm ca tam gic EF G. Li gii sau c rt ra t tng ca thy Thanh Sn v vn
giao im ca cc tip tuyn trong mt ng trn.
Ta c b sau:
B 1. Gi H; I; J; K; L; M theo th t l giao im ca cc cp ng thng .dA I dB /;
.dA I dC /; .dA I dD /; .dB I dC /; .dB I dD /; .dC I dD /1 . Khi cc b im .I I EI LI F /, .F I M I GI H /,
.EI KI GI J / thng hng.
Chng minh. Do vai tr ca E v F l bnh ng trn ng thng IL nn ta c th gi s E
nm trn on thng IL. Gi E l giao im ca AB v IL. p dng nh l Menelaus cho tam
gic IHL vi ct tuyn ABE c:
EI BL AH
EI
:
:
D 1 hay
D
EL BH AI
EL

AI
BL

Gi E 0 l giao im ca CD v IL. Tng t ta cng c:


E 0I
E 0L

CI
D
DL

AI
BL

iu ny chng t E v E 0 chia trong on IL theo cng t s. Do E trng E 0 . Nn vi E l


giao im AB v CD th I; E; L thng hng.
Tng t I; L; F thng hng. Vy b im .I I EI LI F / thng hng.
1

tin cho vic chng minh, ta quy c dK l tip tuyn ti K ca ng trn i qua K.

61

Tp ch Epsilon, S 08, 04/2016


Vi cch chng minh nh vy ta cng suy ra c cc b im .F I M I GI H /, .EI KI GI J /
thng hng.
Mt cch khc, ta c th p dng nh l Pascal cho lc gic suy bin thnh t gic ABCD
suy ra cc b im nu trn thng hng.
Quay li bi ton,

Chng minh. Ta c BOCK v AODJ l nhng t gic ni tip. Mt khc do t gic ABCD
ni tip nn:
F C :FB D FD:FA
Nn F nm trn trc ng phng ca .BOCK/ v .AODJ / nn OF ? KJ . p dng b d
c EG ? OF .
Tng t F G ? OE. Do O l trc tm ca tam gic EF G.
Vy ta c iu phi chng minh.
Nhn xt. Vic chng minh cc b im thng hng nh th gip hng gii quyt c sng
sa hn v a bi ton tr v vi mt tnh cht quen thuc v s vung gc gia ng ni tm
v trc ng phng ca hai ng trn. Vi bi ton th v nh vy ta s xt n nhng m rng
v ng dng quan trng ca n trong cc bi ton quen thuc v thi hc sinh gii ca mt s
nc.

2. Khai thc nh l
Trong b trn ta thu c ba b im thng hng v c nhng tnh cht kh p. Mt tnh cht
c suy ra trc tip ca b d ny c a vo bi hnh trong k thi China MO 1997. Ta
xt bi ton
62

Tp ch Epsilon, S 08, 04/2016


Bi ton 1 (China MO 1997). Cho t gic ABCD vi cc cnh i mt khng song song ni
tip. Gi P; Q ln lt l giao im ca AB v CD, AD v BC . Gi QE; QF ln lt l tip
tuyn ti E; F ca ng trn ngoi tip t gic ABCD. Chng minh rng P; E; F thng hng.
Ta thy nu O l tm ng trn ngoi tip ca t gic ABCD th OQ ? EF . Mt khc theo
nh l Brokard th P G ? OQ vi G l giao im ca AC v BD. Do nu E; G; F thng
hng th P; E; F thng hng, ta xt b sau
B 2. Cho t gic ABCD khng l hnh thang ni tip .O/. M; T ln lt l giao im ca
AB v CD, AC v BD. V ME; MF l cc tip tuyn ca ng trn .O/ vi E; F l tip
im. Chng minh rng E; F; T thng hng.

Chng minh. Gi H l giao im ca OM v EF . Ta c:


MA:MB D ME 2 D MH :MO
Do t gic ABHO ni tip. Tng t t gic CDOH ni tip. Suy ra:
\
\ C MH
\
[ C ODC
\ ) BH
\
\
BH
C D BHM
C D BAO
C D BT
C
Nn t gic BH T C ni tip. Tng t t gic AH TD ni tip. Ta c:
\
\ D ODH
\ v TH
\
[ D TAD
[ D THD
\
MH
C D ODC
C D TBC
Do :


1 \
\
\ C OHD
\ D 90
\
MH C C TH
C C THD
MH
T D
2

Nn TH ? OM . Do TH; EF trng nhau, tc l ba im E; F; T thng hng.


Ta c iu phi chng minh.
Quay li bi ton,
63

Tp ch Epsilon, S 08, 04/2016

Chng minh. T b trn, ETF ? OQ v theo nh l Brokard th ta cng c P T ? OQ, v


vy ta c ngay c E; F; P thng hng.
Bi ton c gii xong.
Nhn xt. Li gii trn s dng nh l Brokard nh mt chic cu lin kt cc im vi
nhau. Hn th sau khi thng qua li gii, r rng bi ton c vn dng kho lo tnh cht
ca tip tuyn v c trnh by ngn gn thng qua hnh hc phng thun ty.
Ni v t gic th hn vn c quan tm khng km hin nay l t gic ton phn cng vi
im Miquel ca t gic ny, v chng c lin h vi nh l Brokard th no, ta cng xt hai bi
ton sau:

Bi ton 2 (IMO 1985). Cho tam gic ABC . Mt ng trn tm O i qua A; C v ct cc


on AB; BC theo th t ti hai im K; N phn bit. Gi s M l giao im th hai ca
\ D 90 .
.ABC / v .KBN /. Chng minh OMB
\ D 90
Ta thy M l im Miquel ca t gic ton phn ACPNFB. Chng minh OMB
tc l chng minh M l chn ng cao k t O xung BP . Mt khc theo nh l Brokard th
OG l ng cao ca tam gic GBP vi G l giao im ca AN v KC . Vy ta ch cn chng
minh O; G; M thng hng l xong.
Chng minh. Gi M 0 l chn ng cao k t O xung BP .
Gi D; E ln lt l giao im ca .dC I dK /; .dA I dN /. Theo b ca nh l Brokard th
E; B; M 0 ; D; P thng hng.
T ta d dng suy ra cc ng gic AEM 0 NO v KM 0 DCO ni tip. Nn:
0 C D AM
0 O C OM
0 C D ANO
\
\
\
\ C OKC
\ D 180
AM

\
KAC

[ D ABC
\
NCA

Do t gic ABM 0 C ni tip. Tng t KBM 0 N ni tip. Nn M 0 l giao im th hai ca


.ABC / v .KBN /. Suy ra M 0 trng M .
Vy ta c iu phi chng minh.
64

Tp ch Epsilon, S 08, 04/2016

Nhn xt. Mt bi ton khng qu kh nhng vn tip tc p dng b quen thuc ny. Song
ta tip tc pht hin mt tnh cht kh hay y, .AKNC /; .AONM /; .KOCM / u nhn
G l giao im ca hai ng cho. iu ny d dng thu c thng qua khi nim tm ng
phng ca ba ng trn. Vy ta c th chng minh nh l Brokard bng kt qu ny hay
khng? Ta cng quay li vi nh l th v ny:

Chng minh. Gi N; P ln lt l chn ng cao k t O; E xung EF; FO.


Khi cc ng gic OBNKD v AOCNH ni tip. Mt khc ta cng c t gic ABCD ni
tip. p dng kt qu ta c AC; BD; ON ng quy ti G hay O; G; N thng hng.
Tng t vy ta cng cn chng minh b ba t gic ABCD; AP CE; BPDE ni tip. Tht vy,
65

Tp ch Epsilon, S 08, 04/2016


ng gic AMDPO ni tip. Do :
[ D ACB
[ C BCE
[ D ACB
[ C BAD
[ D 90 C ADO
\ D 90 C APO
[ D APE
[
ACE
Suy ra t gic AP CE ni tip. Theo ta cng c EBPD ni tip. p dng kt qu ta c
AC; BD; EP ng quy ti G hay E; G; P thng hng.
Vy ta c iu phi chng minh.
Nhn xt. Li thm mt pht kin th v v nh l ny thng qua tm ng phng G ca su
ng trn. Thm ch, qua Bi ton 2, ta cn thy im Miquel ca t gic ton phn l chn
ng cao k t tm O xung cnh EF . Khng nhng vy, ta c th thu c tnh cht vi nhiu
ng dng nh sau:
Tnh cht 1. Chn ba ng cao ca tam gic EF G chnh l ba im Miquel ca t gic
ton phn ABECDF .
Tnh cht 2. Tm O ca ng trn ngoi tip t gic ABCD nm trn ng trn Miquel ca
t gic ton phn ABECDF .
Ta li ngh n tnh cht ng thng Steiner ca t gic ny i qua cc trc tm, vy im G c
quan h g vi ng ny khng? Ta c bi ton sau:

Bi ton 3. Cho tam gic nhn ABC . ng trn tm O i qua B v C ct AB; AC ln lt


ti M; N . Gi P l giao im ca BN v CM . Gi H; K ln lt l trc tm ca tam gic
ABC v tam gic AMN . Chng minh rng P; K; H thng hng.
Ta nhn thy K; H cng nm trn ng thng Steiner nn P cng phi nm trn ng ny.
Nhn vo gi thit bi th ta kh nh hng c li i, nhng ta n tnh cht ca ng
thng Steiner ca t gic ton phn thng qua mt b sau:
B 3. Cho t gic ton phn ABF CDE. Chng minh rng ng thng Steiner ca t
gic l trc ng phng chung ca .AC /; .BD/ v .EF /.
Chng minh. Gi H1 ; H2 l trc tm ca CDE; ABE.
Gi H; I ln lt l hnh chiu ca H1 ln ED; CD. Ta thy H 2 .AC /; I 2 .EF /.
Ta c t gic HH1 ID v CHIE ni tip nn:
H1 H :H1 C D ID:IC D HD:HE D H1 I :H1 E
Nh vy H1 nm trn trc ng phng ca .AC / v .EF /. Tng t H2 cng nm trn trc
ng phng ca .AC / v .EF /. Hon ton theo cch y ta cng suy ra H1 H2 l trc ng
phng ca .BD/ v .EF /.
Ta kt lun ng thng Steiner ca t gic ton phn ABF CDE l trc ng phng chung
ca .AC /; .BD/ v .EF /.
Vy ta c iu phi chng minh.
66

Tp ch Epsilon, S 08, 04/2016

Vi b quan trng ny, bi ton s tr nn nh nhng hn, ta cng tr li:


Chng minh. V P l giao ca BN v CM nn:
PM :P C D PN :PB
Suy ra P nm trn trc ng phng ca .BN / v .CM /. p dng b ta thu c P nm
trn ng thng Steiner ca t gic ton phn AMBPNC . Nh vy P; K; H thng hng.
Vy ta c iu phi chng minh.

Nhn xt. Bi ton trn kho lo vn dng mt vin ngc qu trong t gic ton phn, t
a ta n mt tnh cht khc na ca nh l Brokard:
Tnh cht 3. Trc tm G ca tam gic OEF nm trn ng thng Steiner ca t gic ton phn
67

Tp ch Epsilon, S 08, 04/2016


ABECDF .
Do nh l Brokard cho ta mt h thng tam gic trc tm nn thng qua bi ton sau ta
li c thm mt tnh cht p nh sau
Bi ton 4 (Romanian Master of Mathematics 2013). Cho t gic ABCD khng l hnh thang
ni tip ng trn !. Gi P; Q; R ln lt l giao im ca AB v CD, AD v BC , AC v
BD. Gi M l trung im ca PQ. MR ct ! ti K. Chng minh rng ng trn ngoi tip
tam gic KPQ v ! tip xc nhau.

y l mt bi ton hay v kh, ta s s dng hai b , trc ht ng thng vung gc vi


OR ti R ct ! ti P1 ; Q1 nh hnh v
B 4. AP1 ct CQ1 ti mt im thuc PQ.
Chng minh. Gi X; Y ln lt l giao im ca AP1 vi CQ1 , AQ1 vi CP1 .
p dng nh l Pascal cho lc gic suy bin thnh t gic AP1 CQ1 ta d dng suy ra c
ng thng XY cha P v Q nn AP1 v CQ1 ct nhau ti mt im thuc PQ.
Ta c iu phi chng minh.
B 5. PP1 ; QQ1 ct nhau ti K.
Chng minh. PP1 ct ! ti P2 . Ta s chng minh P2 Q1 i qua Q.
p dng nh l Pascal cho lc gic P2 Q1 CDAP1 kt hp vi b 1 ta c giao im ca P2 Q1
68

Tp ch Epsilon, S 08, 04/2016


v AD, Q1 C v AP1 cng vi giao im ca AD v P1 P2 l P thng hng. Mt khc theo b
1 ta c Q1 C ct AP1 ti mt im thuc PQ, do giao im ca P2 Q1 v AD cng nm
trn PQ, tc l im Q. Vy P2 Q1 i qua Q.
Ta c iu phi chng minh.
Quay tr li bi ton,
Chng minh. p dng b 4 v b 5, ta c t gic PQP1 Q1 l hnh thang, M; R ln lt
l trung im ca PQ; P1 Q1 v K l giao im ca PP1 ; QQ1 nn ta d dng suy ra ng
trn ! ngoi tip tam gic KP1 Q1 v KPQ tip xc nhau.
Vy bi ton c gii xong.
Nhn xt. Vic s dng kho lo nh l Pascal gip ta gii quyt gn gng bi ton trn.
Nh ta bit, mt t gic ni tip khng l hnh thang bt k ch to ra mt tam gic trc tm,
ta tm gi theo nh x tam gic trc tm y l nh ca t gic ni tip thng qua mt nh x
Brokard. Nhng iu ngc li th cha ng, tc l mt tam gic trc tm c th c nhiu to
nh. C th, ta xt tam gic nhn OEF
p , trc tm G, ng cao EK. Bng cng c phng tch,
ta dng ng trn tm O bn knh OE 2 GE:KE, y chnh l ng trn ngoi tip cc
to nh m ta cp n, vi mt ct tuyn t P ct .O/ ti hai im s cho ta mt to nh ca
tam gic trc tm OEF .

T nhn xt ny, ta c cch pht biu rng hn cho bi ton 4 nh sau


Bi ton 5. Cho tam gic ABC trc tm H . ng trn ng knh AB; AC ct nhau ti X .
Gi ! l ng trn tm A bn knh AX. Gi M l trung im ca BC . ng thng MH ct
! ti E; F . Chng minh rng ng trn ngoi tip cc tam gic BCE v BCF tip xc !.
Bi ton trn cn c thm mt tnh cht nh sau
Bi ton 6. Cho tam gic ABC trc tm H . ng trn ng knh AB; AC ct nhau ti X .
Gi !A l ng trn tm A bn knh AX. Gi M l trung im ca BC . ng thng MH
ct !A ti A1 sao cho A1 nm gia M v H . nh ngha tng t cho B1 ; C1 . Chng minh rng
AA1 ; BB1 ; C C1 ng quy.
69

Tp ch Epsilon, S 08, 04/2016


Cc bi ton trn, c bit l b c nu ra c th s dng nh l Pascal gii quyt vn
. Nh th, vic pht trin t t gic ni tip ng trn sang t gic c bn im nm trn mt
conic l hon ton c th. Phn tip theo tc gi xin gii thiu vic vn dng nh l Pascal cho
bn im nm trn mt conic, t cho thy bn cht ca nh l Brokard v t gic ni tip
trong mt ng trn. Ta c b sau l pht trin rng hn ca b nu u ti liu
B 6. Cho bn im A; B; C; D nm trn mt conic C to thnh mt t gic li khng l hnh
thang. M; N ln lt l giao im ca AD v BC , AB v CD. P; Q l giao im tip tuyn
ca C ti A v C , B v D. Khi bn im M; N; P; Q thng hng.

Chng minh. p dng nh l Pascal cho lc gic suy bin thnh t gic AABC CD ta c
M; N; P thng hng. p dng nh l Pascal cho lc gic suy bin thnh t gic ABBCDD ta
c M; N; Q thng hng. Tm li ta c M; N; P; Q thng hng.
Ta c iu phi chng minh.
T b trn ta c tip h qu nh sau
H qu. Cho conic C tip xc cc cnh AB; BC; CD; DA ca t gic ABCD theo th t ti
M; N; P; Q. Khi AC; BD; MP; NQ ng quy.
R rng b trn l mt m rng p ca b nu u bi i vi ellipse, parabol hay
hyperbol. Trong trng hp l ellipse, ch cn thc hin mt php co thnh hnh trn th ta li c
ngay nhng ng dng mi m na. B tip theo chnh l m rng ca B 2:
Gi A1 A2 A3 A4 l t gic khng l hnh thang ni tip trong mt conic C v M1 ; M2 ; M3 ln lt
l giao im ca A1 A2 v A3 A4 , A2 A3 v A4 A1 , A3 A1 v A2 A4 . Gi N1 ; N2 ; N3 ; P1 ; P2 ; P3
l giao im ca tip tuyn ca conic ti A1 v A2 , A1 v A4 , A1 v A3 , A3 v A4 , A2 v
A3 , A2 v A4 . Gi U1 ; U2 l tip im ca hai tip tuyn ca conic i qua M1 . nh ngha
tng t V1 ; V2 i vi M2 v W1 ; W2 i vi M3 . B 1 cho ta ba b im thng hng
l .M2 I M3 I N1 I P1 /; .M3 I M1 I N2 I P2 /, .M1 I M2 I N3 I P3 /. Ln lt gi ba ng thng y l
m1 ; m2 ; m3 .
70

Tp ch Epsilon, S 08, 04/2016

B 7. U1 ; U2 nm trn m1 , V1 ; V2 nm trn m2 v W1 ; W2 nm trn m3 .

Chng minh. Trong mt phng x nh tn ti php chiu ' bin conic C thnh ng trn C 0 .
Nh ta bit, php chiu ' bo ton s thng hng ca cc b im nn thng qua php bin
hnh ny ta c ngay B 2 v theo cc b im im .M2 I M3 I U1 I U2 /; .M1 I M3 I V1 I V2 /;
.M1 I M2 I W1 I W2 / thng hng.
Ta c iu phi chng minh.
Mt kt qu p li c chng minh. Gi O l tm ca conic C, t kt qu ny ta xt tam gic
OM1 M2 sau:

71

Tp ch Epsilon, S 08, 04/2016


Khi conic C l ng trn th ln lt theo tnh cht ca tip tuyn ta c m1 ? OM1 v
m2 ? OM2 v nh th ta hiu c ti sao nh l Brokard li c pht biu nh vy. Cui
cng, tc gi xin xut mt s bi tp bn c c th rn luyn thm nhng t duy khc v
nh l Brokard ny.

3. ng dng
Bi ton 7 (Trng h Bc Trung B 2015 - Ngy 2). Cho tam gic nhn ABC c ng
cao AH , trc tm K. ng thng BK ct ng trn ng knh AC ti D; E (BD < BE).
ng thng CK ct ng trn ng knh AB ti F; G (CF < C G). ng trn ngoi tip
tam gic DHF ct BC ti im th hai l P .
a) Chng minh rng cc im G; H; P; E cng thuc mt ng trn.
b) Chng minh rng cc ng thng BF; CD; PK ng quy.
Bi ton 8. Cho t gic li ABCD khng l hnh thang ni tip ng trn tm O. Gi E; F; G
ln lt l giao im ca AB v CD, AD v BC , AC v BD. Gi X; Y theo th t l trung
im ca AC; BD. Ko di XY ct EF ti Z. Ko di GZ ct ng trn ngoi tip tam gic
OFE ti Q. ng trn ng knh GQ ct ng trn ngoi tip tam gic OFE ti K khc
Q. Gi H l im Miquel ca t gic ton phn ABCDEF . Chng minh rng ng trn
ngoi tip cc tam gic KQG v KZH tip xc nhau.
Bi ton 9 (L B Khnh Trnh). Cho tam gic ABC ni tip .O/ c B; C c nh, A chy
trn cung nh BC . M; N l trung im AB; AC . Ly P trn ng thng MN tha mn
\
\ C ACB
[ D BAC
[ . ng trn .ABP / ct AC ti E, .ACP / ct AB ti F .
BP
C C ABC
Chng minh rng .AFE/ i qua mt im c nh.
Bi ton 10 (L B Khnh Trnh). Cho tam gic ABC ni tip .O/, trung tuyn AI ct .O/
ti D. AB ct CD ti E. AC ct BD ti F . .ABF / ct .ACE/ ti K. .O1 /; .O2 / l cc ng
trn i qua B tip xc vi AC ti A v i qua C tip xuc vi AB ti A. Chng minh ba ng
trn .O1 /; .O2 / v .OK/ c mt im chung.
Bi ton 11. Cho tam gic ABC ni tip .O/, im P nm trong tam gic sao cho AP vung
gc BC . ng trn ng knh AP ct ng thng AC; AB ln lt ti E; F v ct .O/ ti
im G khc A. Chng minh rng BE; CF; GP ng quy.
Bi ton 11 c th xem [6].
Bi ton 12 (USA TST 2012). Cho tam gic ABC nhn, ng cao AA1 ; BB1 , C C1 . Gi A2
l giao im ca BC v B1 C1 . nh ngha B2 ; C2 tng t. Gi D; E; F theo th t l trung
im ca BC; CA; AB. Chng minh rng ng thng vung gc k t D ti AA2 , E ti BB2
v F ti C C2 ng quy.
Bi ton 13. Cho t gic ABCD ni tip .O/. Gi E; F ln lt l giao im ca AB v CD,
AD v BC . Gi M; N theo th t l trung im ca AC; BD. Gi H; K ln lt l trc tm
ca tam gic MEF v tam gic NEF . Chng minh rng HNKM l hnh bnh hnh.
72

Tp ch Epsilon, S 08, 04/2016


Bi ton 14 (Trn Quang Hng). Cho tam gic ABC . ng trn tm K i qua B; C ct
CA; AB ln lt ti E; F . BE; CF ct nhau ti H . D l hnh chiu ca K xung AH . M; N
theo th t di chuyn trn DE; DF sao cho BM ? BE; CN ? CF . Chng minh rng ng
i trung xut pht t nh A ca tam gic ABC chia i cnh MN .
Bi ton trn c tham kho trong [7]. Hn na ta cn c thm hai m rng sau
Bi ton 15 (Trn Quang Hng). Cho tam gic ABC v im P . AP; BP; CP ln lt ct
BC; CA; AB ti D; E; F . Trn DE; DF ln lt ly M; N sao cho BM k AC; CN k AB. Gi
S; T theo th t l trung im ca EF; MN . Chng minh rng A; S; T thng hng.
Bi ton 16 (Trn Quang Hng). Cho tam gic ABC v im D nm trn cnh BC . .DAB/;
.DAC / ct CA; AB ln lt ti E; F . M; N nm trn DE; DF sao cho BM k AC; CN k AB.
Gi S; T ln lt l trung im ca EF; MN . Chng minh rng ST song song vi ng i
trung xut pht t nh A ca tam gic ABC .
Bi ton 17 (Chn i tuyn quc gia Khoa hc t nhin 2014). Cho t gic ABCD ni tip.
PD
BD 2
M; N ln lt l trung im ca CD; AB. P nm trn CD sao cho
D
. AC ct BD
PC
AC 2
ti E. Gi H l hnh chiu ca E ln PN . Chng minh rng ng trn ngoi tip cc tam gic
HMP v EDC tip xc nhau.
Trn y l tng hp ca tc gi v nh l Brokard, xin cm n anh Ng Quang Dng - lp
12A2 trng THPT chuyn KHTN c nhng trao i qu bu gip bi vit c hon thin
hn.

Ti liu tham kho


[1] nh l Brokard.
http://www.imomath.com/index.php?options=334&lmm=0.
[2] Thanh Sn, Mt s chuyn Hnh hc phng bi dng hc sinh gii Trung hc
ph thng, NXB Gio dc, 2013.
[3] Nguyn Vn Nho, Nhng nh l chn lc trong hnh hc phng qua cc k thi Olympic,
NXB Gio dc, 2007.
[4] Romanian Master of Mathematics 2013 - Problem 3.
http://www.artofproblemsolving.com/community/c6h523072p3558896
[5] IMO 2015 - Problem 3.
http://www.artofproblemsolving.com/community/c6h1112748p5079655
[6] Concurrent problem.
http://www.artofproblemsolving.com/community/c6h1161051p5527875
[7] Symmedian bisects segment.
http://www.artofproblemsolving.com/community/c6h1192618p5839116

73

Tp ch Epsilon, S 08, 04/2016

74

T GIC NGOI TIP NG TRN


Xun Anh
(Trng THPT Chuyn KHTN, H Ni)
T gic ngoi tip l mt ch khng qu mi i vi bt k ai am m vi
mn ton v c bit l mn hnh hc nhng c khng nhiu nhng ti liu vit v
ch ny. Vy nn trong bi vit ny ti xin cp n vn ny vi kin thc v
nhng ng dng c bn nht ca t gic ngoi tip.

1. Mt s tnh cht c bn ca t gic ngoi tip ng


trn
Khi nhc ti t gic ngoi tip ng trn, chng ta nn n nhng tnh cht hay s dng
nh sau:
Cho t gic ABCD ngoi tip ng trn .I /. .I / tip xc AB, BC , CD, DA ln lt ti M ,
N , P , Q.
t AM D AQ D a, BM D BN D b, CN D CP D c, DQ D DP D d .
nh l 1.(nh l Pithot) AB C CD D BC C DA.
nh l 2.
1. (nh l Newton) AC , BD, MP , NQ ng quy ti T
2.

AT
CT

a BT
b
,
D
:
c DT
d
B
M
A

T
Q
I

Chng minh. Gi T1 l giao im ca AC vi MP v T2 l giao im ca AC vi NQ.


75

Tp ch Epsilon, S 08, 04/2016


Ta s chng minh T1  T2  T . Tht vy, p dng nh l sin, ta c:
T1 A
AM
sin C T1 M sin AMP
D


T1 C
sin AT1 M
CP
sin CPM
AM
D
CP
a
T2 A
D nn T1 , T2 , T trng nhau. Tnh cht c chng minh.
Tng t,
T2 C
c
nh l 3. AC , MN , PQ ng quy ti V v
1.

AV
CV

a BV
b
,
D . T suy ra .AC; T V / D
c DV
d

B
M
A

T
Q
I

a
. p dng nh l Menelaus cho 4ABC ta c
c
CV
M , N , V thng hng. Tng t suy ra P , Q, V thng hng. Vy tnh cht trn cng c chng
minh.
Chng minh. Ly V trn AC sao cho

AV

nh l 4. ng thng qua A vung gc vi AB ct BI ti X, ng thng qua A vung gc


vi AD ct DI ti Y th XY vung gc vi AC .
B
A
Y

X
E

76

Tp ch Epsilon, S 08, 04/2016


Chng minh. Gi F l hnh chiu ca X ln BC ; E l hnh chiu ca Y ln CD.
Ta c AX 2 XC 2 D AX 2 XF 2 F C 2 D F C 2 .
AY 2
M F C D BC

Y C 2 D AY 2

EC 2 D

AD D EC nn AX 2

AB D DC

XA2 C C Y 2

YE 2

AY 2

EC 2 :

XC 2 D AY 2

Y C 2.

CX 2 D XA2 C AC 2 CX 2 .AY 2 C AC 2
! !
! !
D 2AX  AC 2AY  AC
! !
D 2AC  YX

C Y 2/

Do AC ? XY . Vy ta chng minh xong nh l 4.


Ngoi ra, chng ta cng nn bit mt s cng thc tnh cc yu t trong t gic ngoi tip ng
trn. Vic chng minh xin dnh cho cc bn c.
Trong cc tnh cht di y, ta t r l bn knh ng trn ni tip t gic ABCD v
DAB D ; ABC D ; BCD D ; CDA D :

nh l 5. AB D

NQ IA:IB
: r2 :
2
B
M
A

B'
A'
N

Q
I

Chng minh. Gi IA, IB ln lt ct QM , MN ti A0 , B 0 Ta c IA:IA0 D IB:IB 0 D r 2 nn


A0 B 0
IA0
IB:A0 B 0
IB:IA NQ
0 0
4IA B  4IBA suy ra
D
. T , AB D
D
:
. Vy nh l 5
0
AB
IB
IA
r2
2
c chng minh.

nh l 6.

IB 2
ID 2
D
.
AB:BC
CD:DA
77

Tp ch Epsilon, S 08, 04/2016


B
M
A

N
Q
I

IC:ID NQ
AB
IA:IB
:
suy
ra
. Tng t
D
r2
2
CD
IC:ID
2
2
2
BC
IB:IC
AB BC
IB
IB
ID
D
suy ra

D
hay
D
.
AD
IA:ID
CD AD
ID 2
AB:BC
CD:DA

Chng minh. T nh l 5, ta c CD D

nh l 7. IA:IC C IB:ID D

AB  BC  CD  DA
B
M

P'

N
Q
I

M'

Chng minh. Gi M 0 , P 0 ln lt i xng M , P qua I . Ta c 4MIB  4NM 0 M suy


M 0N
MM 0
MM 0 :MI
2r 2
2r 2
2r 2
ra
D
hay M 0 N D
D
. Tng t M 0 Q D
, P 0Q D
,
MI
IB
IB
IB
IA
ID
2r 2
P 0N D
. p dng nh l Ptoleme cho t gic P 0 NM 0 Q v nh l 5, suy ra IA:IC C
IC
p
IB:ID D AB  BC  CD  DA. Vy nh l c chng minh.

nh l 8. IA:IC D

.a C c/:r
sin C
2

:
78

Tp ch Epsilon, S 08, 04/2016


U
B
M
A

N
Q
I

Chng minh. Ly U trn tia MB sao cho M U D NC suy ra I U D IC nn


1
1
1
C
SAI U D .a C c/:r D IA:I U: sin AI U D IA:IC: sin
2
2
2
2
.a C c/  r
hay IA:IC D
. Vy nh l c chng minh.
sin C
2
nh l 9.SABCD D

AB  BC  CD  DA: sin

C
:
2
B

M
A

N
Q
I

Chng minh. Theo nh l 7 v 8, ta c IA:IC CIB:ID D


suy ra SABCD
chng minh.

.a C b C c C d /  r

AB:BC:CD:DA
sin C
2
p
C
D .a C b C c C d /  r D AB  BC  CD  DA: sin
. Ta c iu phi
2

2. Mt s bi tp p dng
Bi ton 1. 4ABC cn ngoi tip ng trn .I /. Ly E thuc AB, F thuc AC sao cho EF
l tip tuyn ca ng trn .I / v EF k BC . ED vung gc vi BC ti D. Gi H l giao
im ca BI vi ng thng qua F vung gc vi BC . Chng minh rng DH vung gc vi
EC .
79

Tp ch Epsilon, S 08, 04/2016


A

H
I

Chng minh. Gi FH ct BC ti G. D thy E; I; G thng hng v F; I; D thng hng.Ta


c BEG D BEI D IEF D GEF D EGB nn tam gic BEG cn ti B, suy ra

BE D BG. Do BEH D BGH (c.g.c) v vy BEH D BGH D 90 hay EH vung


gc EB ti E. p dng nh l 4, suy ra DH vung gc vi EC . Vy bi ton c chng
minh.
Nhn xt. Chng ta c th s dng tnh cht 4 chng minh mt bi ton trong Mathley [5]
bi thy Trn Quang Hng nh sau
Bi ton. 4ABC , ng trn ni tip .I / tip xc CA, AB ti E, F . Ly P di chuyn trn
EF . BP ct CA ti M , MI ct ng thng qua C vung gc AC ti N . Chng minh rng
ng thng qua N vung gc vi P C lun i qua im c nh khiP di chuyn.
Bi ton 2. Cho t gic ABCD ngoi tip ng trn .I /. AB giao CD ti E, AD giao BC
ti F . Chng minh rng ng trn ni tip 4AEF , 4CEF tip xc ti mt im trn EF .
F

B
X=Y
P
Q

A
S
I

80

Tp ch Epsilon, S 08, 04/2016


Chng minh. .I / ln lt tip xc AB, BC , CD, DA ti P , Q, R, S ; ng trn .I1 / v .I2 /
ln lt l ng trn ni tip 4AEF v 4CEF , hai ng trn ln lt tip xc EF ti X , Y .
Ta c

FC

CE D FQ C QC

CR

RE D F S

EP D FA C AS

AP

EA D FA

EA

.
8
1

< FX D .FE C FA
2
Mt khc
1

: F Y D .FE C F C
2
chng minh.

EA/
nn, FX D F Y suy ra X  Y . Vy bi ton c
CE/

Nhn xt. Ngoi ra, nu k ta thy trn hnh v cng c hai tam gic c ng trn ni tip
tip xc nhau l 4ABC , 4ADC (Tng t l cp tam gic 4ABD, 4CBD).
Trong bi ton, ta rng F C CE D FA EA chnh l nh l Pithot m rng cho t gic
lm ngoi tip ng trn. p dng nh l trn ta c th chng minh c bi ton sau.

Bi ton. T gic ABCD ngoi tip ng trn .I /. AB, AD ln lt ct CD, BC ti E, F .


Gi EP , FM ct BC , CD ti Q, R; EP ct FM ti N . Chng minh rng nu t gic AMNP
ngoi tip th QNRC ngoi tip.

Bi ton tip theo c ngh bi A.Golovanov trn din n AoPS [1].

Bi ton 3. Cho t gic ABCD va ngoi tip va ni tip. ng trn ni tip tip xc vi
AB, CD ti X , Y . ng thng vung gc vi AB, CD ti A, D ct nhau ti U ; ti X , Y ct
nhau ti V ; ti B, C ct nhau ti W . Chng minh rng U , V , W thng hng.
81

Tp ch Epsilon, S 08, 04/2016


P

E
U

B
F
A

Chng minh. AD ct BC ti P , ng trn .E/ l ng trn ni tip 4PAB, tip xc vi


AB ti F . U V ct ng thng qua C vung gc CD ti W 0 . Ta c 4PAB  4P CD (g.g)
DY
BF
AX
UV
v X , F i xng vi nhau qua trung im ca AB. Nn 0 D
D
D
, suy ra
W
V
Y
C
FA
XB
0
AU k X V k BW . T W 0  W . Vy bi ton c chng minh.

Tip theo l mt kt qu ht sc ni ting ca t gic ngoi tip.

Bi ton 4 (ng thng Newton ca t gic ngoi tip). Cho t gic ABCD ngoi tip
ng trn .I / th trung im hai ng cho AC , BD v I thng hng.
82

Tp ch Epsilon, S 08, 04/2016


F

B
M
I

Chng minh. Gi M , N ln lt l trung im ca AC , BD. Ta c AB C CD D AD C BC


1
nn SIAB C SICD D SIAD C SIBC D SABCD . Ta xt 2 trng hp:
2
Trng hp 1. AB k CD th d thy trung im AC , BD v I cch u AB, CD nn chng
thng hng.
Trng hp 2. Gi s AB ct CD ti E. Ly G trn tia EA, ly H trn tia ED sao cho EG D
1
1
AB, EH D CD. Ta c SIE G C SIEH D SIAB C SICD D SABCD nn SIHG D SABCD
2
8 2
1

< SNAB C SNCD D SABCD


2
SEHG . D thy cc im M , N c tnh cht tng t nn
. Suy
1

: SMAB C SM CD D SABCD
2
8
1

< SNHG D SABCD SEHG


2
ra
Do SIHG D SMHG D SNHG nn I , M , N thng hng.
1

: SMHG D SABCD SEHG


2
Vy bi ton c chng minh.
Ti xut thm bi ton sau bn c ng dng nh l trn.
Bi ton. T gic ABCD ngoi tip ng trn .I /. ng trn .I / tip xc AB, BC , CD,
DA ti E, F , G, H . Chng minh rng nu I thuc FH th I thuc EG.
Bi ton sau tham kho [2]
Bi ton 5. Cho t gic ABCD ngoi tip ng trn .I /. .I / tip xc AD, BC ti P , Q.
Gi AB ct CD ti S (A 2 SB). Gi .I1 / l ng trn ni tip 4SAD v tip xc vi AD ti
K. Gi .I2 / l ng trn bng tip ng vi nh S ca 4SBC v tip xc BC ti L. Ly M ,
N ln lt l trung im ca AD, BC . Bit rng S 2 LK. Chng minh rng I thuc MN v
IA:IC D IB:ID.
83

Tp ch Epsilon, S 08, 04/2016

I2

P
N

I1
K
S

Li gii theo Luis Gonzlez. Ta s s dng 2 b .

B 1. 4ABC ngoi tip ng trn .I /. ng trn bng tip .Ia / ng vi gc A ca


4ABC tip xc BC ti D. Gi M l trung im ca BC . Chng minh AD k IM . (IM c
gi l ng thng Nagel).

B 2. 4ABC ngoi tip ng trn .I /. ng trn tip xc BC ti D. Gi Ia l tm


ng trn bng tip gc A ca 4ABC . Gi M l trung im ca BC . Chng minh rng
AD k Ia M . (Ia M c gi l ng thng Gergonne).

p dng 2 b vo bi ton, ta c I N k SL k IM k SK. Li c S , K, L thng hng do


I , M , N thng hng. Ta thy P , K i xng vi nhau qua M (M l trung im ca AD)
v Q, L i xng vi nhau qua N (N l trung im ca BC ) suy ra PQ k MN k KL.
Gi AD ct BC ti T . Do 4TMN 4TPQ ( g.g ). Mt khc 4TMN cn ti T nn
ATB

T NI D 90
D AIB ( do I l tm ng trn bng tip gc T ca 4TAB ). Suy
2
IB
NB
NC
ra 4IAB4NIB ( g.g ) do
D
D
. Tng t, ta c 4I NC 4DIC ( g.g )
IA
NI
NI
IC
NC
IC
IB
suy ra
D
. T , ta c
D
nn IA:IC D IB:ID. Vy bi ton c chng
ID
NI
ID
IA
minh.

Bi ton 6. T gic ABCD ngoi tip ng trn .I /. ng trn .I / tip xc vi AB, BC ,


CD, DA ln lt ti M , N , P , Q. BQ, BP ln lt ct ng trn .I / ti E, F . Chng minh
rng ME, NF , BD ng quy.
84

Tp ch Epsilon, S 08, 04/2016


B
M
A

K
F
N

Chng minh. Gi ME giao NF ti K. Ta c AC , BD, MP , NQ ng quy ti X. p dng


nh l Pascal cho lc gic MEQNFP suy ra K, B, X thng hng do ME, NF , BD ng
quy. Vy bi ton c chng minh.
Bi ton 7 (Romania TST 2012). Cho t gic ABCD ngoi tip ng trn sao cho ABC C

ADC < 180 v ABD C ACB D ACD C ADB. Chng minh rng mt trong hai
ng cho ca t gic ABCD i qua trung im ng cho cn li.
B

C
D

Chng minh. ng trn ngoi tip 4ABC ct BD ti E. D dng nhn thy DAE D
DA
DE
DE
DC
DCE. p dng nh l sin, ta c
D
D
D
.
sin AED
sin DAE
sin DCE
sin DEC
DA
sin AED
sin ACB
AB
Suy ra
D
D
D
. Li c t gic ABCD ngoi tip ng trn
DC
sin DEC
sin BAC
BC
DA
BC C DA CD
do AB D BC C DA CD. Nn
D
.
DC
BC
Hay .BC CD/.DA CD/ D 0 suy ra BC D CD hoc CD D DA.
Khi BC D CD th AB D DA. Do vy AC l trung trc ca BD hay AC i qua trung im
ca BD. Chng minh tng t vi trng hp CD D DA. Vy bi ton c chng minh.
c bit, trong IMO Shortlist cng tng xut hin nhiu bi ton lin quan n t gic ngoi tip
v mt s bi ton sau l in hnh.
85

Tp ch Epsilon, S 08, 04/2016


Bi ton 8 (IMO Shortlist 2009). Cho t gic ABCD ngoi tip ng trn. Mt ng thng
.d / bt k qua A ct BC ti M , ct CD ti N . Gi .I1 /; .I2 /; .I3 / l ng trn ni tip
4ABM , 4MNC , 4NDA. Chng minh rng trc tm 4I1 I2 I3 nm trn .d /.

A
I1

H
E

I3

M
F
I2

Chng minh. Gi X l giao im ca .d / vi tip tuyn k t C vi ng trn .I1 / nn


ABCX l t gic ngoi tip hay CX D BC C AX AB. Li c ABCD ngoi tip nn
AD D AB C CD BC . Suy ra CX C AD D AX C CD hay t gic AXCD ngoi tip ng
trn.I3 /. Gi E; F ln lt i xng vi C qua I2 I3 , I2 I1 ; H l trc tm 4I1 I2 I3 . D thy
E, F thuc .d /. Li c F I2 HI1 , I3 HEI2 ni tip, do FHI2 D F I1 I2 D CI1 I2 D
CI3 I2 D EI3 I2 D EHI2 hay H thuc .d /. Ta c iu phi chng minh.

Nhn xt. Trong qu trnh tm hiu, ti nhn thy bi ton s 5 trong thi VN TST 2015 c
coi l m rng t bi ton trn.
Bi ton. 4ABC nhn v c im P nm trong tam gic sao cho APB D AP C D v

> 180
BAC . ng trn ngoi tip 4APB ct AC E, ng trn ngoi tip 4AP C
ct AB F . Q l im nm trong 4AEF sao cho AQE D AQF . Gi D l im i xng
vi Q qua EF , phn gic EDF ct AP ti T .
a) Chng minh rng DET D ABC ; DF T D ACB.
b) ng thng PA ct cc ng thng DE, DF ln lt ti M , N . Gi I , J ln lt l tm
ng trn ni tip cc 4PEM , 4PF N v K l tm ng trn ngoi tip 4DIJ . ng
thng DT ct .K/ ti H . Chng minh rng HK i qua tm ng trn ni tip ca 4DMN .
Bi ton 9 (IMO Shortlist 2007). Cho im P nm trn cnh AB ca t gic ABCD. Gi .I /,
.I1 /, .I2 / ln lt l ng trn ni tip 4CPD, 4APD, 4CPB. Bit rng ng trn .I /
tip xc vi ng trn .I1 /, .I2 / ti K, L. Gi E, F ln lt l giao im ca AC , BD v
AK, BL. Chng minh rng E, I , F thng hng.

86

Tp ch Epsilon, S 08, 04/2016


D

I1
I2

B
N

Chng minh. Gi M , N l giao im ca hai tip tuyn chung ngoi ca .I /, .I1 / v .I /, .I2 /.
AD C DP AP
DP C DC P C
CP C BC BP
DC C CP
D
v
D
2
2
2
2
AD C CP D AP C CD
nn
hay t gic AP CD, BPDC ngoi tip ng trn.
BP C CD D DP C BC
Ta c

DP

p dng nh l Monge & dAlembert cho b ba ng trn .I /, .I1 /, .AP CD/ v .I /, .I2 /,
.BPDC / th M; A; C v N; B; D. Tng t, p dng nh l Monge & dAlembert cho b ba
ng trn .I /, .I1 /, .I2 / suy ra KL, AB, MN ng quy. Ta tip tc p dng nh l Desargues
cho 4KAM v 4LBN ; suy ra E, I , F thng hng. Vy ta c iu phi chng minh.

Bi ton 10 (Sharygin Geometry Olympiad 2014). Cho t gic ABCD ngoi tip ng trn
.I /. ng trn .I / tip xc BC; DA ti E; F sao cho AB; EF; CD ng quy. ng trn
ngoi tip tam gic AED v BF C ln lt ct .I / ti im th 2 ln lt l E1 ,F1 . Chng
minh rng EF k E1 F1 .
87

Tp ch Epsilon, S 08, 04/2016


P

E
C

E1
G
M

F1
H

I
A

Chng minh. ng trn .I / tip xc vi AB v CD ln lt ti G v H . Gi P l im


ng (
quy ca AB, EF , CD suy ra BC , AD, GH ng quy ti R. Ly EE1 giao AD ti M .
2

MF D ME:ME1
(do M l tm ng phng ca .I /; .AED/; .AID/ ). Suy
ME:ME1 D MA:MD

Ta c
2

raMF D MA:MD. p dng nh l Ceva cho 4PAD vi AC , BD, PF ng quy, ta c


FA CD BP


D 1. Ta li p dng nh l Menelaus cho 4PAD vi R, B, C thng hng, ta
FD CP BA
RD BA CP
RD
FD
c


D 1. T suy ra
D
do vy .AD; RF / D 1.
RA BP CD
RA
FA
T v kt hp h thc Newton, suy ra M l trung im ca RF . Tng t, FF1 giao BC ti
2
2
N , suy ra N l trung im ca RE. M RE D RF D RG:RH , nn 4REF cn ti R. Suy
ra NFE D NEF . Li c t gic EFE1 F1 ni tip ng trn .I / do t gic EFE1 F1 l
hnh thang cn. Suy ra EF k E1 F1 . Vy bi ton c chng minh.

Bi ton 11. T gic ABCD ngoi tip ng trn .I /. Ly M i xng A qua BD. Gi DM
ct BC ti E, BM ct DC ti F . Chng minh rng t gic CEMF ngoi tip ng trn.
88

Tp ch Epsilon, S 08, 04/2016


B

K
E
I

DK
BK
Chng minh. Gi CI ct BD ti K nn CK l phn gic BCD suy ra
D
. Gi
BC
DC
1
phn gic gc MBC giao CI ti J1 suy ra IBJ1 D ABM D ABD D MBD do
2
IK J1 K
BK 2
IBK D CBJ1 D MBJ1 . T ta c

D.
/ . Gi phn gic CDM
IC J1 C
BC
IK J2 K
J1 K
DK 2
J2 K
ct CI ti J2 .Tng t, ta c

D.
/ . T nhng iu trn, suy ra
D
IC J2 C
DC
J1 C
J2 C
hay J1  J2  J . Nn CJ , FJ , EJ MJ ln lt l phn gic ca F CE, CFM , CEM ,
FME do vy J tm ng trn ni tip t gic CEMF . Vy bi ton c chng minh.
Bi ton sau y tham kho [4] l mt ng dng ca cc nh l t 6 n 9 v cc cng thc lin
quan n cc yu t trong mt t gic ngoi tip.
Bi ton 12. Cho t gic ABCD ngoi tip ng trn .I / khng c 2 cp cnh no song song.
Chng minh rng I l trng tm ca ABCD khi v ch khi IA:IC D IB:ID.
B
M E
A

N
Q
H

F
I

P G

Chng minh. Trc tin, ta chng minh chiu thun ca bi ton ngha l nu I l trng tm
t gic ABCD th IA:IC D IB:ID t AM D AQ D a, BM D BN D b, CN D CP D c,
ja d j
jb cj
DP D DQ D d suy ra QH D
; NF D
.
2
2
Gi E, F , G, H ln lt l trung im ca AB, BC , CD, DA. Ta c I l trng tm t gic
ABCD nn I l trung im HF suy ra HQ D NF hay ja d j D jb cj.Trng hp 1:
89

Tp ch Epsilon, S 08, 04/2016


a d D b c nn a C c D b C d . Theo nh l 8, ta c IA:IC D IB:ID. Trng hp 2:
a d D c b nn a C b D c C d . Tng t a b D c d suy ra a D c, b D d . Do t
gic ABCD l hnh bnh hnh (do t gic ABCD khng c 2 cp cnh no song song). Ta
chng minh xong phn thun ca bi ton.
Tip theo, ta chng minh phn o, tc l IA:IC D IB:ID th I l trng tm t gic ABCD.
Ta c IA:IC D IB:ID suy ra HQ D NF , IF D IH hay I thuc ng trung trc ca FH .
Tng t I thuc ng trung trc ca EG. M EG v FH khng song song suy ra I l trng
tm ca ABCD. Vy bi ton c chng minh xong.

3. Bi tp t luyn
Bi ton 13. (Sharygin Geometry Olympiad 2014). Cho I l tm ng trn ni tip t gic
ABCD. Tip tuyn ti A v C ca ng trn ngoi tip 4AIC ct nhau ti X . Hai tip tuyn
ti B v D ca ng trn ngoi tip 4BID ct nhau ti Y . Chng minh rng X , Y , I thng
hng.
Bi ton 14. Cho t gic ABCD ngoi tip ng trn .I /. Gi H1 , H2 , H3 , H4 ln lt l
trc tm 4IAB, 4IBC , 4ICD, 4IDA. Chng minh rng H1 , H2 , H3 , H4 thng hng.
Bi ton 15. Cho t gic ABCD ni tip. Mt ng trn bt k i qua C , D giao AC , AD,
BC , BD ti A1 , A2 , B1 , B2 . Mt ng trn khc i qua A, B giao CA, CB, DA, DB ti C1 ,
C2 , D1 , D2 . Chng minh rng tn ti mt ng trn tip xc vi A1 A2 , B1 B2 , C1 C2 , D1 D2 .
Bi ton 16. Cho hai ng trn .O1 / v .O2 / tip xc trong vi .O/ ti A, B. T A k tip
tuyn At1 , At2 ti .O2 /, t B k tip tuyn Bz1 , Bz2 ti .O1 /. Gi At1 ct Bz1 ti X , At2 ct
Bz2 ti Y . Chng minh rng AXBY ngoi tip ng trn.
Bi ton 17. Cho t gic ABCD ngoi tip ng trn. ng trung trc ca DA, AB, BC ,
CD ln lt ct trung trc ca AB, BC , CD, DA ti X , Y , Z, T . Chng minh rng t gic
X Y ZT ngoi tip ng trn.
Bi ton 18. Cho 4ABC , ly D, E thuc BC . Gi .I1 /, .I2 /, .I3 /, .I4 / ln lt l ng trn
ni tip 4ABD, 4ACE, 4ABE, 4ADC . Chng minh rng tip tuyn chung ngoi khc BC
ca .I1 /, .I2 /, .I3 /, .I4 / ct nhau trn BC .

4. Li kt
Ti xin gi li cm n chn thnh nht ti thy Trn Quang Hng gio vin trng THPT
chuyn KHTN, H Ni, v anh Ng Quang Dng hc sinh lp 12A2 Ton trng THPT
chuyn KHTN, H Ni, c k bn tho v a ra nhng li gp qu bu, xc ng ti
liu c hon chnh hn. Mc d c gng ht sc nhng ti liu cn c nhiu thiu st, ti
rt mong nhn c s gp ph bnh ca bn c chuyn c hon thin hn.

Ti liu tham kho


[1 ] http://www.artofproblemsolving.com/community/q1h490078p2747906
90

Tp ch Epsilon, S 08, 04/2016


[2 ] http://www.artofproblemsolving.com/community/q1h569004p3338258
[3 ] Blog hnh hc s cp http://analgeomatica.blogspot.com/
[4 ] http://www.artofproblemsolving.com/community/c6h35309p220212
[5 ] Mathley No 1, (January 2014).
[6 ] DarijGrinberg, Circumscribed quadrilaterals revisited, 2012.
[7 ] The IMO Compendium.
[8 ] thi Vietnam Team Selection Tests 2015.

91

Tp ch Epsilon, S 08, 04/2016

92

MT S NG DNG CA CC V I CC
Trn Quang Hng (Trng THPT Chuyn KHTN, H Ni)
Nguyn Tin Dng (i hc Ngoi thng, H Ni)

Bi vit s tm tt li mt s kin thc v cc v i cc i vi mt ng trn


ng thi a ra mt s v d nng cao cho thy ng dng kh thay th ca cc khi
nim ny trong thc hnh gii ton.

1. M u
Khi nim im lin hp xut pht t vic ta chia iu ha on thng bi mt ng trn ngha
l khi cho ng trn ct on thng ti hai im th hai im lin hp iu ha vi hai u
mt on thng. Khi nim v cc v i cc c nh ngha thng qua khi nim v im lin
hp, iu c ngha l n gn cht vi cc khi nim v hng iu ha v chm iu ha. Tuy
nhin trong mt s bi ton vic dng cc i cc thc s l cn thit m vic s dng cc khi
nim iu ha thng thng khng th thay th c, hoc nu c mun thay th th li dn n
s rm r trong trnh by. Do chng ti vit bi ny vi mc ch tng kt li nhng chnh
trong cch s dng cng c cc i cc. Mt khc chng ti cng mun a ra mt s v d xc
thc cho thy cn thit phi dng cc i cc ch khng mun s dng khi nim ny mt cch
hnh thc theo kiu dng cng c m khng dng cng c.

2. Tm tt l thuyt
Trong mc ny ti s tm tt li mt s chnh t vic nh ngha cc i cc cho ti mt s
tnh cht hay dng gii ton. Cc v i cc c nhiu cch nh ngha nhng vi ng trn,
nh ngha sau theo chng ti l hay nht. Cch nh ngha ny c tham kho trong [3]. Cc
khi nim cc v i cc cng cc vn lin quan khc cc bn c th tham kho [1,2]. Ta bt
u t khi nim im lin hp
nh ngha 3. Cho ng trn .O/ hai im A; B gi l lin hp vi .O/ nu ng trn ng
knh AB trc giao vi .O/.
Khi nim cc v i cc c nh ngha thng qua khi nim im lin hp nh sau
nh ngha 4. Cho ng trn .O/ v im P . Tp hp tt c cc im lin hp vi P l mt
ng thng vung gc vi OP . ng thng gi l ng i cc ca P i vi .O/. im
P gi l cc ca ng thng i vi .O/.
nh l sau c th coi l nh l c bn nht ca cc khi nim v cc v i cc
93

Tp ch Epsilon, S 08, 04/2016


nh l 1 (La Hire). i vi cng mt ng trn th A nm trn i cc ca B khi v ch khi
B nm trn i cc ca A.
H Qu 2.0.1. i vi cng mt ng trn th cc ng i cc ng quy khi v ch khi cc
ca chng thng hng.
H Qu 2.0.2. i vi cng mt ng trn th t s kp ca chm i cc bng t s kp ca
hng cc tng ng.
nh l tip theo l mt cch khc ngoi nh ngha nhn bit khi nim im lin hp
H Qu 2.0.3. Cho ng trn .O/ v on thng AB ct .O/ ti M; N th A; B lin hp vi
.O/ khi v ch khi hng im .AB; MN / iu ha.
nh l sau nhn bit mt s im c bit khc trn ng i cc t kt hp nh ngha ta
s c thm nhiu tnh hung nhn ra cc v ng i cc
nh l 2. Cho ng trn .O/ v P .
i) Nghch o ca P qua .O/ nm trn ng i cc ca P i vi .O/.
ii) Nu k c cc tip tuyn PA; PB ti .O/ vi A; B thuc .O/ th A; B nm trn i cc
ca P i vi .O/
Cc ng dng ca cc i cc c th tm gn qua hai nh ngha, ba nh l v hai h qu trn.
Mc d nu trin khai cc khi nim ra th s cn ra nhiu nh l khc nhng chng ti mun
dnh li s linh hot cho bn c khi gii ton. Chng ti khng mun vit v l thuyt mt
cch qu hnh thc v rm r v quan im rng mun ng dng mt khi nim v nh l trong
hnh hc s cp th trc ht khi nim v nh l phi n gin v d hiu, nhng nh l
cng n gin m khng tm thng th ng dng trong gii ton hnh hc s cng cao.

3. Mt s nh l ng dng
nh l sau tham kho [4]
nh l 3 (nh l Salmon). Cho ng trn .O/ v P; Q l hai im bt k. Gi K; L l
ln lt l hnh chiu ca P; Q ln ng i cc ca Q; P i vi .O/. Chng minh rng
OP
PK
D
.
OQ
QL

V
O

Q
L

94

Tp ch Epsilon, S 08, 04/2016


Chng minh. Gi S; T l nghch o ca P; Q i vi .O/ th S; T nm trn ng i cc ca
P; Q i vi .O/. OP; OQ ln lt ct K T; LS ti U; V . Ta thy ngay t gic ST V U ni tip
OU
OV
nn OS:OU D OV:OT mt khc OS:OP D OT:OQ. T
D
nn U V k PQ. Ta
OP
OQ
PU
OU
OP
PK
D
D
D
d thy hai tam gic KP U v LQV ng dng g.g nn
, ta hon
QL
QV
OV
OQ
tt chng minh.
nh l sau rt quen thuc vi cc bn hc sinh ph thng Vit Nam qua cc bi ton Olympic,
tuy nhin tn gi ca nh l khng nm trong mt ti liu chnh thc no m c tham kho
qua [5,6]
nh l 4 (nh l Brokard). Cho t gic ABCD c th khng li ni tip trong ng trn .O/.
Gi AB ct CD ti E. AD ct BC ti G v AC ct BD ti G. Chng minh rng EF; F G; GE
ln lt l ng i cc ca G; E; F i vi ng trn .O/.

B
A
Q

P
G
O

Chng minh. Trn BC; AD ly P; Q sao cho cc hng im .BC; PF / v .AD; QF / iu


ha nh vy P; Q u l lin hp ca F i vi .O/ nn PQ l i cc ca F . Mt khc
.FP; BC / D 1 D .FQ; AD/ nn AB; CD v PQ ng quy ti E. ng thi .FP; BC / D
1 D .FQ; DA/ nn AC; BD v PQ ng quy ti G. T PQ i qua E; G hay EG l i
cc ca F . Tng t F G l i cc ca E. Vy hin nhin G l cc ca EF .
Bn cht ca li gii ny l vic dng ra cc im lin hp ca F i vi .O/ trn BC v AD.
Tuy nhin trong nhiu ti liu khi chng minh nh l ny thng dng ra cc tip tuyn i vi
.O/ i qua F . iu ny khng cn thit v cng cha thc s chnh xc v vi iu kin mt t
gic khng cn li th cha chc F nm ngoi .O/ dng tip tuyn.
nh l sau c pht biu tng qut trn Conic tham kho [7], trong bi vit ny chng ti
chng minh cho trng hp ng trn
nh l 5 (nh l Conway). Cho tam gic ABC v mt ng trn .K/ bt k. Gi X; Y; Z
ln lt l cc ca cc ng thng BC; CA; AB i vi .K/. Chng minh rng AX; BY; C Z
ng quy.
95

Tp ch Epsilon, S 08, 04/2016

A
Y
D
K
Z
H

E X

W
Chng minh. Gi Y Z; ZX; XY ct BC; CA; AB ti U; V; W , theo nh l Desargues ta cn
chng minh U; V; W thng hng, mt khc khi p dng nh l Menelaus cho tam gic trung
bnh th U; V; W thng hng khi v ch khi trung im ca AU; BV; C W thng hng. Ta li
d thy trc tm H ca tam gic ABC c cng phng tch vi cc ng trn ng knh
AU; BV; C W nn mun ch ra trung im ca AU; BV; C W thng hng ta ch cn chng minh
cc ng trn ng knh AU; BV; C W ng trc. Do Y; Z ln lt l cc ca CA; AB nn A
l cc ca Y Z. T hnh chiu D ca A ln Y Z chnh l nghch o ca A qua .K/. Tng t
ta c E; F . D thy D; E; F u nm trn cc ng trn ng knh AU; BV; C W mt khc
theo tnh cht nghch o th KD:KA D KE:KB D KF:KC nn K c cng phng tch vi
cc ng trn ny, iu c ngha KH chnh l trc ng phng ca cc ng trn ng
knh AU; BV; C W , ta hon thnh chng minh.
im ng quy c gi l tm thu x ca tam gic ABC v ng trn .K/. Hai tam gic
ABC v X Y Z c gi l hai tam gic lin hp i vi ng trn .K/. nh l trn cng l
mt tnh cht quan trng ca cc v i cc c ng dng nhiu trong cc bi ton v tam gic.
Ba bi ton sau c th coi l h qu trc tip ca bi ton trn
H Qu 3.0.4. Cho tam gic ABC v ng trn .K/ bt k. Chng minh rng i cc ca
A; B; C vi .K/ ct BC; CA; AB theo ba im thng hng.
H Qu 3.0.5. Cho tam gic ABC v P bt k. D; E; F l hnh chiu ca P ln BC; CA; AB.
Trn PD; PE; PF ln lt ly cc im X; Y; Z sao cho PD:PX D PE:P Y D PF :P Z.
Chng minh rng AX; BY; C Z ng quy.
96

Tp ch Epsilon, S 08, 04/2016


H Qu 3.0.6 (M rng tnh cht ca ct tuyn trc giao). Cho tam gic ABC v P bt k.
D; E; F ln lt nm trn PA; PB; P C sao cho PD:PA D PE:PB D PF :P C . Chng minh
rng ng thng qua D; E; F ln lt vung gc vi PA; PB; P C ct BC; CA; AB theo ba
im thng hng.
Qua nh l Brokard ta d thy rng trong mt t gic ni tip th tam gic to bi giao im hai
ng cho v cc cnh i t lin hp vi ng trn ngoi tip t gic . H qu sau c th
coi l pht biu ngc ca nh l Brokard.
H Qu 3.0.7. Nu mt tam gic t lin hp i vi mt ng trn th tm ca ng trn
phi l trc tm tam gic.
Tn gi v ni dung nh l quan trng sau c tham kho trong [13]
nh l 6 (nh l Steinbart). Cho tam gic ABC c ng trn ni tip .I / tip xc
BC; CA; AB ti D; E; F . X; Y; Z nm trn .I / sao cho DX; EY; F Z ng quy. Chng minh
rng AX; BY; C Z ng quy.

A
G

H
F

X
K

Chng minh. Gi tip tuyn ti X; Y; Z ca .I / ln lt ct BC; CA; AB ti U; V; W . D thy


ba ng ng quy DX; EY; F Z ln lt l i cc ca U; V; W i vi .I / nn U; V; W thng
hng. Gi AX; BY; C Z ln lt ct BC; CA; AB ti M; N; P . Gi UX ct CA; AB ti G; H th
t gic BC GH ngoi tip. Theo tnh cht quen thuc th CH; BG v DX ng quy ti K nn ta
c bin i t s kp .BC; UM / D A.BC; UM / D .HG; UX/ D K.HG; UX/ D .CB; UD/.
UB MB
UC DC
MB
UB 2 DC
T ta suy ra
W
D
W
hay
D

. Do AD; BE; CF ng quy
UC 2 DB
UC M C
UB DB
MC
MB NC PA
v U; V; W thng hng nn khi nhn cc t s tng t ta d thu c


D 1
M C NA PB
do AX; BY; C Z ng quy.
Chng minh trn do chng ti a ra s dng cc v i cc theo chng ti l rt mi cho nh
l kinh in ny. Chng minh ny da trn tng xut pht ca bn Ng Quang Dng hc
sinh lp 12 Ton THPT chuyn KHTN khi bn Dng gii mt bi ton tng qut hn.
Mt h qu n gin c rt ra t mt phn cch chng minh nh l trn nh sau
H Qu 3.0.8. Cho tam gic ABC c ng trn ni tip .I /. P thuc .I /. Tip tuyn ti P
MB
UB 2 DC
ca .I / ct BC ti U . AP ct BC ti M th
D

UC 2 DB
MC
97

Tp ch Epsilon, S 08, 04/2016


Vic pht biu li h qu trn s cho chng ra thy tm p dng rng hn ca h qu ny.
H Qu 3.0.9. Cho ng trn .K/ tip xc on BC ti D. T B; C v cc tip tuyn khc
BC ct nhau ti X . P thuc .I /. Tip tuyn ti P ca .I / ct BC ti U . XP ct BC ti M
MB
UB 2 DC

th
D
UC 2 DB
MC
Mc d tng chng cch pht biu th hai khng c ngha v n vn chnh l h qu th nht
nhng thc cht n c rt nhiu gi tr trong cc bi ton thc hnh khc nhau v ch rng
khi pht biu nh vy .K/ c th l ng trn bng tip tam gic XBC ch khng bt buc l
ng trn ni tip.
nh l sau l mt nh l kinh in ca hnh hc x nh c chng minh n gin bng cc
i cc v nh l La Hire
nh l 7 (nh l Brianchon). Chng minh rng cc ng cho chnh ca mt lc gic ngoi
tip ng quy.

M
B

S
O

F
R

N
C

P
Q

Y
Z
Chng minh. Gi s lc gic ABCDEF ngoi tip ng trn .O/. Cc tip im ca .O/
vi AB; BC; CD; DE; DF; FA l M; N; P; Q; R; S . Thy ngay giao im ca cc cp ng
thng MN v RQ, NP v SR, PQ v SM ln lt l cc ca cc ng thng BE; CF v
AD i vi ng trn .O/. Theo nh l Pascal cc im ny thng hng nn AD; BE; CF
ng quy.
Mt cch ngc li khi cng nhn nh l Brianchon ta hon ton c th chng minh nh l
Pascal bng cc i cc bng cch dng ra cc tip tuyn i vi ng trn ct nhau to ra lc
gic ngoi tip
nh l 8 (nh l Pascal). Chng minh rng giao im ca cc cnh i trong mt lc gic
ni tip thng hng.
98

Tp ch Epsilon, S 08, 04/2016

A
N

F
O

S
E

B
P

C
D

Y
Z
Chng minh. Xt lc gic ABCDEF ni tip ng trn .O/ ta cn chng minh giao im
X; Y; Z theo th t ca cc cp ng thng AB v DE, BC v FE, CD v FA thng hng.
Gi tip tuyn ti A; B; C; D; E; F ca .O/ ct nhau ln lt to thnh lc gic MNPQRS
ngoi tip nh hnh v. D thy X; Y; Z ln lt l cc ca cc ng thng MQ; PS v NR.
Theo nh l Brianchon th MQ; PS v NR ng quy nn X; Y; Z thng hng.
Lc gic trong hai nh l trn c th suy bin khng nht thit phi li nh hnh minh ha v
chng ta cng nn hiu mt linh hot cc khi nim "ng cho chnh" v "cnh i" ca lc
gic trong pht biu nh l.
Mc d khi t hai chng minh ny cnh nhau chng tng chng nh khng c ngha v dng
nh l ny chng minh nh l kia, nhng khng phi vy. Trong hnh hc s cp chng ta
bit nhiu cch khc chng minh nh l Pascal v Brianchon khi chng ta hon ton c th
dng nh l ny chng minh nh l kia thng qua phng php s dng cng c cc v i
cc nh trn. Tuy nhin mc ch chnh ca chng ti khi trnh by hai chng minh ny cnh
nhau lm ni bt s i ngu ca hai nh l kinh in l Pascal v Brianchon thng qua hai
khi nim i ngu quan trng khc ca hnh hc x nh l cc v i cc.
Nh vy cng vi nh l c bn ca La Hire th cc nh l ca Salmon, Brokard, Conway,
Steinbart, Brianchon v Pascal cng l cc nh l rt quan trng gn lin vi hnh hc x nh v
c trnh by mt cch thng nht trong bi vit ny thng qua cc khi nim v nh l ca
cc v i cc. Mt khc mi nh l trn u l cc nh l ln m nhng khai thc cng nh
tng qut chng c cp v vit nhiu n mc kh c th lit k ra ht, do vi mt
bi vit nh chng ti ch mun gii thiu cc nh l cng vi mt ci nhn thng nht di khi
nim cc v i cc ch khng c tham vng i su vo mt nh l no.

99

Tp ch Epsilon, S 08, 04/2016

4. Mt s v d
Cc bi ton v d chng ti ngh trong mc ny mang hi hng ch o l cc bi ton thi
Olympic ch khng phi l cc bi ton thuc v i tng nghin cu chuyn su. Trong mt
s bi ton chng ti s s dng cc nh l phn trc nhng chng ti cng khng c tham
vng dng ht tt c cc nh l, v mi nh l cn c nhng ng dng chuyn su m mt
bi vit di cng cha th vit ht.
Bi ton u tin tham kho [19]
Bi ton 1. Cho tam gic ABC vi ng trn ni tip .I /. P l im bt k. PA; PB; P C ct
BC; CA; AB ti K; L; N . Gi X; Y; Z ln lt l cc ca cc ng thng LN; NK; KL i
vi .I /. Chng minh rng AX; BY; C Z ln lt ct BC; CA; AB ti ba im thng hng trn
mt ng thng v ng thng ny tip xc vi .I /.

X
A

V
W
N

E
Q
L

F
I

Y
Chng minh. Gi AX; BY; C Z ln lt ct BC; CA; AB ti U; V; W . Trc ht ta s chng
minh YB; C Z ng quy vi EF , t ta d thy V W tip xc .I /. Tng t th U W; U V
tip xc .I / nn U; V; W thng hng trn ng thng tip xc .I /, tht vy. Gi DE; DF
ln lt ct KL; KN ti S; T do AD; BE; CF ng quy v AK; BL; CN cng ng quy nn
D.KA; EF / D 1 D K.DA; LN / do A; S; T thng hng. V B; Y ln lt l cc ca
DF; NK nn T l cc ca BY . Tng t S l cc ca C Z. Mt khc A l cc ca EF . V
A; S; T thng hng nn d thy BY; C Z v EF ng quy. Vy l iu phi chng minh.
Bi ton sau tham kho [18]
100

Tp ch Epsilon, S 08, 04/2016


Bi ton 2. Cho tam gic ABC ni tip ng trn .O/. ng trn .Ka / tip xc CA; AB v
tip xc trong .O/. Gi D l cc ca BC i vi .Ka /. Tng t c E; F . Chng minh rng
AD; BE; CF ng quy.

Q
N

Ka
X

Chng minh. Gi .Ka / tip xc CA; AB ti M; N th MN v i cc ca A i vi .Ka /. Vy


MN ct BC ti X th X l cc ca AD i vi .Ka /. T AD ct MN; BC ti Q; U th
.MN; QX / D 1 nn .BC; UX/ D A.MN; QX/ D 1. T tng t c cc giao im Y; Z
v U; V . Gi I l tm ni tip th IX; I Y; IZ theo th t vung gc vi IA; IB; IC theo bi
ton ct tuyn trc giao th X; Y; Z thng hng. T c AU; BV; C W ng quy hay ni cch
khc l AD; BE; CF ng quy.
Mt sai lm kh ph bin trong khi gii bi ny l sau khi ch ra X l cc ca AD v tng t
vi Y; Z th nhn xt X; Y; Z thng hng nn AD; BE; CF ng quy. Nguyn nhn ca sai lm
ny l do X l i cc ca AD vi ng trn .Ka / cn Y; Z tng ng l cc ca BE; CF vi
cc ng trn .Kb /; .Kc /. H qu ca nh l La Hire ch ng khi xt cc i cc vi cng
mt ng trn ch khng phi vi ba ng trn khc nhau.
Bi ton 3. Cho tam gic ABC ni tip ng trn .O/. ng trn .Ka / tip xc CA; AB
v tip xc trong .O/. Tng t c cc ng trn .Kb /; .Kc /. Gi D l cc ca ng thng
Kb Kc vi .Ka /. Tng t c E; F . Chng minh rng AD; BE; CF ng quy.
101

Tp ch Epsilon, S 08, 04/2016

Kb

Kc

I O
M

Ka
X

B
N

Chng minh. Gi .Kb /; .Kc / tip xc .O/ ti P; Q. Theo tnh cht quen thuc th PI i
qua trung im N cung AC cha B v QI i qua trung im M ca cung AB cha C .
ng thng qua I vung gc AI ct PQ ti X . D thy AI ? MN nn XI k MN , do
XIQ D NMI D IPQ. Do XI 2 D XP:XQ. Ta d thy tm ngoi tip ca tam gic
BIC l giao ca AI vi .O/ nn XI l tip tuyn ca .BIC / nn XI 2 l phng tch ca X
i vi .BIC / mt khc XP:XQ l phng tch ca X i vi .O/. Do X nm trn trc ng
phng ca .O/ v .BIC / chnh l BC . T vic BC l tip tuyn chung ca .Kb /; .Kc / cn
P; Q cng l cc tm v t ngoi ca .Kb /; .Kc / vi .O/ nn theo nh l Dlambert giao im
X ca PQ v BC phi l tm v t ngoi ca .Kb /; .Kc / nn Kb Kc i qua X. Theo gi thit D
l cc ca Kb Kc vi .Ka / m Kb Kc i qua X nn D v X lin hp vi .Ka /. Li d thy A v
X cng lin hp vi .Ka / nn AD l ng i cc ca X i vi .Ka /. Tng t vi BE; CF .
Theo chng minh bi trc th cc ng i cc ny ng quy.

Chng ta i tip ti bi ton sau l thi chn i tuyn KHTN vng 2 nm 2011-2012 ngy th
nht do tc gi xut
Bi ton 4. Cho tam gic khng cn ABC . ng trn ni tip .I / ca tam gic ABC tip
xc vi BC; CA; AB ln lt ti D; E; F . AD giao EF ti J . M; N di chuyn trn ng trn
.I / sao cho M; J; N thng hng v M nm v pha na mt phng cha C b AD, N nm v
pha na mt phng cha B b AD. Gi s DM; DN ln lt ct AC; AB ti P; Q.

a) Gi s MN giao PQ ti T . Chng minh rng T lun thuc mt ng thng d c nh.


b) Gi s tip tuyn ti M; N ca .I / ct nhau ti S . Chng minh rng S thuc d .
c) Gi s SJ giao BC ti K. Chng minh rng IK vung gc TD.
102

Tp ch Epsilon, S 08, 04/2016

S
A

E
P

F N

J
M

Li gii. a) Gi EF giao BC ti G. MN giao AG ti T . Ta s chng minh P; Q; T thng hng


t suy ra T l giao ca MN v PQ thuc AG c nh, tht vy, xt cc i cc vi ng
trn .I /. D thy G l cc ca AD nn G; J lin hp, J; A cng lin hp nn J l cc ca AG.
MN qua J ct AG ti T suy ra .MN; J T / D 1 suy ra D.MN; J T / D 1 D A.CB; DG/
t y suy ra T; P; Q thng hng.
b) S l cc MN suy ra S; J lin hp nn S thuc d l i cc ca J .
c) S l cc MN , T thuc MN nn S; T lin hp, J; T lin hp nn T l cc SJ , K thuc SJ
nn T; K lin hp, ta cng c D; K lin hp nn K l cc ca TD do IK vung gc TD.
Bi ton sau c tc gi m rng bi ton trong [8], li gii da theo tng ca Telv Cohl
trong [8]
Bi ton 5. Cho tam gic ABC ni tip ng trn .O/. Cc im E; F ln lt nm trn cnh
CA; AB. ng i trung qua A ct ng trn ngoi tip tam gic AEF ti P . OP ct EF
ti D v ct ng trn ngoi tip tam gic AEF ti Q khc P . DA ct .O/ ti T khc A.
Tip tuyn ti T ca .O/ ct BC ti S. Ly R thuc AD sao cho OR ? AQ. Chng minh rng
RS ? OR.

E
F V

U
P

L
B
S

K
W
103

Tp ch Epsilon, S 08, 04/2016


Chng minh. Gi AQ ct OR; EF ti U; V v ct .O/ ti W khc A. D thy .AP; EF / D 1
nn chm Q.AP; EF / D 1 chiu chm ny ln ng thng EF th .VD; EF / D 1 nn
chm A.VD; EF / D 1, chiu chm ny ln .O/ th hng .W T; BC / D 1. T nu T W
ct BC ti K th .SK; BC / D 1 nn T; K u lin hp vi S i vi .O/. Ni cch khc TK
l i cc ca S i vi .O/ nn SW tip xc .O/, t T; W u thuc ng trn ng
knh OS. Ta li c T WO D 90 TAW D TRO nn t gic RTOW ni tip, ng
trn ngoi tip chnh l ng trn ng knh OS. T RS ? OR.
Bi ton sau tham kho [9] l m rng IMO nm 1998
Bi ton 6. Cho ng trn .I / ni tip trong tam gic ABC . Gi K; L v M l cc im tip
xc ca ng trn ni tip tam gic ABC tng ng vi cc cnh AB; BC v CA. ng
thng MK v ML ct ng thng qua B v song song vi KL tng ng ti cc im Q v R.
ng trn vi ng knh QR ct .I / ti S; T . Chng minh rng ST chia i on thng KL.

R
BD
Q
L
S J
K

T
I
C

Li gii. Gi .D/ l ng trn vi ng knh QR. Ta d dng thy 4BLR  4BQK


nn BQ:BR D BK:BL D BK 2 . Gi ta c hai ln phng tch ca I vi ng trn .D/ l
IR2 C IQ2 RQ2 D .BI 2 C BR2 / C .BI 2 C BQ2 / .BR C BQ/2 D 2.BI 2 BR:BQ/ D
2.BI 2 BK 2 / D 2IK 2 . iu c ngha l ng trn .D/ v .I / l trc giao, nn DS; DT
l tip tuyn ca .I / suy ra ST l i cc ca D i vi .I /. Mt khc nu J l trung im EF
th J l cc ca ng thng qua B v vung gc vi IB i vi .I / nn J; D lin hp i vi
.I /. T S; T; J thng hng.
Bi ton sau l m rng thi v ch Nga nm 2015 trong [10]
Bi ton 7. Cho tam gic ABC c ng trn ni tip .I /. P; Q thuc BC sao cho .BC; PQ/ D
1. K cc tip tuyn PM; QN khc BC ti .I / vi M; N thuc .I /. Chng minh rng A; M; N
thng hng.
104

Tp ch Epsilon, S 08, 04/2016

A
N E
S
F

B D

M
P

Chng minh. Gi AP; AQ ct EF ti S; T . Do A.BC; PQ/ D 1 nn hng .EF; ST / D 1.


Vy S; T lin hp vi .I /, d thy S; A lin hp nn S l cc ca AT , suy ra S; Q lin hp vy
S nm trn DN . Tng t T nm trn DM . T chiu chm D.EF; ST / D 1 ln ng
trn .I / th hng .EF; MN / D 1 nn t gic EMF N iu ha. T MN i qua A.

Bi ton 8. Cho tam gic ABC c ng trn ni tip .I /. M; N; P thuc cc cnh BC; CA; AB
sao cho AM; BN; CP ng quy. K cc tip tuyn MX; N Y; PZ ca .I /. Chng minh AX; BY; C Z
ng quy.
105

Tp ch Epsilon, S 08, 04/2016

N'

F
P
Z
Y'
X

M'

X'

YN

I
Z'
M D

P'

Chng minh. Ly M 0 ; N 0 ; P 0 trn BC; CA; AB sao cho .BC; MM 0 / D .CA; N N 0 / D .AB; PP 0 / D
M 0B N 0C P 0A
MB NC PA
1 . Ch rng AM; BN; CP ng quy ta c 0  0  0 D


D1
MC NA PB
M C NA PB
nn M 0 ; N 0 ; P 0 thng hng. D; E; F l tip im ca .I / vi BC; CA; AB. K cc tip
tuyn M 0 X 0 ; N 0 Y 0 ; C 0 P 0 ca .I / th theo bi ton 6 ta thy AX 0 ; BY 0 ; C Z 0 cng chnh l
AX ; BY; C Z. DX 0 ; EY 0 ; F Z 0 c cc cc i vi .I / tng ng l M 0 ; N 0 ; P 0 thng hng nn
chng ng quy. p dng nh l Steinbart ta c AX 0 ; BY 0 ; C Z 0 ng quy hay AX; BY; C Z
ng quy.

Bi ton sau l mt b rt th v ca cc v i cc, tham kho [11]


Bi ton 9. Cho tam gic ABC v ng trn ni tip .I /. K l trc tm tam gic IBC . Chng
minh rng ng i cc ca K i vi .I / l ng trung bnh ng vi A ca tam gic ABC .
106

Tp ch Epsilon, S 08, 04/2016

K
A
L

S
Q

R
I
B

Li gii th nht. Gi hnh chiu ca A ln IB; IC l Q; R th d thy QR l ng trung


bnh ng vi A ca tam gic ABC . Gi .I / tip xc CA ti S. BI ct CK ti L. D thy cc t
gic ISLC v IQSA ni tip. T ISQ D IAQ D AIB 90 D ICA D ILS .
Vy IQ:IL D IS 2 , mt khc CK vung gc vi IQ ti L nn CK l i cc ca Q i vi
.I /. Vy K v Q lin hp i vi .I /. Tng t K v R lin hp i vi .I /. Vy QR chnh l
i cc ca K i vi .I /.

K
A
E
N
F
B

M
I
C

Li gii th hai. Gi D; E; F l tip im ca .I / vi BC; CA; AB. M; N l giao ca BI; CI


vi DE; DF th d thy MN l ng trung bnh ng vi nh A ca tam gic ABC . DF l
i cc ca B i vi .I / nn B; N lin hp vi .I /. Do BK ? I N l i cc ca N i vi
.I / nn K; N lin hp vi .I /. Tng t K; M lin hp vi .I /. Vy MN l i cc ca K i
vi .I /.

Bi ton sau c tc gi xut trong [12]


Bi ton 10. Cho tam gic ABC vi phn gic BE v CF ct nhau ti I . Ly im M sao cho
IM ? AB v AM ? IC . Ly im N sao cho I N ? AC v AN ? IB. ME ct NF ti P .
Gi K l trc tm tam gic IBC . Chng minh rng AK ? IP .
107

Tp ch Epsilon, S 08, 04/2016

S
R

F
M T

Q
I
C

Chng minh. Gi .I / tip xc CA; AB ti S; T . Gi AM; AN ln lt vung gc vi IC; IB


ti R; Q. D thycc im S;
 Q; T; R u nm trn ng trn ng knh AI . p dng nh
RSI
l Pascal cho b
suy ra ME; QR v ST ng quy. Tng t th NF; QR v ST
T QA
ng quy.T theo bi th bn ng thng ME; NF; QR; ST ng quy ti P . Ch A l
cc ca EF v theo bi trn th K l cc ca QR i vi .I /. Ta suy ra P l cc ca AK i
vi .I / vy IP ? AK.

Bi ton 11. Cho tam gic ABC c ng trn ni tip .I /. D; E; F l trung im BC; CA; AB.
i cc ca D i vi .O/ ct EF ti X. Y; Z c nh ngha tng t. Chng minh rng
X; Y; Z thng hng.
108

Tp ch Epsilon, S 08, 04/2016

Z
Ib

Ic
F

M
I
K
B

L
Y
Ia
Chng minh. Cc i cc ca D; E; F i vi .I / ct nhau to thnh tam gic H; K; L. E; F
cng lin hp H i vi .I / nn EF l i cc ca H i vi .I /. Theo bi ton 8 th H l trc
tm tam gic IBC . Tng t K; L l trc tm cc tam gic ICA; IAB. Gi Ia ; Ib ; Ic l tm
bng tip gc A; B; C ca tam gic ABC . D thy H; K; L i xng vi Ia ; Ib ; Ic qua D; E; F .
Ch rng hai tam gic Ia BC; Ia Ib Ic ng dng nn Ia D l ng i trung ca tam gic
Ia Ib Ic . T d thy DH; EK; FL ng quy ti M l im Lemoine ca tam gic Ia Ib Ic cng
l im Mittenpunkt ca tam gic ABC . X thuc i cc ca D; H i vi .I / nn D; H cng
lin hp X i vi .I /. Do DH l i cc ca X i vi .I / nn X lin hp M i vi .I /.
Tng t Y; Z cng lin hp M i vi .I /. Vy X; Y; Z cng thuc i cc ca M i vi
.I /.
S kin DH; EK; FL ng quy c th c suy ra trc tip bng vic s dng nh l Conway
cho tam gic HKL v ng trn .I / vi ch D; E; F l cc ca KL; LH; HK i vi .I /.
Bng vic s dng bi ton 8, qua lng knh cc v i cc chng ta c th gii quyt d dng
bi ton trong [14]. Ta i ti bi ton sau
109

Tp ch Epsilon, S 08, 04/2016


Bi ton 12. Cho tam gic ABC c D; E; F l trung im BC; CA; AB. Ly L; M; N trn
EF ; FD; DE sao cho DL; EM; F N l tip tuyn ca ng trn ni tip tam gic ABC .
Chng minh rng L; M; N thng hng.
Ta cn mt b sau
B 1. Cho tam gic ABC v mt ng trn .O/ ct cc cnh BC; CA; AB ti cc cp
im A1 ; A2 I B1 ; B2 I C1 ; C2 . Khi AA1 ; BB1 ; C C1 ng quy khi v ch khi AA2 ; BB2 ; C C2
ng quy.
B trn quen thuc vi nhiu bn c, xin khng chng minh y. Tr li bi ton

M
A
P

F
Z

E
V

I
R
U

X D

N
Chng minh. Gi ng trn .I / ni tip tam gic ABC tip xc vi BC; CA; AB ti X; Y; Z.
DL; EM; F N tip xc .I / ti U; V; W . XU; Y V; ZW ct nhau to thnh tam gic PQR. T
bi ton 8 d thy P; Q; R l cc ca EF ; FD; DE i vi .I / v cng l trc tm cc tam gic
IBC; ICA; IAB. Ch rng PX; QY; RZ ng quy ti I , p dng b trn cho tam gic
PQR v ng trn .I / ta c P U; QV; RW ng quy. Xt cc i cc vi .I /. L thuc cc i
cc EF; DU ca P; U nn L lin hp vi P; U . Do P U l i cc ca L. Lp lun tng t
QV; RW l i cc ca M; N . P U; QV; RW ng quy nn cc cc tng ng L; M; N thng
hng.
110

Tp ch Epsilon, S 08, 04/2016


Tng t vi k thut s dng trong bi ton 11, chng ta c th gii quyt bi ton tng qut do
Nguyn Phm t xut xem [14]
Bi ton 13. Cho tam gic ABC c ng trn ni tip .I /. A1 ; B1 ; C1 bt k trn BC; CA; AB.
A2 l giao im ca B1 C1 vi tip tuyn qua A1 ca .I / khc BC . B2 ; C2 c xc nh tng
t. Chng minh rng A2 ; B2 ; C2 thng hng khi v ch khi AA1 ; BB1 ; C C1 ng quy.
Li gii sau da theo Luis Gonzlez

B'

C'
A
C1
B1
B2

C2

E'

A2

F'

B0

A0
I

C0
D'
B

A1

A'

Chng minh. Gi .I / tip xc vi BC; CA; AB ti D; E; F . A1 A2 ; B1 B2 ; C1 C2 tip xc vi


.I / ti D 0 ; E 0 ; F 0 . DD 0 ; EE 0 ; FF 0 ct nhau to thnh tam gic A0 B0 C0 . A0 ; B 0 ; C 0 l giao im
ca BC; B1 C1 I CA; C1 A1 I AB; A1 B1 . Xt cc i cc vi .I /. A0 l giao cc i cc EE 0 ; FF 0
ca B1 ; C1 nn B1 ; C1 cng lin hp A0 ; do A0 l cc ca B1 C1 . Lp lun tng t B0 ; C0
l cc ca C1 A1 ; A1 B1 . A2 l giao cc i cc B1 C1 ; A1 D 0 ca A0 ; D 0 nn A2 l cc ca A0 D 0 .
Lp lun tng t ta c B2 ; C2 l cc ca B0 E 0 ; C0 F 0 ; A0 ; B 0 ; C 0 l cc ca A0 D; B0 E; C0 F .
Ta c cc khng nh sau tng ng
1) A2 ; B2 ; C2 thng hng
2) A0 D 0 ; B0 E 0 ; C0 F 0 ng quy (H qu 1.1)
111

Tp ch Epsilon, S 08, 04/2016


3) A0 D; B0 E; C0 F ng quy (p dng b 11.1 vi tam gic A0 B0 C0 v .I /)
4) A0 ; B 0 ; C 0 thng hng (H qu 1.1)
5) AA1 ; BB1 ; C C1 ng quy (p dng nh l Desargues cho hai tam gic ABC v A1 B1 C1 )
Bi ton sau tham kho [17]
Bi ton 14. Cho tam gic ABC vi tm ni tip I , cc trung tuyn AD; BE; CF ng quy ti
G. Trn EF; FD; DE ly X; Y; Z sao cho IX ? IA; I Y ? IB; IZ ? IC . Chng minh rng
X; Y; Z thng hng trn ng thng vung gc vi IG.

F
I G
M
B

J
D

Chng minh. Gi .I / l ng trn ni tip tam gic ABC . H l trc tm tam gic IBC . Theo
bi ton 8 H l cc ca EF vy H v X lin hp vi .I / vy ng i cc ca X l ng
thng qua H vung gc vi IX. Gi M l tm ni tip tam gic DEF v K l tm bng tip
gc A ca tam gic ABC . D thy IK l ng knh ca ng trn ngoi tip tam gic IBC
nn K v H i xng nhau qua D. Gi J i xng I qua M . D thy DM k IA. T theo
tnh cht ng trung bnh hnh thang th HJ k IA ? IX. Ni cch khc ng i cc ca
X lun i qua J . Tng t th cc ng i cc ca Y; Z cng i qua J . Vy X; Y; Z thng
hng v J chnh l cc ca ng thng qua X; Y; Z. D thy I; G; M; J thng hng nn ng
thng qua X; Y; Z vung gc vi IG.
Trong k thi Romanian Masters In Mathematics nm 2012 c bi ton hnh hc hay c vit li
nh sau
112

Tp ch Epsilon, S 08, 04/2016


Bi ton 15. Cho tam gic ABC c ng trn ni tip .I / v tm ngoi tip O. ng trn
.Oa / qua B; C tip xc vi .I /. Cc ng trn .Ob /; .Oc / c nh ngha tng t. Chng
minh rng tm ng phng ca .Oa /; .Ob /; .Oc / thuc OI .
Li gii sau da theo Da theo Ilya I. Bogdanov, Fedor A. Ivlev v Pavel A. Kozhevnikov trong
[15]

O
K

I
Y

l
H
Z

Chng minh. Gi .Oa /; .Ob /; .Oc / tip xc vi .I / ti X; Y; Z. Tip tuyn ti X ca .I / ct


BC ti M . Ta c MX 2 D MB  M C nn M thuc trc ng phng l ca .O/; .I /. Gi K l
cc ca l i vi .I / th OI k IK ? l nn K thuc OI . Xt cc i cc vi .I /. M thuc i
cc l ca K nn K thuc i cc DX ca M . Lp lun tng t EY; F Z i qua K. Cc tip
tuyn ti Y; Z ca .I / ct nhau ti U th AU l trc ng phng ca .Ob /; .Oc /. EF; Y Z ct
nhau ti H . H thuc i cc EF; Y Z ca A; U nn A; U cng lin hp vi H , v th AU l i
cc ca H . D thy H thuc i cc l ca K nn K thuc i cc AU ca H . Do K cng
phng tch vi .Ob /; .Oc /. Lp lun tng t ta c K c cng phng tch vi .Oa /; .Ob /; .Oc /.
Vy tm ng phng K ca .Oa /; .Ob /; .Oc / thuc OI .

Cng trong k thi Romanian Masters In Mathematics nm 2013, c bi ton nh sau


Bi ton 16. Cho t gic ABCD ni tip ng trn ! c AB ct CD ti P , AD ct BC ti
Q, AC ct BD ti R. M l trung im PQ. MR ct ! ti K. Chng minh rng ng trn
ngoi tip tam gic KPQ tip xc vi !.
113

Tp ch Epsilon, S 08, 04/2016

S
Q

A
B

R
K

T
O
D

Chng minh. Gi O l tm ca ng trn !. ng thng qua O vung gc vi MR ct


PQ; MR ln lt ti S; T . Theo nh l Brokard d thy R l trc tm tam gic OPQ. Ch
rng M l trung im PQ v T l hnh chiu ca O trn MR, theo kt qu quen thuc
T thuc ng trn .OPQ/. D c PQ l i cc ca R i vi ! nn S lin hp R i
vi !. Do MR?OS l i cc ca S i vi ! nn SK l tip tuyn ca .O/. Ta c
SK 2 D S T  SO D SP  SQ nn SK cng l tip tuyn ca .KPQ/. Vy ng trn ngoi tip
tam gic KPQ tip xc ! ti K.

Bi ton 17. Cho tam gic ABC ni tip .O/ c ng trn ni tip .I / tip xc vi BC; CA; AB
ti A0 ; B0 ; C0 . A1 l im chnh gia cung BAC ca .O/. Ly A2 trn IA1 sao cho A0 A2 k IA.
Ly A3 trn B0 C0 sao cho A2 A3 ? IA1 . B3 ; C3 c nh ngha tng t. Chng minh rng
A3 ; B3 ; C3 cng nm trn ng thng vung gc vi OI .

Chng minh. IA1 i qua tip im X ca ng trn A - mixtilinear ca tam gic ABC vi
.O/. ng thng qua I vung gc vi IA ct BC ti A4 . Theo tnh cht quen thuc XA4 l
phn gic ngoi ca BXC nn XA4 ? XI . Xt cc i cc vi .I /. D thy A0 A2 ? IA4 l
i cc ca A4 nn A2 lin hp vi A4 . Do XA4 ? IA2 l i cc ca A2 nn X lin hp
vi A2 . V th A2 A3 ? IX l i cc ca X nn X lin hp vi A3 . A3 thuc i cc B0 C0
ca A nn A lin hp vi A3 . T AX l i cc ca A3 i vi .I /. AX i qua tm v t
ngoi S ca .O/; .I / nn A3 lin hp vi S. Lp lun tng t B3 ; C3 u lin hp vi S . Vy
A3 ; B3 ; C3 nm trn i cc ca S vung gc vi OI .
114

Tp ch Epsilon, S 08, 04/2016

A1
A

C0
B0

O
I

A3

S
A2
B

A0

A4

X
Kt thc cc v d, chng ti cng c i li mun chia s. Cc v i cc l mt khi nim quan
trng trong hnh hc x nh cng l mt cng c quan trng trong vic gii ton hnh hc s cp.
Trong mt bi vit nh khng th no din t ht nhng bi ton s dng cng c ny. nh l
c bn trong phn ny l nh l La Hire c s dng nhiu nhng bn cnh nhng nh l
ln ca phn ny nh nh l Salmon, nh l Brokard, nh l Conway cng mi ch gii thiu
qua cng vi cch chng minh ch bi vit cn cha cp c ti nhng ng dng ht sc
phong ph ca cc nh l ny. Mt khc nhng ng dng ln ca cc i cc gn lin vi t
gic ngoi tip, lc gic cng cha c cp ti trong bi vit. Do bi vit ny mi l mt
ci nhn lt qua mt s bi ton ng dng hay ca cc i cc ch cha i su khai thc c th.
Mi bi ton trong chuyn ny cn c rt nhiu khai thc m c th vit thnh mt chuyn
ring. V mt bn cht cc i cc xut pht t khi nim im lin hp v hng iu ha v n
c pht trin ln nhng vn gi c tnh n gin nn c ng dng rt ln. Nu trong mt bi
ton ln trnh dng cc i cc, ngi gii hon ton c th lm thm mt cng on na
chng minh mt cch tr hnh cc tnh cht ca cc i cc v bn thn cc tnh cht rt n
gin. Tuy nhin r rng l iu ny khng c li v v mt s phm s gy ra kh hiu, do
chng ti vn lun khuyn khch cc bn, nht l cc em hc sinh thi Olympic vn nn dng v
ngh theo hng ny nu thy tin li.

5. Mt s bi ton luyn tp
Bi ton 18. Cho tam gic ABC c ng trn ni tip .I /. Gi H; K; L ln lt l trc tm
tam gic IBC; ICA; IAB. Chng minh rng AH; BK; CL ln lt ct BC; CA; AB theo ba
im thng hng trn mt ng thng tip xc .I /.
Bi ton 19. Cho tam gic ABC nhn ni tip ng trn .O/ c ng cao AD; BE; CF ct
nhau ti H . M l trung im BC . MH ct EF ti N . Tia ND ct .O/ ti P . Chng minh
115

Tp ch Epsilon, S 08, 04/2016


rng PA l phn gic EPF .
Bi ton 20. Cho tam gic ABC nhn ni tip ng trn .O/. P nm trong tam gic sao cho
AP ? BC . D; E; F l hnh chiu ca P ln BC; CA; AB. ng trn ngoi tip tam gic
DEF ct BC ti M khc D. MP ct EF ti N . Tia ND ct .O/ ti Q. Chng minh rng QA
l phn gic EQF .
Bi ton 21. Cho A; B; M l cc ca cc ng thng a; b; m vi ng trn .O/. Chng minh
rng
d.O; a/
d.B; a/
d.M; a/ d.A; m/
W
D
D
d.M; b/ d.B; m/
d.O; b/
d.A; b/
Trong d.X; d / k hiu khong cch t im X ti ng thng d .
Bi ton 22. Cho tam gic ABC ni tip ng trn .O/ v P nm trn phn gic BAC .
E; F l hnh chiu ca P ln CA; AB. OP ct EF ti Q. S thuc BC sao cho SQ ? OQ.
Gi AQ ct .O/ ti T khc A. Chng minh rng ST tip xc .O/.
Bi ton 23. Cho tam gic ABC c ng trn ni tip .I /. P l im bt k. PA; PB; P C
ln lt ct BC; CA; AB ti D; E; F . Trn EF; FD; DE ly X; Y; Z sao cho DX; EY; F Z
tip xc .I /. Chng minh rng X; Y; Z thng hng.
Bi ton 24 (Telv Cohl). Cho tam gic ABC vi P; Q l hai im ng gic. H; K l trc tm
ca tam gic PBC; QBC . Chng minh rng trung im HK l cc ca ng trung bnh ng
vi A ca tam gic ABC i vi ng trn Pedal ca P; Q.
Bi ton 25. Cho tam gic ABC ng trn ni tip .I /. Gi Ha ; Hb ; Hc l trc tm cc tam
gic IBC; ICA; IAB. on IA ct .I / ti A1 , tip tuyn ti I ca A1 ct Hb Hc ti A2 . Tng
t c B2 ; C2 . Chng minh rng A2 ; B2 ; C2 thng hng.
Bi ton 26. Cho tam gic ABC ni tip .O/. ng cao ti A; B; C ct ng trn ti im
th hai A0 ; B0 ; C0 . Tip tuyn ti B0 ; C0 ct nhau ti A1 . A2 l i xng ca A0 qua O. A1 A2
ct .O/ ti A3 . A0 A3 ct tip tuyn ti A ca .O/ ti A4 . Tng t c B4 ; C4 . Chng minh rng
A4 ; B4 ; C4 thng hng.
Bi ton 27. Cho I; K l tm ni tip v im Nagel ca tam gic ABC . D; E; F l tip im
ca .I / ni tip vi BC; CA; AB. M; N; P ln lt l trung im BC; CA; AB. Cc ng
thng qua D; E; F v vung gc vi AK; BK; CK ct cc cnh NP; PM; MN ti X; Y; Z.
Chng minh rng X; Y; Z thng hng trn ng thng vung gc vi IL trong L l im
Lemoine ca tam gic to bi ba tm bng tip.
Bi ton 28. Cho t gic ABCD ngoi tip ng trn .I /. Cc tip im tng ng vi
AB; BC; CD; DA l M; N; P; Q. MP giao NQ ti K. AB giao CD ti E, BC giao AD ti
F . Chng minh rng IK vung gc vi EF .
Bi ton 29 (Turkey TST 2009). Cho t gic ABCD ngoi tip ng trn .I; r/. Gi AB ct
CD ti E. AD ct BC ti F . AC ct DB ti G. K l hnh chiu ca I ln EF . Chng minh
rng IK:IG D r 2 .
116

Tp ch Epsilon, S 08, 04/2016


Bi ton 30 (Bi T12/445 - s thng 7 nm 2014). Cho t gic ABCD ni tip ng trn
.I /. Cc cnh AB; BC tip xc vi .I / ln lt ti M; N . Gi E l giao im ca AC v MN ,
F l giao im ca BC v DE. DM ct .I / ti T khc M . Chng minh rng F T l tip tuyn
ca .I /.
Bi ton 31 (VN TST 2003). Cho tam gic ABC ni tip ng trn .O/. A0 ; B0 ; C0 l trung
im cc ng cao AH; BK; CL ca tam gic ABC . ng trn .I / ni tip tam gic ABC
tip xc vi BC; CA; AB ti D; E; F . Chng minh rng A0 D; B0 E; C0 F v OI ng quy .

Ti liu tham kho


[1] http://www.cut-the-knot.org/Curriculum/Geometry/PolePolar.shtml
[2] http://mathworld.wolfram.com/Polar.html
[3] Nathan Altshiller-Court, College Geometry: An Introduction to the Modern Geometry of
the Triangle and the Circle, Dover Publications; 2 Rev Enl edition (April 19, 2007)
[4] http://mathworld.wolfram.com/SalmonsTheorem.html
[5] http://www.mit.edu/ alexrem/ProjectiveGeometry.pdf
[6] http://www.imomath.com/index.php?options=334&lmm=0
[7] http://www.maths.gla.ac.uk/wws/cabripages/triangle/conics.htm
[8] http://artofproblemsolving.com/community/c6t48f6h1213584
[9] http://analgeomatica.blogspot.com/2015/07/nhat-ky-mot-chuyen-i.html
[10] http://artofproblemsolving.com/community/c6t48f6h1211140
[11] http://www.xtec.cat/ qcastell/ttw/ttweng/resultats/r276.html
[12] http://analgeomatica.blogspot.com/2016/02/moi-tuan-mot-bai-toan-tuan-2-thang-2.html
[13] http://web.mit.edu/ darij/www/geometry2.html
[14] http://www.artofproblemsolving.com/community/c80019h307278/
[15] On circles touching the incircle, Journal of Classical Geometry
http://jcgeometry.org/Articles/Volume2/JCG2013V2pp43-52.pdf
[16] http://www.artofproblemsolving.com/community/q1h612004p3639577
[17] http://www.artofproblemsolving.com/community/c6h260772
[18] http://www.artofproblemsolving.com/community/c6h328754
[19] http://artofproblemsolving.com/community/c6t48f6h1221797

117

Tp ch Epsilon, S 08, 04/2016

118

TNH CHT HNH HC CA NG CONG


BC BA
Nguyn Tin Lm, Ng Quang Dng - THPT Chuyn KHTN, H Ni

Bi vit trnh by mt s tnh cht n gin ca ng cong bc ba.

1. M u
Trong hnh hc s cp, nh l Menelaus l mt nh l ni ting lin quan n bi ton chng
minh thng hng ca cc im. nh l ny c chng minh bng nh l Thales. Di y, ta
s a ra cch chng minh bng phng php ta .
Quy c trong bi vit, ta Descartes ca im M l .xM ; yM /.
nh l 1 (nh l Mnlaus). Cc im A1 , B1 , C1 ln lt chia cc on thng BC , CA, AB
theo t s , , , trong , , 2 R. Ba im A1 , B1 , C1 thng hng khi v ch khi D 1.
Chng minh. Chiu thun. Gi s ba im A1 , B1 , C1 thng hng v l ng thng i
qua ba im . Trong h trc ta Oxy, ng thng c phng trnh ax C by C
c D
t f .x/ D axC by C c. im A1 chia BC theo t s nn ta ca A1 l
 x0 v x
yC
B
C yB
A1
I
: V A1 2 nn
1
1
a
dn n D

xB xC
yB yC
Cb
C c D 0;
1
1

f .xB /
:
f .xC /

f .xC /
f .xA /
I D
: T , ta c ngay D 1:
f .xA /
f .xB /
Chiu o. Gi s ng thng MN ct ng thng AB ti P 0 v gi s P 0 chia on AB
theo t s 0 : Theo chiu thun th 0 D 1 m D 1 nn D 0 ; tc l P 0  P: Suy ra
M; N; P thng hng.
Mt cch tng t, ta cng c D

Tip theo, ta xt nh l Carnot l dng tng qut ca nh l Menelaus. Ni dung ca nh l


c pht biu di y:
nh l 2 (nh l Carnot). Gi s cc im Ai , Bi , Ci ln lt chia cnh BC , CA, AB theo t
s i ; i ; i ; vi i D 1; 2: Khi , 6 im A1 , A2 , B1 , B2 , C1 , C2 thuc mt conic khi v ch khi
1 2 1 2 1 2 D 1.
nh l trn c chng minh tng t nh nh l Menelaus c s dng thm nh l Viete cho
phng trnh bc 2.
119

Tp ch Epsilon, S 08, 04/2016

2. Mt s tnh cht ca ng cong bc ba


2.1. ng cong bc ba c phng trnh y D ax 3 C bx 2 C cx C d
Mc ny a ra cc tnh cht ca ng bc ba c phng trnh dng y D ax 3 C bx 2 C cx C d .
B 1. Khi b D 0, gi s A, B, C l ba im phn bit thuc ng cong bc ba K. Khi ,
A, B, C thng hng khi v ch khi xA C xB C xC D 0.
Chng minh. Gi s ng thng qua hai im A, B c phng trnh W x C y C D 0:
Chiu thun. Gi s C thuc ng thng AB; ta cn ch ra xA C xB C xC D 0: Tht vy, ta
A, B, C l nghim ca h
(
x C y C D 0;
ax 3 C cx C d D y:
Thay y t phng trnh th hai vo phng trnh u tin, ta c phng trnh honh giao
im ca .C / v l
ax 3 C . C b/x C C c D 0:
V xA , xB , xC l nghim ca phng trnh trn nn theo nh l Viete, ta c ngay xA CxB CxC D
0:
Chiu o. Gi s xA C xB C xC D 0: Gi C 0 l giao im ca K v ng thng AB, theo
phn thun th xA C xB C xC 0 D 0; dn n xC D xC 0 hay C  C 0 : Do , A, B, C thng
hng.
T b trn, ta thu c mt kt qu p lin quan ti ng cong bc ba K trong trng hp
b D 0 c trnh by trong mnh di y.
Mnh 1. Khi b D 0; gi s A, B, C l ba im phn bit thuc ng cong K. Tip tuyn
ca K ti cc im A, B, C ct ng cong K ti cc im th hai A0 , B 0 , C 0 . Chng minh rng
A0 , B 0 , C 0 thng hng khi v ch khi A, B, C thng hng.
Chng minh. Gi s A, B, C l ba im thng hng thuc ng cong K. Phng trnh tip
tuyn ca K ti A l y D .3axA2 C c/.x xA / C axA3 C cxA C d: Do vy, xA l nghim kp
ca phng trnh
ax 3 C cx C d D .3axA2 C c/.x

xA / C axA3 C cxA C d

hay
.x

xA /2 .x C 2xA / D 0:

Phng trnh trn c hai nghim xA ; 2xA nn honh ca im A0 l xA0 D 2xA : Mt cch
tng t, ta cng c xB 0 D 2xB ; xC 0 D 2xC : Suy ra xA0 C xB 0 C xC 0 D 2.xA C xB C xC /:
T y, p dng b 1 ta thy ngay A0 , B 0 , C 0 thng hng khi v ch khi A, B, C thng
hng.
Tip theo, ta s xt mt m rng ca mnh 1.
Mnh 2 (M rng mnh 1). Khi b D 0; xt hai ng thng d1 , d2 : Gi s di ct K ti ba
im Ai , Bi , Ci vi i D 1; 2. Khi , giao im ca cc ng thng A1 A2 , B1 B2 , C1 C2 vi
ng cong K thng hng.
120

Tp ch Epsilon, S 08, 04/2016


Chng minh. Gi A3 , B3 , C3 tng ng l giao im ca cc ng thng A1 A2 , B1 B2 , C1 C2
vi ng cong K v gi s ai , bi , ci ln lt l honh ca cc im Ai , Bi , Ci vi i D 1; 2; 3.
Theo b 1 th ta phi c
a1 C a2 C a3 D b1 C b2 C b3 D c1 C c2 C c3 D 0
Suy ra 3iD1 .ai C bi C ci / D 0. Nhng v Ai , Bi , Ci thuc ng thng di vi i D 1; 2 nn
ai C bi C ci D 0 vi mi i D 1; 2: T , ta phi c a3 C b3 C c3 D 0: Cng theo b 1 th
ba im A3 , B3 , C3 thng hng. Php chng minh hon tt.
Trong bi ton trn, ta ch cn cho hai ng thng d1 ; d2 trng nhau th cc ng thng
A1 A2 ; B1 B2 ; C1 C2 s suy bin thnh tip tuyn ca ng cong K v ta thu c li kt qu
mnh 1. Mt cu hi t ra l liu kt qu trn cn ng khng vi ng cong K bt k? Ta
xt bi ton m rng sau y:
Mnh 3 (M rng mnh 2). Xt ng cong K W y D ax 3 C bx 2 C cx C d , vi a 0
v hai ng thng d1 , d2 : Gi s rng di ct ng cong K ti ba im phn bit Ai , Bi , Ci
vi i D 1; 2. Gi A3 , B3 , C3 ln lt l giao im ca cc ng thng A1 A2 , B1 B2 , C1 C2 vi
ng cong K. Khi , ba im A3 , B3 , C3 thng hng.

Hnh 12.1: A3 , B3 , C3 thng hng

Chng minh. Ta chia lm hai trng hp.


1. A1 A2 , B1 B2 , C1 C2 i mt ct nhau.
Gi A, B, C ln lt l giao im ca cc cp ng thng .B1 B2 ; C1 C2 /, .C1 C2 ; A1 A2 /,
.A1 A2 ; B1 B2 /. Gi s im Ai chia on BC theo t s i ; im Bi chia on CA theo
t s i ; im Ci chia on AB theo t s i , vi i D 1; 2; 3. p dng nh l Carnot
cho ng cong K bc 3 vi 4ABC ta c 1 2 3 1 2 3 1 2 3 D 1: Nhng v ba im
Ai , Bi , Ci thng hng (cng nm trn ng thng di )nn p dng nh l Menelaus cho
4ABC vi ct tuyn d1 ta c i i i D 1; vi i D 1; 2: Suy ra 3 3 3 D 1; ngha l A3 ,
B3 , C3 thng hng theo nh l Mnlaus.
2. Trong cc cp .B1 B2 ; C1 C2 /, .C1 C2 ; A1 A2 /, .A1 A2 ; B1 B2 / c t nht 1 cp song song.
Lc ny ta thc hin mt php chiu xuyn tm bin Ai , Bi , Ci thnh A0i , Bi0 , Ci0 , ng
bc ba K thnh K0 . Hin nhin tn ti php chiu tha mn A01 A02 , B10 B20 , C10 C20 i mt
ct nhau. Lc ny ta li quay v trng hp 1.
121

Tp ch Epsilon, S 08, 04/2016

bi ton trn, nu cho hai ng thng d1 , d2 trng nhau th cc ng thng A1 A2 , B1 B2 ,


C1 C2 s suy bin thnh tip tuyn ca ng cong K th khi kt qu mnh 1 cng ng
trong trng hp b 0 hay ng vi ng cong bc ba tng qut.

2.2. ng cong bc ba bt k
trn, ta trnh by mt s tnh cht ca ng cong bc ba c phng trnh quen thuc. Phn
ny s cp n ng cong bc ba bt k, m trong ta Descartes, phng trnh c cho
di dng hm n:
ax3 x 3 C ay3 y 3 C ax2y x 2 y C axy2 xy 2 C bx2 x 2 C by2 y 2 C bxy xy C cx x C cy y C d D 0
Trong cc h s ax3 , ay3 , ax2y , axy2 , bx2 , by2 , bxy , cx , cy , d l cc s thc v ax3 , ay3 , ax2y ,
axy2 khng ng thi bng 0.
Ni n ng cong bc 3 th ta c mt kt qu rt ni ting:
nh l 3 (nh l Cayley-Bacharach). Hai ng cong bc 3 ct nhau ti 9 im (theo nh l
Bezout, hai ng cong bc m, n ct nhau ti m  n im). Mt ng cong bc 3 khc i qua 8
trong s 9 im th cng i qua im th 9.
nh l Cayley-Bacharach [1] c th c suy bin: thay ng cong bc 3 bi hp ca 3 ng
thng, hay hp ca 1 conic v 1 ng thng,... Do , mt trong nhng ng dng ca nh
l ny l chng minh 3 im thng hng, hay 6 im thuc mt conic. Chng hn nh nh l
ny mang n mt chng minh ngn gn cho mnh 2 trong 2.1: 2 ng cong bc 3 l K
v A1 ; A2 ; A3 [ B1 ; B2 ; B3 [ C1 ; C2 ; C3 c 9 giao im l Ai , Bi , Ci vi i D 1; 2; 3. Khi
ng bc 3 suy bin A1 ; B1 ; C1 [ A2 ; B2 ; C2 [ A3 ; B3 i qua 8 trong s 9 im trn nn cng
s cha im C3 . iu ny ngha l A3 , B3 , C3 thng hng.
H qu. Cc im A, B, C , D, P , P1 , P2 thuc ng cong bc ba K. 6 im A, B, C , D, P1 ,
P2 thuc mt conic khi v ch khi giao im khc A, B, C , D ca AB, CD vi K v P thng
hng.

Chng minh. AB, CD ct K ti M , N v M , N , P thng hng. K v cubic suy bin A; B; M [


C; D; N [ P; P1 ; P2 c 9 giao im A, B, C , D, M , N , P , P1 , P2 . Theo nh l CayleyBacharach, ng bc ba suy bin ABCDP1 [ M; N; P (trong ABCDP1 l conic i qua 5
im A, B, C , D, P1 ) i qua P2 nn A, B, C , D, P1 , P2 thuc mt conic.
Phn o, A, B, C , D, P1 , P2 thuc mt conic. Xt ba ng bc ba K, A; B; M [ C; D; N [
P; P1 ; P2 , ABCDP1 P2 [ M; N t p dng nh l Cayley-Bacharach ta suy ra M , N , P
thng hng.
Mt s bi ton khc c chng minh p dng nh l Cayley-Bacharach tng t trn, bn c
xem ti [2].
122

Tp ch Epsilon, S 08, 04/2016

Hnh 12.2: Mt ng dng trc tip ca nh l Cayley-Bacharach

Ti liu tham kho


[1] Cayley-Bacharach theorem, Wikipedia
[2] Generalization of some triangle geometry results associated with cubics
http://blogcuaquangduong.blogspot.com/2016/03/
generalizations-of-some-triangle.html

123

Tp ch Epsilon, S 08, 04/2016

124

BIU DIN S NGUYN DNG DI


DNG TNG CC S CHNH PHNG
Hong Cao Phong

Chuyn nghin cu bi ton Waring trong trng hp k D 2. Trong chuyn


, ta s chng minh g.2/ D 4. lm c iu , ta ch cn ch ra c mt s s
nguyn dng khng th biu din di c dng tng hai s chnh phng, mt
s s khng th biu din c di dng tng ba s chnh phng, nhng mi s
nguyn dng u biu din c di dng tng ca bn s chnh phng. Ngoi
ra, chng ta s i tm p n cho cu hi: "Nhng s nguyn dng no biu din
c di dng tng hai s chnh phng v ba s chnh phng". Cui cng, chng
ta s xem xt bi ton Waring tng qut v cc vn m rng cho hm g.k/.

1. Gii thiu
Vic tm cch biu din mt s nguyn dng di dng tng cc s chnh phng c rt
nhiu i tng quan tm, t nhng ngi yu ton cho n nhng nh ton hc.
Vo nm 1632, Albert Girard l ngi u tin a ra nhn nh: mt s nguyn t l ng d
vi 1 mod 4 l tng ca hai s chnh phng, iu ny c cng b vo nm 1634, sau ci
cht ca ng. Fermat c cho l ngi u tin a ra li gii cho bi ton, v n c a vo
mt l th ca ng gi Marin Mersenne vo ngy 25 thng 12 nm 1640.
Tuy nhin, trong bc th, Fermat khng a ra chng minh cho khng nh ca mnh. Li gii
u tin c tm ra bi Euler vo nm 1747, khi ng 40 tui. Mt cch t nhin, nh l Fermat
v tng hai s chnh phng dn n cu hi: Tm gi tr nh nht ca n sao cho mi s nguyn
dng u c th biu din bng tng ca khng qu n s chnh phng". y l trng hp
ring ca bi ton Waring khi k D 2.
Trong chuyn , ta s chng minh n D 4 v ch ra nhng s nguyn dng no c th biu din
di dng tng ca hai hoc ba s chnh phng.
Trong mc 2, ta chng minh mt s nguyn t c th biu din di dng tng ca hai s chnh
phng khi v ch khi n khng ng d vi 3 mod 4 v tr li cu hi: "Nhng s nguyn dng
no c th biu din c di dng tng ca hai s chnh phng?"
Trong mc 3, ta s chng minh mi s nguyn t u c th biu din c di dng tng ca
bn s chnh phng qua chng minh mi s nguyn dng u c th biu din c di
dng tng ca bn s chnh phng.
125

Tp ch Epsilon, S 08, 04/2016


Trong mc 4, ta chng minh mt s nguyn dng c th biu din c di dng tng ca ba
s chnh phng khi v ch khi n c dng 4a .8n C 7/. Mc ny s cp n hnh hc s hc
v nh l Minkowski.
Trong mc 5, ta s a ra thm thng tin v bnh lun xoay quanh bi ton Waring tng qut

2. Biu din mt s nguyn dng di dng tng ca hai


s chnh phng
Trc ht, chng ta quan tm n bi ton : Nhng s nguyn t no c th biu din c di
dng tng ca hai s chnh phng?, p n bi ton dn n nh l mang tn Fermat v tng
hai s chnh phng.
B 2.1. p l mt s nguyn t cho trc. Nu p  3 .mod 4/ v x 2 C y 2 chia ht cho p th
x chia ht cho p v y chia ht cho p.
Chng minh. Gi s .x; p/ D .y; p/ D 1, x 2  y 2 .mod p/ dn n x p 1  . 1/
p 1
suy ra . 1/ 2  1 .mod p/, suy ra 1  1 .mod p/ dn n iu v l.

p 1
2

yp

.mod p/

nh l 2.1 (nh l Fermat v tng hai s chnh phng). S nguyn t p c th biu din
c di dng tng hai s chnh phng khi v ch khi p 6 3 .mod 4/.
Chng minh. Gi s p D 4k C 3 biu din c di dng tng hai s chnh phng x; y. Theo
b 2, x chia ht cho p v y chia ht cho p. Suy ra p chia ht cho p 2 . V l.
Nu p D 2 th p D 12 C 12 .
Nu p D 4k C 1 th 1 l s chnh phng modulo p ([5]), tn ti a 2 N tha mn a2 
1 .mod p/.
p 
p , xt .1 C q/2 s c dng x C ay vi x D 0; 1; : : : ; q v y D 0; 1; : : : ; q.
t q D
Do .q C 1/2 > p, tn ti .x1 ; y1 / .x2 ; y2 / tha mn x1 C ay1  x2 C ay2 .mod p/ nn
.x1 x2 /2  .y1 y2 /2 .mod p/.
p
p
V .x1 x2 /  q < p v .y1 y2 /  q < p, ta c c .x1 x2 /2 C .y1 y2 /2 D p.
nh l 2.1 c chng minh.
By gi, chng ta xem xt : Nhng s t nhin no c th biu din c di dng tng ca
hai s chnh phng?".
B 2.2. Tch ca hai s, vi mi s l tng ca mt s chnh phng, cng l s chnh phng.
Chng minh. Gi s m D a2 C b 2 v n D c 2 C d 2 , suy ra mn D .ac C bd /2 C .ad bc/2
Y
Y
nh l 2.2. t n D 2r
pisi
qit i vi pi  1 .mod 4/ v qi  3 .mod 4/, n c th biu
din c di dng tng hai s chnh phng khi v ch khi ti chn vi mi i
126

Tp ch Epsilon, S 08, 04/2016


Chng minh. Gi s n c th biu din c di dng hai tng hai s chnh phng v tn ti
ti l: n D x 2 C y 2 D q t b; .b; q/ D 1.
Theo b 2, x D qx1 ; y D qy1 . iu ny dn n x12 C y12 D q t 2 b.
Sau mt s hu hn bc lp li, ta thu c:
q.xk 2 C yk 2 / D b;
dn n iu v l.
Gi D l tp hp

n j n 2 N; n D x 2 C y 2

Y s
Gi s ti chn vi mi i . Do 2 2 D; pi 2 D, b 2.2 ch ra rng m 2 D vi m D 2r
pi 1 .
Y t
Tn ti x;
E y 2 N tha mn x 2 C y2 D m. Do ti chn vi mi i nn
qi i D h2 . V vy,
n D .xh/2 C .yh/2 .
nh l 2.2 c chng minh.

3. Biu din mt s nguyn dng di dng tng ca


bn s chnh phng
Trong mc ny tm thi chng ta s khng xt n vn biu din mt s nguyn dng thnh
tng ba s chnh phng do khng c mt li gii hon ton s cp cho vn trn. V li, li
gii cho nh l bn s chnh phng c phn tng t vi li gii cho nh l hai s chnh phng
nn ta s quay li sau khi chng minh thnh cng nh l Lagrange v tng ca bn s chnh
phng.


B 3.1. t D D n j n D x 2 C y 2 C z 2 C t 2 I n; x; y; z; t 2 N . Nu m; n 2 D th mn 2
D.
Chng minh. Gi s m D a2 C b 2 C c 2 C d 2 v n D e 2 C f 2 C g 2 C h2
mn D .aeCbf CcgCdh/2 C.af

beCch dg/2 C.ag bh ceCdf /2 C.ahCbg cf

de/2

Suy ra mn 2 D.

B 3.2. Gi s p l mt s nguyn t l cho trc, khi tn ti 1  k < p sao cho kp 2 D.



p
Chng minh. Xt hai tp A D x 2 ; x D 0; 1; : : : ;

v B D

y2


p
1 ; y D 0; 1; : : : ;

2
2
Hin nhin, mi phn t ca A u phn bit theo modulo p, mi
phn
t
ca
B
cng
vy.
Ta
li

p 1
c: jAj C jBj D p C 1. Suy ra tn ti x; y 2 0; 1; : : : ;
sao cho x 2  y 2 1 .mod p/
2
nn kp D x 2 C y 2 C 12 C 02 ) kp 2 D
127

Tp ch Epsilon, S 08, 04/2016


Do kp D x 2 C y 2 C 1 <

p2
p2
C
< p 2 nn k < p.
4
4

t M D f1  k < p; kp 2 Dg. Theo b 3.2, M .


Gi s m l phn t b nht ca tp hp M ,
nh l 3.1. Ta s chng minh m D 1 t suy ra p 2 D vi mi s nguyn t p.
Chng minh. Gi s 1 < m < p, mp 2 D. Ta c, mp D x 2 C y 2 C z 2 C t 2 .
Nu m chn th mp D x 2 C y 2 C z 2 C t 2  a 2 0; 2 .mod 4/
Trng hp 1: Nu x; y; z; t cng chn hoc cng l
xCy x y zCt z t
;
;
;
2Z
2
2
2
2
v

.x C y/2
.x y/2
.z C t/2
.z t/2
m
m
C
C
C
D p)
2M
4
4
4
4
2
2
iu ny mu thun bi m l phn t nh nht trong M .
Trng hp 2: Nu x; y chn v z; t l
xCy x y zCt z t
;
;
;
2Z
2
2
2
2

Lp lun tng t, ta cng suy ra c mu thun


 vi nh ngha ca m.
m 1
Nu m l: Xt S D 0; 1; 2; : : : ;
. Do S l mt h thng d y mod m, tn ti
2
a; b; c; d 2 S sao cho x  a .mod m/; y  b .mod m/; z  c .mod m/; t  d .mod m/. V
a2 C b 2 C c 2 C d 2  0 .mod m/ nn tn ti k tha mn a2 C b 2 C c 2 C d 2 D km:
T a2 C b 2 C c 2 C d 2 < m2 suy ra 0  k < m.
Mt ln na, chng ta xt hai trng hp:
Trng hp 1: k D 0 dn n a D b D c D d D 0 suy ra x  y  z  t  0 .mod p/ nn
m2 j x 2 C y 2 C z 2 C t 2 D mp ng ngha vi m chia ht p dn n s mu thun.
Trng hp 2: 1  k < m. Theo b 3.1, ta c:
mp:km D .axCbyCczCdt/2 C.bx ayCdz ct/2 C.cx dy azCbt/2 C.dxCcy bz at/2 :
Do
X D ax C by C cz C dt  a2 C b 2 C c 2 C d 2  0 .mod m/
Y D bx

ay C dz

Z D cx

dy

T D dx C cy

ct  ab

az C bt  ca
bz

ab C dc C cd  0 .mod m/
db

at  da C cb

ac C bd  0 .mod m/
bc

ad  0 .mod m/

X 2 C Y 2 C Z 2 C T 2 D m2 .X12 C Y12 C Z12 C T12 / D m2 kp. kp 2 D dn n k 2 M . V


l.
nh l 3.2 (nh l Lagrange v bn s chnh phng). Mi s t nhin u c th biu din
c di dng tng ca bn s chnh phng
128

Tp ch Epsilon, S 08, 04/2016


Chng minh. p dng b 3.1 v nh l 3.1, nh l Lagrange v bn s chnh phng tr nn
hin nhin dn n g.2/  4.
Cho n gi, ta gn nh hon thin mc ch chnh ca chng ta. iu duy nht cn li l
chng minh c nhng s t nhin khng th biu din bng tng ca ba s chnh phng.

4. Biu din mt s nguyn dng di dng tng ca ba


s chnh phng
Nh ni trn, khng c mt li gii hon ton s cp cho nh l Legendre v tng ca ba s
chnh phng. Tuy nhin, bng cch s dng nh l Minkowski: "bt k mt tp li trong Rn
i xng qua gc ta v c th tch ln hn 2n d.L/ u cha t nht mt im nguyn khc
khng", da vo chng minh ca Ankeny vo nm 1957, ta c th a ra mt li gii rt p cho
bi ton trn.
nh l 4.1. S t nhin m c th biu din di dng tng ca ba s chnh phng khi v ch
khi m khng c dng 4a .8n C 7/.
Chng minh. Nu m l mt s nguyn dng c dng 4a .8n C 7/ th m khng th biu din
c bng tng ca ba s chnh phng:
Gi s m c th biu din di dng tng ca ba s chnh phng: 4a .8n C 7/ D m D
x 2 C y 2 C z 2 . Bi mt s chnh phng ch c th ng d vi 0; 1 hoc 4 theo modulo 8 nn
m D x 2 C y 2 C z 2  b 2 .0; 1; 2; 3; 4; 5; 6/ .mod 8/.
Nu a > 0, ta c m  0 .mod 4/, x 2 C y 2 C z 2  b 2 .0; 4/ .mod 8/ nn x 2  y 2  z 2 
0 .mod 4/ suy ra x D 2x1 ; y D 2y1 ; z D 2z1 dn n x12 C y12 C z12 D 4a 1 .8k C 7/.
Sau mt s hu hn ln lp li bc trn, ta thu c:
xa2 C ya2 C za2 D 8k C 7  7 .mod 8/;
V l. Sau khi chng minh c chiu "ch khi", Ta thnh cng trong vic ch ra rng g.2/ D 4.
Tuy nhin, ta s tip tc chng minh chiu khi" vn phc tp hn nhiu.
Nu m khng c dng 4a .8n C 7/ th m c th biu din c di dng tng ca ba s chnh
phng:
Ta ch cn chng minh bi ton trong trng hp m l mt s chnh phng t do (square free)
(nu khng, ta vn c th coi m nh vy ). Do tng ca ba s chnh phng khng th ng d
vi 7 .mod 8/, Ta gii quyt hai trng hp ring bit:
Trng hp 1: m  3 .mod 8/. Theo nh l Dirichlet v cp s cng ([3]), tn ti mt s nguyn
t q tha mn 2q  1.mod m/ v q  1 .mod 4/. Cng thc sau c s dng k hiu Jacobi
([5]).
Ta c:
    
m
1
m
m
.
/D
D
(Do q  1.mod 4/)
q
q
q
q
129

Tp ch Epsilon, S 08, 04/2016


D

q

(Theo lut thun nghch bnh phng)


m  
2
q
D
(Do m  3 .mod 8/)
m
m
2q
D.
/
m
D1
Tn ti s nguyn b tha mn b 2  m .mod q/ hay b 2 kh D m.k 2 Z/.
Do 1 k  1 .mod 4/, k D 4k1 Dn n b 2  m .mod 4q/.


Xt li nguyn: L D .x; y; z/ 2 Z3 j x  y .mod m/; y  bz .mod 2q/ . C th tch n
v l 2mq trong R3 v mi vec-t .x; y; z/ 2 L u tha mn:
2qx 2 C y 2 C mz 2


x 2 C y 2 .mod m/  0 .mod m/

 .bz/2 C mz 2 .mod 2q/  0 .mod 2q/:


p
4 .2 q m/3
2
2
2
Hnh Ellipsoid E: 2qX C Y C mZ  4mq c th tch
, ln hn 23 :2q m. Theo
p
3
2q m
nh l Minkowski, tn ti vec-t khc khng .x; y; z/ 2 L \ E sao cho 2qx 2 C y 2 C mz 2 
0 .mod 2q m/ nn 2qx 2 C y 2 C mz 2 D 2q m.
chng minh m c th biu din di dng tng ca ba s chnh phng ta s chng minh
y 2 C mz 2
biu din c di dng tng hai s chnh phng. nh l 2.2 s gip chng ta
2q
gii quyt iu ny. Ta ch cn chng minh: Tt c nhng s nguyn t p tha mn p > 2 v
p .y 2 C mz 2 / D 2n C 1 ([6]) u ng d vi 1 .mod 4/.
Nu p khng chia ht m
y 2 C mz 2  0 .mod p/ dn n y 2  mz 2 .mod p/
Tn ti z 0 sao cho zz 0  1 .mod p/ nn .z 0 y/2  m .mod p/ suy ra m l mt s chnh
phng .mod p/. Hn na, v x 2  m .mod p/ nn m l mt s chnh phng mod p

     
m
1
m
1
D
D
1D
p
p
p
p
suy ra
p  1 .mod 4/
Nu p chia ht m, p cng s chia ht x v y, dn n mz 2  2q m .mod p 2 /
Do m l mt s chnh phng t do nn 2q l s chnh phng mod p v 2q  1 .mod p/, ta
thu c: p  1 .mod 4/
y 2 C mz 2
Trong c hai trng hp, ta u c p  1.mod 4/ nn
c th biu din c di
2q
dng tng ca hai s chnh phng dn n m c th biu din c di dng tng ba s chnh
phng.
Trng hp 2: Nu m 6 3 .mod 8/, Chng minh gn nh tng t ngoi tr mt s thay i
nh.
130

Tp ch Epsilon, S 08, 04/2016


Theo nh l Dirichlet v cp s cng, tn ti s nguyn t q tha mn:
8

<1 .mod 4/ khi m  1; 5 .mod 8/


q  1 .mod m/; q 
1 .mod 8/ khi m  2 .mod 8/

: 5 .mod 8/ khi m  6 .mod 8/


D dng chng minh m l s chnh phng mod q nn tn ti b 2 Z sao cho b 2  m .mod p/.


Ln ny, ta xt li nguyn L D .x; y; z/ 2 Z3 j x  y .mod m/; y  bz .mod q/ v hnh
ellipsoid E: qX 2 C Y 2 C mZ 2  2mq.
Sau mt vi bc tng t, ta thu c: tn ti vec-t khc khng .x; y; z/ 2 L \ E: qx 2 C y 2 C
mz 2 D q m. Vic chng minh rng mi s nguyn t p tha mn p > 2 v p .y 2 C mz 2 / D
y 2 C mz 2
2n C 1 u ng d 1 mod 4 l tng t. Cui cng, ta vn c th biu din
di
q
dng tng ca hai s chnh phng v kt thc bi ton.
Trong c hai trng hp, m u c th biu din c di dng tng ca ba s chnh phng.
nh l 4.1 c gii quyt trit .
Qua chuyn ny, ta khng ch bit n=4 l s nguyn nh nht tha mn mi s nguyn dng
u c th biu din c bng tng ca n s chnh phng m cn bit c nhngY
s noY
c th
r
si
biu din c di dng tng ca hai hoc ba s chnh phng. C th, n D 2
pi
qit i
vi pi  1 .mod 4/ v qi  3 .mod 4/ c th biu din c bng tng ca hai s chnh
phng khi v ch khi ti chn vi mi i ; m c th biu din c bng tng ca ba s chnh
phng khi v ch khi m khng c dng 4a .8n C 7/.

5. Bi ton Waring
Vo th k 18, Waring, mt nh ton hc li lc ngi Anh a ra nhn xt rng mi s nguyn
dng c th biu din bi tng ca 9 lp phng ng v tng ca 19 ly tha bc 4. ng m
rng gi thuyt ca mnh: Vi k l s nguyn dng cho trc, lun tn ti m (ph thuc vo k)
sao cho mi s nguyn dng c th biu din c bng tng ca m ly tha bc k. y chnh
l bi ton Waring tng qut.
Vo nm 1906, David Hilbert, mt nh ton hc ni ting ngi c chng minh thnh cng
gi thuyt trn nhng li gii v cng phc tp.
Gi g.k/ l s m nh nht, c ngha mi s nguyn dng u c th biu din di dng tng
ca g.k/ ly tha bc k v tn ti t nht mt s nguyn dng khng th biu din di dng
tng ca (g.k/ 1) ly tha bc k. Trong chuyn , ta thnh cng trong vic chng minh
g.2/ D 4. Gn y, ngi ta
minh c g.3/ D 9; g.4/ D 19; g.5/ D 37 v nu
 chng

3 k
k  471600000 th g.k/ D . / C 2k 2.
2
Vn cn kh nhiu cu hi m xung quanh hm g.k/ rt ng c quan tm v khm ph. C
th xem [9] bit thm chi tit v bi ton Waring.
131

Tp ch Epsilon, S 08, 04/2016

Ti liu tham kho


[1] Wikipedia, Albert Girard. https://en.wikipedia.org/wiki/Albert Girard.
[2] Wikipedia, Pierre de Fermat. https://en.wikipedia.org/wiki/Pierre de Fermat.
[3] PETE
L.
Clark,
Dirichlets
Theorem
on
Primes
Progressions.
Department
of
Mathematics
-University
http://math.uga.edu/ pete/4400DT.pdf.

in
Arithmetic
of
Georgia.

[4] H. Davenport, the geometry of number, Mathematical Gazette vol 31 (1947) (206-210)
[5] Titu Andreescu and Dorin Andrica, 2009. Number Theory: Structures, Examples, and
Problems (179-188).
[6] Titu Andreescu and Gabriel Dospinescu, 2008. Problem from the Books (49-67)
[7] Micheal Wong. Representing integers as sum of squares. University of Chicago: Department
of Mathematics.
[8] N. c. Ankeny, 1957. Sums of three squares. Proceedings of the AMS 8 (316-319)
[9] Hardy, Wright, 1954. An Introduction to the Theory of Numbers. Oxford.
[10] Phan Huy Khai, 2004. Cac bai toan co ban cua so hoc (Number theorys elementary
problems) (255-282).

132

MT S DNG TON
V BT PHNG TRNH HM
Trnh o Chin
(Trng Cao ng S Phm Gia Lai)
Cc bi ton v gii bt phng trnh hm thng l nhng bi ton kh. Trong nhng nm gn
y, cc dng ton loi ny i khi xut hin trong cc thi chn hc sinh gii cc cp v
Olympic Ton quc t. Chng hn Bi ton 3, trong IMO 2011:
Gi s f W R ! R l mt hm gi tr thc xc nh trn tp cc s thc v tha mn
f .x C y/  yf .x/ C f .f .x//
vi mi s thc x v y: Chng minh rng f .x/ D 0 vi mi x  0:
Bi vit ny cp n phng php gii mt lp cc bt phng trnh hm dng c bn. y
l mt trong nhng phng php c th tham kho tm ti li gii cho mt bi ton v bt
phng trnh hm.

1. Bt phng trnh hm vi cp bin t do


Xt hm bin s thc f tha mn cc tnh cht sau
f .x C y/  f .x/f .y/:
Ta c th tm c hm f tha mn tnh cht trn nu f tha mn thm mt s iu kin ban
u no , chng hn (xem [1])
f .x/  ax ;

a > 0:

gii bi ton trn, trc ht ta cn gii cc bi ton sau


Bi ton 1. Xc nh cc hm s f .x/ tha mn ng thi cc iu kin sau
i) f .x C y/  f .x/ C f .y/ vi mi x; y 2 RI
ii) f .x/  0 vi mi x 2 R:
Chng minh. T cc iu kin ca bi ton, thay x D 0 ta thu c f .0/  2f .0/ v f .0/  0.
Do f .0/ D 0. Vy nn
0 D f .0/ D f .x C . x//  f .x/ C f . x/  0:
Suy ra f .x/  0: Th li, ta thy hm s f .x/  0 tha mn iu kin bi ra.
133

Tp ch Epsilon, S 08, 04/2016


Bi ton 2. Cho trc a 2 R. Xc nh cc hm s f .x/ tha mn ng thi cc iu kin sau
i) f .x C y/  f .x/ C f .y/ vi mi x; y 2 R;
ii) f .x/  ax vi mi x 2 R.
Chng minh. Xt hm s g.x/ D ax. rng g.x C y/ D g.x/ C g.y/. t f .x/ D
g.x/ C h.x/. Khi , ta thu c cc iu kin
i) h.x C y/  h.x/ C h.y/ vi mi x; y 2 RI
ii) h.x/  0 vi mi x 2 R:
Theo Bi ton 1, ta c h.x/  0 hay f .x/ D ax. Th li, ta thy hm s f .x/ D ax tha mn
iu kin bi ra.
By gi, ta tr li bi ton nu ban u.
Bi ton 3. Cho trc a > 0. Xc nh cc hm s f .x/ tha mn ng thi cc iu kin sau
i) f .x C y/  f .x/f .y/ vi mi x; y 2 R;
ii) f .x/  ax vi mi x 2 R.
Chng minh. Nhn xt rng f .x/ > 0 vi mi x 2 R. Vy ta c th logarit ha hai v cc bt
ng thc ca iu kin cho
i) ln f .x C y/  ln f .x/ C ln f .y/ vi mi x; y 2 R;
ii) ln f .x/  .ln a/x vi mi x 2 R.
t ln f .x/ D '.x/, ta thu c
i) '.x C y/  '.x/ C '.y/ vi mi x; y 2 R;
ii) '.x/  .ln a/x vi mi x 2 R.
Ta nhn c dng ca Bi ton 2. Vy '.x/ D .ln a/x. Suy ra f .x/ D ax . Th li, ta thy hm
s f .x/ D ax tha mn iu kin bi ra.
Nhn xt rng, cc bi ton trn vn gii c nu tp xc nh R ca cc hm s trn c thay
bi mt khong m U cha 0 sao cho vi mi x; y 2 U th x C y 2 U .
Mt cu hi t nhin c t ra: Trong Bi ton 3, c th thay hm s g.x/ D ax bi hm s
no bi ton cng c nghim khng tm thng?
Nhn xt rng
 Vi 0 < a < 1 th ax > 1 C x, 8x < 0 v ax  1 C x, 8x  0;
 Vi a  1 th ax > 1 C x, 8x < 0; ax  1 C x, 8x 2 0; 1/ v ax  1 C x, 8x  1.
T , mt cch t nhin, tip theo ta xt hm s g.x/ D x C 1. Ta c bi ton sau
134

Tp ch Epsilon, S 08, 04/2016


Bi ton 4. Gi s U l khong m cha 0 sao cho vi mi x; y 2 U th x C y 2 U . Xc nh
cc hm s f W U ! R tha mn ng thi cc iu kin sau
i) f .x C y/  f .x/f .y/ vi mi x; y 2 U ;
ii) f .x/  1 C x vi mi x 2 U .
Chng minh. Bi i), ta c
f .x/ D f

x
2

x 
x
f2
 0;
2
2

Nu f .x0 / D 0, th

8x 2 U:

 
x0 
2 x0
C
f
:
0 D f .x0 / D f
2
2
x 
x  2
0
0
D 0: Quy np, ta c f n D 0 vi mi s nguyn dng n. Tuy nhin, t ii)
Do f
2
2
suy ra rng rng f .x/ > 0 vi mi x 2 U v x gn 0. Do iu trn l mu thun. Vy
x

8x 2 U:

f .x/ > 0;

Tip theo, t i) v ii), ta s thy rng f kh vi ti mi im x 2 U v f 0 .x/ D f .x/. Tht vy,


t i) v ii), vi h > 0 nh, ta c
f .x C h/

f .x/  f .x/f .h/

f .x/ D f .h/

1f .x/  hf .x/:

Do

f .x C h/ f .x/
 f .x/:
h
Mt khc, cng t i) v ii), vi h > 0 nh, ta c
f .x/ D f .x C h

h/  f .x C h/f . h/  .1

h/f .x C h/:

Suy ra
.1

h/f .x/ C hf .x/  .1

h/f .x C h/:

Do
hf .x/  .1

h/f .x C h/

f .x/;

hay
f .x C h/
h

f .x/

f .x/
:
1 h

Vy, vi h > 0 nh, ta c


f .x/ 

f .x C h/
h

f .x/

f .x/
:
1 h

Tng t, bt ng thc trn cng ng i vi chiu ngc li, vi h < 0 nh. Do , ta c


f .x C h/ f .x/
f 0 .x/ D lim
tn ti v bng f .x/, vi mi x 2 U . T , vi mi x 2 U ,
h!0
h
ta c


f 0 .x/ f .x/
f .x/ 0
D
D 0:
ex
ex
Do f .x/ D C e x (C l hng s). Hn na, t i) ta c f .0/  f 2 .0/ hay f .0/  1 v t ii)
ta c f .0/  1. Do C D f .0/ D 1. Th li, hm f .x/ D e x tha mn cc yu cu.
135

Tp ch Epsilon, S 08, 04/2016


Nh vy, vi g.x/ D ax hoc g.x/ D 1 C x, Bi ton 3 v Bi ton 4 u gii c. Mt cu
hi tip theo c t ra: Vi nhng lp hm g.x/ no th bi ton tng qut l gii c?
Ta c kt qu sau
nh l 1. Gi s U l khong m cha 0 sao cho vi mi x; y 2 U th x C y 2 U . Nu hm
s f W U ! R tha mn ng thi cc iu kin sau
i) f .x C y/  f .x/f .y/ vi mi x; y 2 U ;
ii) f .x/  g.x/, 8x 2 U ; trong g.x/ l hm s cho trc kh vi ti 0; g.0/ D 1,
g 0 .0/ D k, th f .x/ D e kx .
Chng minh. Tng t li gii Bi ton 4, t cc iu kin cho, ta suy ra f .x/ > 0 vi mi
x 2 U . Gi s rng f .x/ l hm s tha mn cc iu kin ca nh l.
Th th, vi h > 0 nh, ta c
f .x C h/

f .x/  f .x/f .h/

f .x/ D .f .h/

1/f .x/  .g.h/

1/f .x/:

Do

f .x C h/ f .x/
g.h/ g.0/

f .x/:
h
h
Mt khc, cng t i) v ii), vi h > 0 nh, ta c
f .x/ D f .x C h

h/  f .x C h/f . h/  f .x C h/g. h/:

V hm g.x/ kh vi ti 0 nn n lin tc ti im . Do , vi h > 0 nh, ta c g. h/ > 0.


Khi , vi h > 0 nh, ta c
f .x C h/

f .x/ 

g. h/ 1
g. h/ g.0/
f .x/ D
f .x/:
g. h/
hg. h/

Vy vi h > 0 nh, t cc kt qu trn, ta c


g.h/

g.0/
h

f .x/ 

f .x C h/
h

f .x/

g. h/ g.0/
f .x/:
g. h/

Tng t, bt ng thc trn cng ng i vi chiu ngc li, vi h < 0 nh. Do , ta c


f .x C h/ f .x/
f 0 .x/ D lim
tn ti v bng g 0 .0/f .x/ D kf .x/, vi x 2 U .
h!0
h
T , vi x 2 U , ta c


f .x/ 0
kf .x/ kf .x/
f 0 .x/ kf .x/
D
D
D 0:
kx
kx
e
e
e kx
Do f .x/ D C e kx (C l hng s). Hn na, t i) ta c f .0/  f 2 .0/ hay f .0/  1 v t ii)
ta c f .0/  1. Do C D f .0/ D 1. Vy f .x/ D e kx . Ta c iu phi chng minh.
136

Tp ch Epsilon, S 08, 04/2016


R rng f .x/ D e kx tha mn iu kin i). Nu gi thit bi ton c thm iu kin g.x/  e kx ,
vi x 2 U , th hm s f .x/ D e kx tha mn tt c cc iu kin ca bi ton.
T kt qu trn, ta c
H qu 2. Gi s U l khong m cha 0 v f W U ! R tha mn iu kin i) vi mi x; y 2 U
sao cho x C y 2 U . Nu f kh vi ti 0; f .0/ D 1 v f 0 .0/ D k th f .x/ D e kx , x 2 U .
Chng minh. p dng nh l 1, vi g.x/ D f .x/, 8x 2 U , ta c iu phi chng minh.
H qu 3. Gi s F l hm xc nh trn khong m U cha 0 v tha mn
F .x C y/  F .x/ C F .y/
vi mi x; y 2 U sao cho x C y 2 U . Nu F b chn trn bi mt hm G kh vi ti 0 v tha
mn G.0/ D 1, th F .x/ D kx, x 2 U , trong k l mt hng s.
Chng minh. p dng nh l 1, vi f .x/ D e

F .x/

v g.x/ D e

G.x/

; ta c pcm.

Tng t phng php chng minh nh l 1, ta c kt qu sau y


nh l 4. Gi s U l khong m cha 0 sao cho vi mi x; y 2 U th x C y 2 U . Nu hm
s f W U ! R tha mn iu kin sau
f .x C y/  f .x/g.y/;

8x; y 2 U;

trong g.x/ l hm s cho trc kh vi ti 0, g.0/ D 1; g 0 .0/ D k; th mi nghim ca bt


phng trnh hm trn u c dng f .x/ D C e kx ; C l hng s.
H qu 5. Ta c f .x/ D e kx v g.x/ D e kx l nghim duy nht ca h bt phng trnh hm
(
f .x C y/  f .x/g.y/
g.x C y/  g.x/f .y/
vi iu kin f .0/ D 1, g.x/ l kh vi ti 0, g.0/ D 1 v g 0 .0/ D k.
Chng minh. T bt phng trnh hm th nht, p dng nh l 2 ta c f .x/ D C e kx (C l
hng s). V f .0/ D 1, nn C D 1. Do
f .x/ D e kx :
Tng t, t bt phng trnh hm th hai, p dng nh l 2 ta cng c
g.x/ D e kx :
R rng f .x/ D e kx v g.x/ D e kx tha mn h bt phng trnh hm cho, vi nhng iu
kin nu. H qu c chng minh.
137

Tp ch Epsilon, S 08, 04/2016


nh ngha 5. Hm g.x/ xc nh trn mt khong m U cha 0 c gi l hm ta bi l
ti 0 nu tn ti mt hm k.x/ xc nh trn U sao cho k.0/ D g.0/, k 0 .0/ D l tn ti v
k.x/  g.x/ vi mi x 2 U .
H qu 6. Bt phng trnh hm
f .x C y/  f .x/g.y/;
trong g l mt hm cho trc xc nh trn I vi g.0/ D 1 v l hm ta bi l ti 0; c
nghim khng m f khi v ch khi e lx  g.x/ trn I v trong trng hp ny mi nghim khng
m u c dng f .x/ D C e lx , trong C  0 l hng s.
Chng minh. Gi s f .x/ l mt nghim khng m ca bt phng trnh hm cho. V
g.x/  k.x/ trn U; nn ta c
f .x C y/  f .x/k.y/;
trong k.x/ tha mn k 0 .0/ D l v k.0/ D g.0/ D 1. p dng nh l 2 vo bt phng trnh
hm ny, ta c f .x/ D C e lx , trong C  0 l hng s. R rng, f .x/ D C e lx l mt nghim
khng m ca bt phng trnh hm cho nu e lx  g.x/ trn U:
T H qu 4, ta c th sng tc ra cc bi ton, chng hn sau y
Bi ton 5. Tm tt c cc hm s f .x/, xc nh trn khong m . e; 1/, tha mn h bt
phng trnh hm sau
(
f .x C y/  f .x/ log f .y/
f .x/  x C e
Chng minh. Trn khong m . e; 1/, mi nghim dng f c suy ra bi bt phng trnh
hm th hai. Ngoi ra, t h bt phng trnh hm cho, ta c f .0/ D e. p dng H qu 4
1
i vi trng hp g.x/ D log f .x/ l hm ta bi ti 0; qua hm k.x/ D log.x C e/. Do
e
x
x
, theo chng minh ca H qu 4 , ta c f .x/ D f .0/e e D e 1C e . Th li, ta thy hm s
x
f .x/ D e 1C e tha mn h bt phng trnh hm cho trn khong m . e; 1/.
Bi ton 6. Trn khong m cha 0 c mt nghim ca h bt phng trnh hm
(
f .x C y/  f .x/e f .y/
f .x/  x 2
Chng minh. Gi s f .x/ l mt nghim xc nh trn mt khong m cha 0 no . Th
th, bi bt phng trnh hm th hai, f .x/ l khng m. T h bt phng trnh cho suy ra
f .0/ D 0. p dng H qu 4 i vi trng hp g.x/ D e f .x/ l hm ta bi 0 ti 0, qua hm
2
k.x/ D e x . Hn na, v f .0/ D 0, nn ta c f .x/  0 tha mn bt phng trnh hm th hai
trn khong khng m cha 0.
nh l sau y cho ta kt qu v vic gii mt dng bt phng trnh hm c bn khc
138

Tp ch Epsilon, S 08, 04/2016


nh l 7. Gi s U l khong m cha 0 sao cho vi mi x; y 2 U th x C y 2 U . Xt bt
phng trnh hm
f .x C y/  f .x/g.y/ C f .y/g.x/;

8x; y 2 U;

trong g.x/ l mt hm gii ni, kh vi ti 0, g.0/ D 1 v g 0 .0/ D k. Th th f .x/  0 l


f .x/
hm s duy nht tha mn bt phng trnh cho, vi iu kin lim
D 0.
x!0 x
Chng minh. Gi s rng f .x/ l nghim ca bt phng trnh cho, vi iu kin
f .x/
D 0:
x!0 x
lim

Th th, vi h > 0 nh, ta c


f .x C h/  f .x/g.h/ C f .h/g.x/
hay
f .x C h/
Do

f .x C h/
h

f .x/  .g.h/
f .x/

g.h/

1/f .x/ C f .h/g.x/:


g.0/

f .x/ C

f .h/
g.x/:
h

Mt khc, ta c
f .x/ D f .x C h

h/  f .x C h/g. h/ C f . h/g.x C h/

hay
g. h/.f .x/

f .x C h//  g. h/f .x/

f .x/ C f . h/g.x C h/

V hm g.x/ kh vi ti 0 nn n lin tc ti im . Do , vi h > 0 nh, ta c g. h/ > 0.


Vy, vi h > 0 nh, ta c
f .x C h/
h

f .x/

1/f .x/ C f . h/g.x C h/


g. h/
g. h/ g.0/
f . h/
D
f .x/ C
g.x C h/:
hg. h/
hg. h/


.g. h/

Vy vi h > 0 nh, t cc kt qu trn, ta c


g.h/

g.0/
h

f .x/ C

f .h/
f .x C h/ f .x/
g.x/ 
h
h
g. h/ g.0/
f . h/

f .x/ C
g.x C h/:
hg. h/
hg. h/

Tng t, bt ng thc trn cng ng i vi chiu ngc li, vi h < 0 nh. Do , ta c


f .x C h/
h!0
h

f 0 .x/ D lim

139

f .x/

Tp ch Epsilon, S 08, 04/2016


f .x/
D 0 v g.x/ l mt hm gii ni.
x!0
x

tn ti v bng g 0 .0/f .x/ D kf .x/, vi x 2 U , v lim

T , vi x 2 U , ta c


f .x/ 0
f 0 .x/ kf .x/
kf .x/ kf .x/
D
D
D 0:
kx
kx
e
e
e kx

f .x/
D 0; suy ra rng C D 0. Vy
x!0
x
f .x/
D 0.
f .x/  0 l hm s duy nht tha mn yu cu, vi iu kin lim
x!0
x
Do f .x/ D C e kx (C l hng s). Hn na, t lim

2. Bt phng trnh hm dng cng nhn tnh


Phn ny cp n vic gii cc h bt phng trnh hm, vi cc dng sau y
 Dng cng: f .a C x/  C f .x/, f .b C x/  C f .x/, x 2 R;
 Dng cng - nhn: f .a C x/  f .x/, f .b C x/  f .x/, x 2 R;
 Dng nhn - cng: f .ax/  C f .x/, f .bx/  C f .x/, x 2 I , I  R;
 Dng nhn: f .ax/  f .x/, f .bx/  f .x/, x 2 I , I  R; trong a; b; ; l
cc s thc cho trc.
Ch rng, nu D f .a/, D f .b/, th h bt phng trnh hm dng cng trn l s thu
hp ca bt phng trnh hm Cauchy c in
f .x C y/  f .x/ C f .y/;

8x; y 2 R:

Trc ht, ta nhc li rng, mt tp hp M tr mt trong tp s thc R nu nh trong mi ln


cn ca mt im ty ca tp R u c t nht mt im ca tp M . Chng hn, tp Q cc s
hu t l tp tr mt trong tp R.
Tnh cht sau y l mt kt qu quen thuc (nh l Kronecker), c th tm thy chng minh
cc ti liu l thuyt c bn: Nu a v b l cc s thc khng thng c vi nhau, th tp
A D fma C nb j m; n 2 Zg tr mt trong R.
Hn na, ta c th chng minh c cc kt qu sau y
B 8. Gi s a; b 2 R v a < 0 < b l cc s cho trc. K hiu
A D fma C nb j m; n 2 Ng:
1) Nu

b
Q, th tp A tr mt trong R.
a

2) Nu

b
2 Q, th tn ti d > 0 sao cho A D fkd j k 2 Zg.
a

Dng nhn ca b ny nh sau


140

Tp ch Epsilon, S 08, 04/2016


B 9. Gi s a; b 2 R v 0 < a < 1 < b l cc s cho trc. K hiu
M D fam b n j m; n 2 Ng:
log b
Q, th tp M tr mt trong .0; 1/.
log a
n
o
log b
k
2 Q, th tn ti d > 0 sao cho M D d j k 2 Z .
2) Nu
log a
1) Nu

By gi, ta chng minh cc nh l sau y


nh l 10 (Dng cng). Gi s a; b; ; l cc s thc cho trc tha mn a < 0 < b,

D , v gi s rng hm f W R ! R lin tc ti t nht mt im.


a
b
1) Nu

b
Q, th f tha mn h bt ng thc hm
a
f .a C x/  C f .x/;

f .b C x/  C f .x/;

khi v ch khi f .x/ D px C f .0/, x 2 R, trong p WD

8x 2 R

.1/

.
a

b
2) Nu 2 Q, th tn ti duy nht mt nghim hm lin tc f W R ! R ca h phng trnh
a
hm tng ng
f .a C x/ D C f .x/;

f .b C x/ D C f .x/;

8x 2 R

.2/

sao cho f 0;d D f0 , trong d WD minfma C nb > 0 j m; n 2 Ng tn ti, l s dng


v f0 W 0; d ! R l hm lin tc cho trc tha mn iu kin
f0 .d / D

d C f0 .0/:
a

Hn na, nu f0 l n iu nghim ngt, th n trng vi hm f trn on 0; d .


Chng minh. 1) T (1), d dng suy ra
f .ma C x/  m C f .x/;

f .nb C x/  n C f .x/;

8m; n 2 N; x 2 R:

Trong bt ng thc u tin trn, thay x bi nb C x, ta c


f .ma C nb C x/  m C f .nb C x/  m C n C f .x/:
Do
f .ma C nb C x/  m C n C f .x/;
t p WD

8m; n 2 N; x 2 R:

D , ta c th vit bt ng thc ny di dng


a
b
f .t C x/  pt C f .x/;
141

8t 2 A; x 2 R;

.3/

Tp ch Epsilon, S 08, 04/2016


trong , theo B 1, tp A D fma C nb j m; n 2 Ng tr mt trong R.
Gi s rng x0 l im m ti hm f lin tc v x l mt gi tr thc ty . Bi tnh cht tr
mt ca A trong tp R, tn ti mt dy .tn / sao cho
tn 2 A .n 2 N/;

lim tn D x0

x!C1

x:

T bt ng thc (3), ta c
f .tn C x/  ptn C f .x/;

8n 2 N:

Cho n ! 1, bi tnh lin tc ca hm f ti x0 , ta thu c


f .x0 /  p.x0

x/ C f .x/;

By gi, chng minh phn o, thay x bi x


f .x/  pt C f .x

8x 2 R:

t trong (3), ta dc
8t 2 A; x 2 R:

t /;

Chn mt im x 2 R c nh ty v, bi tnh tr mt ca A trong R, mt dy .tn / sao cho


tn 2 A .n 2 N/;

lim tn D x

n!C1

x0 :

Th th, ta c
f .x/  ptn C f .x

8n 2 N:

tn /;

Cho n ! 1, bi tnh lin tc ca hm f ti x0 , ta thu c bt ng thc


f .x/  p.x

x0 / C f .x0 /;

8x 2 R:

Do , ta c
f .x/ D p.x/ C .f .x0 /

px0 /;

8x 2 R:

Ta c iu phi chng minh.


2) T (2), d dng suy ra
f .ma C nb C x/ D

.ma C nb/ C f .x/;


a

8m; n 2 N; x 2 R:

.4/

Theo B 1, phn 2, s d WD minfma C nb > 0 j m; n 2 Ng l xc nh v l s dng. Hn


na fma C nb > 0 j m; n 2 N g D fkd j k 2 Zg. D , (4) c dng
f .kd C x/ D

kd C f .x/;
a

8k 2 Z; x 2 R:

D dng thy rng h phng trnh hm ny tng ng vi phng trnh


f .d C x/ D

d C f .x/;
a
142

8x 2 R:

Tp ch Epsilon, S 08, 04/2016


By gi, ta xc nh f1 W .d; 2d ! R bi cng thc

f1 .x/ WD d C f0 .x d /;
a

x 2 .d; 2d :

Gi s rng fn W .nd; .n C 1/d ! R .n 2 N/ c xc nh. Th th, ta xc nh


fnC1 W ..n C 1/d; .n C 2/d ! R
bi h thc truy hi
fnC1 .x/ WD

d C fn .x
a

Tng t, gi s
f 1 .x/ WD

d /;

x 2 ..n C 1/d; .n C 2/d ; n 2 N:

d C f0 .x C d /;
a

x 2 d; 0/:

Gi s rng ta c nh ngha
f n .x/ WD nd; . n C 1/d / ! R .n 2 N /:
Th th, ta nh ngha
f

.nC1/ .x/

WD

d C f n .x C d /;
a

x 2 .n C 1/d;

nd /; n 2 N:

D dng kim tra c rng f W R ! R, xc nh bi


8

< f n .x/ khi x 2 nd; . n C 1/d /


f .x/ D f0 .x/ khi x 2 0; d
.n 2 N/;

: f .x/ khi x 2 .nd; .n C 1/d


n

tha mn h (2), l hm lin tc v f 0; d D f0 . nh l c chng minh hon ton.


nh l 11 (Dng cng nhn). Gi s a; b 2 R v ; > 0 l cc s cho trc tha mn
log
log
a < 0 < b,
D
v gi s rng hm f W R ! R lin tc ti t nht mt im.
a
b
b
1) Nu Q, th f tha mn h bt ng thc hm
a
f .a C x/  f .x/; f .b C x/  f .x/; x 2 R
.5/
log
.
a

khi v ch khi f .x/ D f .0/e px , x 2 R, trong p WD

b
2) Nu 2 Q, th tn ti duy nht mt nghim hm lin tc f W R ! R ca h phng trnh
a
hm tng ng
f .a C x/ D f .x/; f .b C x/ D f .x/; x 2 R
.6/

sao cho f 0; d D f0 , trong d WD minfma C nb > 0 j m; n 2 Ng tn ti, l s dng


v f0 W 0; d ! R l hm lin tc cho trc tha mn iu kin
f0 .d / D f0 .0/e

log
a d

Hn na, nu f0 l n iu nghim ngt, th n trng vi hm f trn on 0; d .


143

Tp ch Epsilon, S 08, 04/2016


Chng minh. 1) T (5), d dng suy ra
f .ma C x/  m f .x/;

f .nb C x/  n f .x/;

8m; n 2 N; x 2 R:

Trong bt ng thc u tin trn, thay x bi nb C x, ta c


f .ma C nb C x/  m f .nb C x/  m n f .x/:
Do
f .ma C nb C x/  m n f .x/; 8m; n 2 N; x 2 R:
log
log
D
, ta c th vit bt ng thc ny di dng
t p WD
a
b
f .t C x/  e pt f .x/;

t 2 A; x 2 R

.7/

trong , theo B 1, tp A D fma C nb j m; n 2 Ng tr mt trong R.


Gi s rng x0 l im m ti hm f lin tc v x l mt gi tr thc ty . Bi tnh cht tr
mt ca A trong tp R, tn ti mt dy .tn / sao cho
tn 2 A .n 2 N/;

lim tn D x0

x:

n!C1

T bt ng thc (7), ta c
f .tn C x/  e ptn f .x/;

n 2 N:

Cho n ! 1, bi tnh lin tc ca hm f ti x0 , ta thu c


f .x0 /  e p.x0

x/

By gi, chng minh phn o, thay x bi x


f .x/  e pt f .x

x 2 R:

f .x/;

t trong (7), ta dc
t /;

t 2 A; x 2 R:

Chn mt im x 2 R c nh ty v, bi tnh tr mt ca A trong R, mt dy .tn / sao cho


tn 2 A .n 2 N/;

lim tn D x

n!C1

x0 :

Th th, ta c
f .x/  e ptn f .x

tn /;

n 2 N:

Cho n ! 1, bi tnh lin tc ca hm f ti x0 , ta thu c bt ng thc


f .x/  e p.x

x0 /

f .x0 /;

x 2 R:

Do , ta c
f .x/ D e p.x/ .f .x0 /

px0 /;

x 2 R:

Ta c iu phi chng minh.


2) Chng minh tng t phn 2 ca nh l 4. nh l c chng minh hon ton.
144

Tp ch Epsilon, S 08, 04/2016


nh l 12 (Dng nhn cng). Gi s a; b; ; l cc s thc cho trc tha mn

D
;
log a
log b

0 < a < 1 < b;

v gi s rng hm f W I ! R lin tc ti t nht mt im.


1) Nu

log b
Q, th f tha mn h bt ng thc hm
log a
f .ax/  C f .x/;

f .bx/  C f .x/;

x 2 I;

.8/

th
i) Trng hp I D .0; 1/: f .x/ D p log x C f .1/, x > 0,
ii) Trng hp I D . 1; 0/: f .x/ D p log. x/ C f . 1/, x < 0, trong p WD

.
log a
log b
2 Q, th tn ti duy nht mt nghim hm lin tc f W I ! R ca h phng
log a
trnh hm tng ng

2) Nu

f .ax/ D C f .x/;

f .bx/ D C f .x/;

x2I

.9/

sao cho f 1; d D f0 , trong d WD minfam b n > 1 j m; n 2 N g tn ti, ln hn 1 v


f0 W 1; d ! R l hm lin tc cho trc tha mn iu kin
f0 .d / D

log d C f0 .1/:
log a

Hn na, nu f0 l n iu nghim ngt, th n trng vi hm f trn on 1; d .


Chng minh. 1) i) Gi s I D .0; 1/. T (8), chng minh tng t nh cc phn trn, ta c
f .am b n x/  m C n C f .x/;
t p WD

m; n 2 N; x > 0:

D
, ta c th vit bt ng thc ny di dng
log a
log b
f .am b n x/  p log.am b n / C f .x/;

m; n 2 N; x > 0;

hay
f .tx/  p log t C f .x/;

t 2 M; x > 0;

.10/

trong , theo B 2, tp M D fam b n j m; n 2 Ng tr mt trong I .


Gi s rng x0 > 0 l im m ti hm f lin tc v x > 0 l mt gi tr ty . Bi tnh cht
tr mt ca M trong tp I , tn ti mt dy .tn / sao cho
tn 2 M .n 2 N/;
145

lim tn D

n!C1

x0
:
x

Tp ch Epsilon, S 08, 04/2016


T bt ng thc (10), ta c
f .tn x/  p log tn C f .x/;

n 2 N:

Cho n ! 1, bi tnh lin tc ca hm f ti x0 , ta thu c


f .x0 /  p log

x0
C f .x/;
x

By gi, chng minh phn o, thay x bi

x
trong (10), v chn mt dy .tn / sao cho
t

tn 2 M .n 2 N/;
Th th, ta c
f .x/  p log

x > 0:

lim tn D

n!C1

x
C f .x0 /;
x0

x
:
x0

x > 0:

Do , ta c
f .x/ D f .x0 /

p log x0 C p log x;

x > 0:

Phn i) c chng minh.


ii) Gi s rng I D . 1; 0/. Ta xt hm g W .0; 1/ ! R xc nh bi cng thc
g.x/ D f . x/;

x < 0;

tha mn h (8) v chng minh tng t nh chng minh phn i).


2) Phn ny chng minh tng t nh chng minh nh l 4, phn 2.
H qu
h 13. Gi
[s a; b;i; 2 R tha mn cc gi thit ca nh l 4, phn 1. Nu hm
f W . 1; 0/ .0; 1/ ! R tha mn h bt ng thc .8/ v trong mi khong .1; 0/,
.0; 1/ tn ti t nht mt im m ti hm f lin tc, th
(
p log x C f .1/;
khi x 2 .0; 1/
f .x/ D
p log. x/ C f . 1/; khi x 2 . 1; 0/
trong p WD

.
log a

Ch 1. Gi s a; b; ; l cc s thc cho trc tha mn 0 < a < 1 < b v

D
.
log a
log b

Nu 0 2 I , th khng tn ti hm no tha mn h .8/:


Tht vy, trong (8) nu t x D 0, th 0  , 0  , mu thun vi gi thit < 0.
nh l 14 (Dng nhn). Gi s a; b; ; l cc s thc cho trc tha mn a < 1 < b,
log
log
D
, v gi s rng hm f W I ! R lin tc ti t nht mt im.
log a
log b
146

Tp ch Epsilon, S 08, 04/2016


1) Nu

log b
Q, th f tha mn h bt ng thc hm
log a
f .ax/  f .x/;

f .bx/  f .x/;

x 2 I;

.11/

th
i) Trng hp I D .0; 1/: f .x/ D f .1/x p , x > 0,
ii) Trng hp I D . 1; 0/: f .x/ D f . 1/. x/p , x < 0, trong p WD

log
.
log a

log b
2 Q, th tn ti duy nht mt nghim hm lin tc f W I ! R .I D .0; 1/
log a
hoc I D . 1; 0// ca h phng trnh hm tng ng

2) Nu

f .ax/ D f .x/;

f .bx/ D f .x/;

x 2 I;

.12/

sao cho f 1; d D f0 , trong d WD minfam b n > 1 j m; n 2 Ng tn ti, ln hn 1 v


f0 W 1; d ! R l hm lin tc cho trc tha mn iu kin
log

f0 .d / D f0 .1/d log a :
Hn na, nu f0 l n iu nghim ngt, th n trng vi hm f trn on 1; d .
Chng minh. 1) i) Gi s I D .0; 1/. T (11), chng minh tng t nh cc phn trn, ta c
f .am b n x/  m n f .x/;
t p WD

m; n 2 N; x > 0:

log
log
D
, ta c th vit bt ng thc ny di dng
log a
log b
f .am b n x/  .am b n /p f .x/;

m; n 2 N; x > 0;

hay
f .tx/  t p f .x/;

t 2 M; x > 0;

.13/

trong , theo B 2, tp M D fam b n j m; n 2 Ng tr mt trong I .


Gi s rng x0 > 0 l im m ti hm f lin tc v x > 0 l mt gi tr ty . Bi tnh cht
tr mt ca M trong tp I , tn ti mt dy .tn / sao cho
tn 2 M .n 2 N/;

lim tn D

n!C1

x0
:
x

T bt ng thc (13), ta c
f .tn x/  tn p f .x/;

n 2 N:

Cho n ! 1, bi tnh lin tc ca hm f ti x0 , ta thu c


 x p
0
f .x0 / 
f .x/; x > 0:
x
147

Tp ch Epsilon, S 08, 04/2016


By gi, chng minh phn o, thay x bi

x
trong (13), v chn mt dy .tn / sao cho
t

tn 2 M .n 2 N/;
Th th, ta c

f .x/ 

x
x0

lim tn D

n!C1

x
:
x0

p
f .x0 /;

x > 0:

Phn i) c chng minh.


ii) Gi s rng I D . 1; 0/. Ta xt hm g W .0; 1/ ! R xc nh bi cng thc
g.x/ D f . x/;

x < 0;

tha mn h (11) v chng minh tng t nh chng minh phn i).


2) Phn ny chng minh tng t nh chng minh nh l 4, phn 2.
log
log
D
.
log a
log b
Nu I D R hoc I D 0; 1/ hoc I D . 1; 0 v f W I ! R tha mn h .11/; th
Ch 2. Gi s a; b; ; l cc s thc cho trc tha mn 0 < a < 1 < b v

f .0/ D 0:
Tht vy, bi mt trong hai gi thit < 1 < hoc < 1 < v, hn na, f .0/.1
v f .0/.1 /  0, ta suy ra f .0/ D 0. T Ch ny, ta c

/  0

Ch 3.

i) Gi s f W 0; 1/ ! R tha mn h .11/: Nu f .0; 1/ v a; b; ; tha mn tt c


cc gi thit ca nh l 7, phn 1, th
(
f .1/x p khi x 2 .0; 1/;
f .x/ D
0
khi x D 0;
trong p WD

log
.
log a

ii) Gi s f W . 1; 0 ! R tha mn h .11/: Nu f . 1; 0/ v a; b; ; tha mn tt c


cc gi thit ca nh l 7, phn 1, th
(
f . 1/. x/p khi x 2 . 1; 0/;
f .x/ D
0
khi x D 0;
trong p WD

log
.
log a

H qu 15. Gi s a; b; ; 2 R tha mn cc gi thit ca nh l 7, phn 1.


148

Tp ch Epsilon, S 08, 04/2016


h
i
[
i) Nu hm f W . 1; 0/ .0; 1/ ! R tha mn h bt ng thc .11/ v trong mi
khong . 1; 0/, .0; 1/ tn ti t nht mt im m ti hm f lin tc, th
(
f .1/x p
khi x 2 .0; 1/
f .x/ D
p
f . 1/. x/ khi x 2 . 1; 0/
trong p WD

log
.
log a

ii) Nu hm f W R ! R tha mn h bt ng thc .11/ v trong mi khong . 1; 0/ v


.0; 1/ tn ti t nht mt im m ti hm f lin tc, th
8
p

khi x 2 .0; 1/;

< f .1/x
f .x/ D 0
khi x D 0;

: f . 1/. x/p khi x 2 . 1; 0/


trong p WD

log
.
log a

Ch 4. Ta lun c cc nh l tng t nh cc nh l 4 - nh l 7, vi hm f tha mn cc


bt ng thc c du ngc li.

Ti liu tham kho


[1] Nguyn Vn Mu, Bt ng thc, nh l v p dng, Nh xut bn Gio dc, 2006.
[2] Trnh o Chin, Mt s dng bt phng trnh hm dng c bn, K yu Hi ngh khoa
hc v cc chuyn chuyn Ton bi dng hc sinh gii Trung hc ph thng, H Ni Nam nh, 26-28/11/2010.
[3] Th. M. Rassias, Functional equations, inequalities and applications, 73 - 89, Kluwer Academic Publishers, 2003.
[4] PI. Kannappan, Functional equations and with applications, 617 - 636, Springer Monographs
in Mathematics, 2009.

149

Tp ch Epsilon, S 08, 04/2016

150

BI TON HAY LI GII P


Trn Nam Dng - i hc Khoa hc T nhin - HQG TP.HCM

LI GII THIU
Chuyn mc ny c ly cm hng t bi vit ca thy Nguyn Duy Lin v bi
ton s 6 trong k thi IMO 2001 vi 5 cch gii khc nhau. Mc ny s dnh vit
v nhng bi ton hay, li gii p v nhng cu chuyn th v xung quanh nhng
bi ton v li gii .
Tn ca chuyn mc c mn t tn ca mt nhm nhng ngi yu ton trn
Facebook do anh Nguyn Vn Li sng lp Bi ton hay Li gii p am m
ton hc. Chuyn mc ghi nhn cc c ca bn c v s chn ng mi k 1; 2
bi ton.

Nh chng ta u bit, Euler gii thiu mt cng thc ni ting trong tam gic, th hin mi
lin h gia bn knh ca ng trn ngoi tip v ni tip cng khong cch gia hai tm ng
trn ny trong mt tam gic. l
d 2 D R2

2Rr

vi .O; R/; .I; r/ ln lt l ng trn ngoi tip, ni tip ca mt tam gic no v d D OI:

Cn Nicolaus Fuss, hc tr v cng l bn ca L.Euler gii thiu mt cng thc khc, tng
t nhng hnh thc p khng km trong t gic lng tm, tc l t gic va ni tip c trong
ng trn ny v ngoi tip c mt ng trn khc.

Bi ton 1. Trong mt t gic ABCD ngoi tip ng trn .I; r/ v ni tip c ng trn
.O; R/ th
1
.R

d/

1
2

.R C d /

1
:
r2

ng cng tm ra c cc cng thc cho ng gic, lc gic, bt gic lng tm,... Di y l


chng minh cho nh l ny.

151

Tp ch Epsilon, S 08, 04/2016

Gi K; L l tip im ca .I; r/ ln AB; BC: Do t gic ABCD ni tip nn A; C l hai gc


b nhau, suy ra
BAI C ICB D 90 :
Ta cng ch rng IK D IL D r nn hai tam gic AIK v CIL c th ghp li thnh mt tam
gic vung c hai cnh gc vung l IC; IA nh hnh v.
T , ta thy tng din tch ca hai tam gic ny c th tnh theo hai cch:
r.AK C CL/ D AI  CI:
Ngoi ra, theo nh l Pythagorean trong tam gic mi trn th:
.AK C CL/2 D AI 2 C C I 2 :
T ta thy
1
1
1
D
C
:
2
2
r
AI
CI2
Gi s AI; CI ct ng trn .O; R/ theo th t ti F; E: Khi EF l ng knh ca .O; R/
v E; F l trung im ca hai cung chung dy BD.
r 2 .AI 2 C C I 2 / D AI 2  C I 2 ,

Theo cng thc ng trung tuyn trong tam gic EF I th


EI 2 C F I 2 D 2I O 2 C

EF 2
D 2.d 2 C R2 /:
2

Theo tnh cht phng tch ca I vi .O; R/ th


AI  F I D CI  EI D R2
Do

d 2:

1
1
1
F I 2 C EI 2
2.R2 C d 2 /
D
C
D
D
r2
AI 2
CI2
.R2 d 2 /2
.R2 d 2 /2
.R C d /2 C .R d /2
1
1
D
C
:
D
2
2
.R C d /
.R d /2
.R2 d 2 /
152

Tp ch Epsilon, S 08, 04/2016


nh l c chng minh.

p
T y, ta cn c th chng minh c rng trong t gic lng tm, ta lun c R  r 2.
Tht vy,
1
1
2
1
D
C
q
2
2
2
r
.R C d /
.R d /
.R C d /2 .R

d /2

2
R2

d2

2
R2

p
nn R  r 2. ng thc xy ra khi d D 0 v ta d dng chng minh c lc , ABCD phi
l hnh vung.
nh l ny cng cho ta bit v tr ca hai ng trn .I; r/ v .O; R/ th no tn ti mt t
gic ngoi tip .I / v ni tip .O/:

153

Tp ch Epsilon, S 08, 04/2016

154

EUCLID V C S CA HNH HC
Ng Bo Chu, Richard Fitzpatrickn

Trong s ny, chuyn mc im sch ca Epsilon trn trng gii thiu vi c


gi bi im sch ca GS. Ng Bo Chu v gio s Richard Fitzpatrick cho quyn
sch va mi ra i "Euclide v C s ca hnh hc".

A. Li gii thiu ca Gio s Ng Bo Chu cho bn dch


ting Vit
Euclid vit sch C s ca hnh hc Alexandria khong 300 nm trc Cng nguyn. y l
thi k Hellenistic ca trit hc c i, thi k m trit hc c i lan to ti nhng vng t
chu nh hng ca vn ho Hy Lp m tiu biu l thnh Alexandria bn b Phi ca a Trung
Hi. Nt chung ca trit hc thi k Hellenistic, phn no th hin trong sch C s ca hnh
hc, l t duy t n mc tinh tu, nhng c l mt i tnh bay bng ca thi k trc
Socrates v sc mnh t duy ca Plato, Socrates.
Ngi ta cho rng hu ht ni dung ca sch C s ca hnh hc c truyn li t nhng tin
nhn nh Pythagoras, Plato, Eudoxus . . . Tuy nhin, khc vi Pythagoras v Plato, Euclid loi
b trit cc yu t siu hnh c gn cho cc s v cc hnh. Cc s hu t khng cn c
coi l minh chng cho s hi ho ca v tr, cc khi u trong khng gian khng cn c coi
l nim ton hc np ng sau cc phm tr siu hnh nh kim thu ho th. . . H thng suy
lun logic xut pht t h tin ca Plato c Euclid s dng mt cch trit , cc ch tiu v
tnh cht ch ca chng minh c p dng mt cch khng khoan nhng. Theo mt ngha no
,C s ca hnh hc l quyn sch thun tu ton hc u tin ca nhn loi v l t giy khai
sinh ra ton hc nh mt b mn c lp, tuy vn cn l mt b phn ca trit hc. Cch Euclid
xy dng mt h thng kin thc cao vt da trn s t tin nn v ly lut logic lm cht gn
kt, l hnh mu cho s pht trin ca ton hc cho n ngy hm nay.
Tri qua 2400 nm, cc mnh pht biu v chng minh trong C s ca hnh hc vn cn
ti tn mt cch ng ngc nhin. T hnh hc tam gic m chng ta hc nhng nm cp hai,
cho n chng minh tuyt p bng phn chng cho s tn ti v hn nhng s nguyn t, t
thut ton Euclid tm c s chung ln nht m chng ta vn phi hc trong gio trnh c s ton
hc trong tin hc, cho n chng minh khng tn ti khi u no khc ngoi nm khi u ca
Platon, u l nhng ni dung c trin khai mt cch y trong sch C s ca hnh hc.
y c l l nhng l do ti sao C s ca hnh hc c coi l mt trong nhng quyn sch c
nh hng nht ti s pht trin ca vn minh nhn loi. Sch c ti bn hng ngn ln, s
ln ti bn c l ch thua Kinh thnh. T thi k phc hng cho n u th k hai mi, sch
ca Euclid c coi l mt trong nhng quyn sch m nhng ngi c hc phi c.
155

Tp ch Epsilon, S 08, 04/2016

Sch C s ca hnh hc ca Euclid. HTN: Cun C s ca hnh hc ca Euclid ln u tin c


dch v xut bn: NXB Tri Thc v ZenBook, Qu IV nm 2015. D n dch Euclid do m Thanh Sn v
Nguyn Tr Dng khi xng thng 5 nm 2013.

Ln ln t C s ca hnh hc, Ton hc i nhng bc rt xa. By gi bn c th tm c


v s sch ton vi nhiu ni dung hn, trnh by sng sa hn sch ca Euclid. Tuy vy, ti vn
tin rng ngi c hc vn cn c Euclid vo mt thi im no trong cuc i mnh, vn cn
c C s ca hnh hc t trn gi sch.
Cm n nh xut bn Tri Thc v Nhm dch gi em sch C s ca hnh hc ca Euclid
n vi c gi Vit nam.
Ng Bo Chu

B. Li gii thiu ca Gio s Richard Fitzpatrick cho bn


ting Anh[1]
Cho n nay, C s ca hnh hc ca Euclid l tc phm ton hc kinh in tr danh nht, v
cng chim lnh v tr t bit khi l quyn sch gio khoa v ton hc c nht c s dng lin
tc. Chng ta khng bit nhiu v tc gi, ngoi chuyn ng sng Alexandria vo khong
nm 300 TCN. Cc ch chnh ca quyn sch bao gm hnh hc, t l thc v l thuyt s.
Hu ht cc nh l trong tc phm khng phi l khm ph ca c nhn Euclid, m l thnh
qu ca cc nh ton hc Hy Lp trc nh Pythagoras (v mn ca ng), Hippocrates
Chios, Theaetetus Athens v Eudoxus Cnidos. Tuy nhin, Euclid c ghi nhn cng lao v
ng sp xp cc nh l ny theo mt trt t l-gic (cho d phi th nhn rng, khng phi lc
no cng cht ch nh trong yu cu i vi ton hc hin i), th hin rng chng c dn ra
156

Tp ch Epsilon, S 08, 04/2016


t nm tin n gin. Euclid cng c nh gi cao do a ra nhng chng minh c th
ti tnh cho nhng nh l c khm ph trc : chng hn nh nh l 48 Quyn 1.
Cc php dng hnh c khai trin trong quyn sch ch gii hn trong cc thao tc thc hin
bng thc thng v compa. Hn th na, cc chng minh thc nghim da vo o c hon ton
khng c s dng: tc l bt c s so snh no gia hai i lng ch c php kt lun l
chng bng nhau hoc l i lng ny ln hn i lng kia. C s ca hnh hc gm 13 quyn.
Quyn 1 nu cc nh l cn bn ca hnh hc phng, bao gm ba nh l ca tam gic ng
dng, mt s nh l lin quan n ng thng song song, nh l v tng cc gc trong mt
tam gic, v nh l Pythagoras.
Quyn 2 thng c cho l lin quan n hnh hc gii tch, bi hu ht cc nh l trong
u c mt h qu i s n gin.
Quyn 3 nghin cu hnh trn v cc tnh cht ca chng, v bao gm cc nh l v ng tip
tuyn v gc ni tip.
Quyn 4 ni v cc a gic thng thng ni tip v ngoi tip hnh trn.
Quyn 5 xy dng l thuyt v t l thc s hc.
Quyn 6 p dng l thuyt t l thc vo hnh hc phng, v bao gm cc nh l v cc hnh
ng dng [2].
Quyn 7 lin quan n l thuyt s cn bn: v d nh s nguyn t, mu s chung ln nht v.v.
Quyn 8 ni v cp s nhn.
Quyn 9 bao gm cc ng dng ca hai quyn trc, v cc nh l v s v hn ca cc s
nguyn t.
Quyn 10 phn lp cc i lng v c (t l v t) s dng phng php gi l phng php
vt kit, l tin thn ca php tch phn.
Quyn 11 lin quan n cc nh l c bn ca hnh hc khng gian.
Quyn 12 tnh ton th tch tng i ca hnh nn, hnh chp, hnh tr v hnh cu s dng
phng php vt kit.
Cui cng, quyn 13 nghin cu nm khi a din u Platon.
n bn ny trnh by ni dung ting Hy Lp ca tc phm c J.L.Heiberg bin tp
(1883-1885) - cng vi bn dch sang ting Anh hin i, v mc lc t vng Hy Lp Anh. Cc
quyn 14 v 15 gi mo, v cc bnh gii, vn c thm vo trong nhiu th k qua s khng c
trong n bn ny. Mc ch ca bn dch l lm cho cc lun im sng sa v mch lc hn,
trong khi vn bm st ca bn gc ting Hy Lp. Cc ni dung t trong ngoc vung (c trong
phn ting Hy Lp v ting Anh) l ca Heiberg, c suy ra t vn bn gc (mt s qu r rng
hoc khng hu dng c b i). Ni dung trong ngoc n (phn ting Anh) l ni dung
ng nhng khng vit ra trong phn ting Hy Lp. Xin cm n Mariusz Wodzicki (Berkeley)
c vn sp ch, Sam Watson & Jonathan Fenno (U. Mississippi) v Gregory Wong (UCSD)
ch ra nhiu sai st khi thc hin Quyn 1.
Richard Fitzpatrick
157

Tp ch Epsilon, S 08, 04/2016

C. TIU S EUCLID[3]
Ngi ta khng bit nhiu v cuc i ca Euclid. T in Tiu s Khoa hc (Dictionary of
Scientific Biography) m u bi vit di v Euclid bng nhng li ny: Mc d Euclid l nh
ton hc tr danh nht mi thi i, l ngi m tn tui ng ngha vi hnh hc cho n
tn th k 20, nhng ch c hai s kin v cuc i ng c bit n, m ngay c nhng s kin
ny cng cha phi l khng cn tranh ci. Nhng s kin ny c suy din hay n on
da vo vic tham kho cc tc phm c i. u tin l ng sng thi sau Plato (mt vo nm
347 TCN) v trc Archimedes (sinh nm 287 TCN). Th n l ng lm vic Alexandria.
ng khng phi l Euclid Megara, l mt ngi bn ca Plato, l ngi vn b nhm vi ng.
Heath[4] cho bit kh nng kh d nht l Euclid (tc gi ca C s ca hnh hc) tip nhn gio
dc v ton ti Athen t cc hc tr ca Plato bi v hu ht nhng nh hnh hc c th dy ng
u xut thn t trng , v cc nh ton hc m C s ca hnh hc ca Euclid da vo u
sng v dy Athen. Nu chng ta ng vi nhng iu ny th chng ta xc nh rng Euclid
sng sau nm 347 TCN.
Quan im cho rng ng sinh trc Archimede l da vo mt ch mc tham kho dn n tc
phm V hnh cu v hnh tr ca Archimede. Tuy nhin, ch mc tham kho by gi b
cho l c chn vo sau ny.
Mc d vy, mt s ngi vn gi quan im l ch c th hi chc chn rng ng sng trc
hoc cng thi vi Appollonius (l ngi sng vo khong nm 200 TCN). Mt cht bng chng
cho chuyn ny cng l bng chng cho vic ng lm vic Alexandria l mt tham kho t
Pappus (khong nm 320 CN). Pappus nhn xt v Apollonius rng Apollonius sng kh lu
vi cc hc tr ca Euclid Alexandria, v nh th m ng c c thi quen t quy khoa hc
nh vy. Nu chng ta tin Pappus trong chuyn ny th chng ta phi t Euclid vo thi gian
trc nm 200 TCN.
Heath cng rt ra t nhn xt ca Pappus rng Euclid dy v m mt ngi trng Alexandria.
Tuy nhin, nhng ngi khc phn bc rng cho d r rng l ng c hc tr Alexandria th
iu cng khng chng minh c l ng lm vic . D g i na, chnh Apollinius
tham kho Euclid trong li gii thiu ca Quyn I b Cc ng conic. Bi v Apollonious sinh
vo khong nm 262 TCN cho nn Euclid s phi trc 200 TCN.
y l tt c nhng g chng ta bit v cuc i ca Euclid. Th cn tc phm? Heath trch
Proclus (410-485 CN) nh sau:
Euclid bin son C s ca hnh hc t vic tp hp cc nh l ca Euxodus, hon chnh
nhiu nh l ca Theaeterus, v cng chng minh mt cch khng th chi ci c nhng nh
l vn ch c chng minh mt cch lng lo bi cc bc tin bi.
T in Tiu s Khoa hc ni rng:
Danh ting lng ly ca Euclid nm b C s ca hnh hc, trong ng vit mi ba quyn
v li mt nh hng to ln ln t duy ca con ngi mnh m hn bt c tc phm no
khc ngoi tr Kinh thnh. Bi l do , ng c bit n trong thi c i nh l Ngi vit
nn C s ca hnh hc v i khi ch n gin l Nh hnh hc. tng c cc C s ca
158

Tp ch Epsilon, S 08, 04/2016


hnh hc c vit trc Euclid ng ch nht l cc tc phm ca Hippocrates, Leo and
Theudius Magnesia nhng tc phm ca Euclid vt qua chng mt cch ngon mc
n ni by gi chng ch cn c bit n qua cc ch mc tham kho ca Eudemuss nh l
Proclus dn li.
Nhiu tc phm khc c cho l ca Euclid, trong thin vn hc, quang hc, l thuyt m nhc,
cng nh l trong ton hc. Nhiu tc phm trong s khng cn hin hu na, hoc ch
cn li nhng mnh v. Sau C s ca hnh hc, cng trnh ton hc quan trng nht ca ng
vn cn gi tr l D liu (Data), l tc phm r rng c vit ra nh mt cng c gii ton
bng php phn tch. Quyn V s phn chia (ca cc s) vn cn lu truyn qua bn ting
rp ni v vic phn chia cc s thnh cc s khc khng ng dng. Quyn Cc h qu c
cho l ca Euclid b mt, nhng vn c bit n thng qua cc tc phm ca Pappus. N
bao gm khng phi l cc h qu nh trong C s ca hnh hc, m l theo Heath, cc nh l
v l thuyt ca cc ng ct tuyn v hnh hc x nh hin i. Euclid cng c cho l
vit mt chuyn lun v cc mt ct vi hnh nn, dy bn quyn, l tc phm vt qua tc
phm Cc hnh nn ca Apollonius, vn b mt.
Hai tc phm quang hc c cho l ca Euclid, Quang hc (Optics) v Phn x hc (Catropics),
m ra truyn thng rt di ca mn quang hnh hc, duy tr cho n tn u th k mi by.
[...].
Nhng g chng ta bit v Eucid l bit qua tc phm sng chi ca ng li. Quyn sch
m bn ang cm trn tay cho bn c hi c nghin cu tc phm mt cch trc tip v t
mnh nghin ngm xem v sao nh ton hc kh knh nht ca mi thi i li c vinh danh
trong sut hn hai thin nin k.
[1] c s ng ca Gio s Richard Fitzpatrick, l ngi thc hin bn ting Anh m chng
ti dng dch, chng ti ng li nguyn vn li gii thiu ca ng c gi c thm thng
tin v tc phm. (ND)
[2] Do iu kin xut bn ln ny, chng ti ch gii thiu ni dung ca cc quyn t 1 n 6
trong bn dch ting Vit. Chng ti hy vng s c iu kin thc hin v gii thiu tip cc
quyn cn li n c gi Vit Nam trong tng lai gn. (ND)
[3] c gi c thm thng tin v Euclid v di sn ca ng, chng ti dch nguyn vn phn
tiu s ca Euclid trong quyn sch Elements do Nh xut bn Green Lion Press n hnh.
[4] Sir Thomas Little Heath (1861 1940): tc gi, nh ton hc Anh, cng l nh nghin cu
lch s ton hc c i; bn dch quyn Elements ca ng c Green Lion Press n hnh.

159

Tp ch Epsilon, S 08, 04/2016

160

L VN THIM: CON NGI V S NGHIP


H Huy Khoi

1. S lc tiu s
L Vn Thim sinh ngy 29 thng 3 nm 1918 ti lng Trung L, c Th, H Tnh. Trung L l
mt lng c, thnh lp cch y khong 600 nm trn vng t trng, quanh nm b e do v
nn hn hn, lt li. Dn Trung L thun nng, ngho v hiu hc. T th k XV c ng Trn
Tc Tin s (Khoa Bnh Thn, 1496). H L Trung L ni ting v truyn thng Nho hc
v yu nc. C thn sinh ra L Vn Thim l ng L Vn Nhiu .1869 1929/, nhiu ni vit
l Nhiu (theo cch pht m ca ngi H Tnh), u c nhn Khoa Canh T .1900/: Mu thn
ca c C L Vn Nhiu, tc b ni ca L Vn Thim, l b Phan Th i, ch rut nh yu
nc Phan nh Phng. Ch rut ca L Vn Thim l ng L Vn Hun, u Gii nguyn Khoa
Bnh Ng .1906/; tham gia phong tro yu nc Duy Tn hi, ri Tn Vit ng, v t st trong
nh lao Vinh nm 1929:

C nhn L Vn Nhiu
C L Vn Nhiu tuy t nhng khng ra lm quan, m li qu nh dy hc, bc thuc,
phng dng cha m, nui dy con ci. C sinh c 13 ngi con, 8 ngi con trai, 5 ngi con
gi. Ngi anh c ca L Vn Thim l L Vn K u Tin s nm K Mi .1919/ trong khoa
thi cui cng ca Triu Nguyn. Vy l c C Nhiu c mt ngi con u Tin s cui cng
ca nn Hn hc, v mt ngi con u Tin s u tin ca nn Ty hc nc nh! Anh th hai
161

Tp ch Epsilon, S 08, 04/2016


ca L Vn Thim, ng L Vn Lun, l B th Huyn u ng Cng sn ng Dng Huyn
c Th, b Php x t hnh nm 1931: Trong s 5 ngi ch gi ca L Vn Thim c hai ngi
tham gia phong tro cch mng 1930 1931; v c cng nhn l Lo thnh cch mng.
L Vn Thim l con t trong nh, nn khi cn b, c t tn l Thm, tc l a con Tri
cho thm. Khi ra i, cu b Thm rt yu, v b m sinh n n ln th 13: M cu khng
cn sa, nn cu phi b nh ngi ch du tn l Sm, v ca anh L Vn Lun. V th, i vi
cu, b Sm cng gn nh ngi m th hai. ng Lun, b Sm u hot ng cho Tn Vit
ng. B ng vai ngi bn hng t la, ng ng vai ngi ch thu, hai ngi i khp ni
tuyn truyn cch mng, in ti liu, ri truyn n. Khi cn nh, cu b Thm hc qu nh vi
ch rut, Gii nguyn L Vn Hun. Cu ni ting hc gii, nhng cng ni ting l kh. Ln
ln, L Vn Thim theo anh c - ng Ngh K, i hc Hu, ri Quy Nhn.

Sinh ra trong mt gia nh giu truyn thng yu nc, anh thanh nin L Vn Thim sm nui
trong mnh hoi bo hc tp phng s T quc. Nm 1941; L Vn Thim thi vo trng
Ecole Normale Suprieure Ph dUlm ca Paris (trong ting Vit, ngi ta thng dch l
Trng Cao ng s phm, mt tn gi d b hiu nhm). l trng i hc danh gi nht
nc Php, ni o to nhng nh khoa hc ni ting nht. Thi vo Ecole Normale l mt
vinh d ln i vi bt k hc sinh no ca nc Php. Tt nghip Ecole Normale, L Vn Thim
tip tc lm lun n Tin s ti Thu S, ri lun n Tin s quc gia ti Php. ng tng hc
vi nhng ngi thy gii nht thi , nh Nevanlinna, Teichmuler, Valiron, v nghin cu mt
lnh vc thi s nht thi by gi: L thuyt phn phi gi tr cc hm phn hnh. ng bo v
lun n Tin s quc gia nm 1949 vi nhng kt qu m ngy nay tr thnh kinh in.
Nh nhng kt qu xut sc trong nghin cu khoa hc, nm 1949; L Vn Thim nhn c
mt gh gio s ti trng i hc Zurich, Thu S. ng l ngi Vit Nam u tin nhn chc
gio s mt i hc danh ting ca Chu u.
162

Tp ch Epsilon, S 08, 04/2016


Mt ch lm vic tut vi, mt hng nghin cu thi s, nhng kt qu u tay tr thnh
ni ting, tt c u m ra trc mt nh ton hc tr L Vn Thim mt con ng thnh thang
i n nhng nh cao ca khoa hc.
Nhng mc ch ca i ng trc ht l ng gp sc mnh cho cuc u tranh ginh t do ca
T quc. V th, nghe theo li ku gi ca Ch tch H Ch Minh, cui nm 1949; ng ri b
con ng cng danh Chu u b mt tr v nc tham gia khng chin. T Chu u, ng
v Bng Cc, ri t qua Campuchia v Nam B.

Gio s L Vn Thim, Thu S, 1943:


Nam B, Gio s L Vn Thim gia nhp ng Cng sn ng Dng v cng tc ti S
Gio dc. ng gp phn o to nhiu gio vin cho vng khng chin. t lu sau, ng ln
ng ra Vit Bc nhn nhim v mi: Lnh o trung tm i hc u tin ca nc Vit Nam
dn ch cng ho. y tht l mt nhim v quan trng v ph hp vi kh nng, nguyn ca
ng. Sau 6 thng gian nan i b t Nam B ln chin khu Vit Bc, Gio s L Vn Thim c
giao trng trch Hiu trng Trng S phm cao cp v Trng Khoa hc c bn. ng lm
ht sc mnh trn cng v , v tr thnh ngi t nn mng cho gio dc i hc ca nc
Vit Nam mi, ngi thy ca hu ht nhng nh khoa hc Vit Nam c o to trong hn
mi, mi lm nm u tin sau cch mng Thng Tm.
T sau khi ho bnh lp li, Gio s L Vn Thim c giao nhiu trng trch: Gim c Trng
i hc S phm Khoa hc H Ni .1954 1956/; Ph Hiu trng Trng i hc Tng hp
H Ni .1957 1970/; Vin trng u tin ca Vin Ton hc .1970 1980/: ng l i biu
quc hi cc Kho II v III. ng cng l i din ton quyn ca Vit Nam ti Vin nghin cu
ht nhn upna, Lin X (t 1956 n 1980/; Ch tch u tin ca Hi Ton hc Vit Nam,
Tng bin tp u tin ca hai t bo ton hc ca Vit Nam l Acta Mathematica Vietnamica v
Vietnam Journal of Mathematics.
163

Tp ch Epsilon, S 08, 04/2016

Gio s L Vn Thim (ngi ng gia) Nam B, 1949:

2. Nhng ng gp chnh v khoa hc


2.1. Cc cng trnh v l thuyt Phn phi gi tr cc hm phn hnh
L thuyt Phn phi gi tr cc hm phn hnh c xem l mt trong nhng l thuyt p nht
ca Gii tch ton hc th k XX. C th xem l thuyt ny l s m rng ca nh l c bn
ca i s. Theo nh l , a thc bc n tu c ng n nghim, k c bi. V mt no ,
hm chnh hnh l m rng t nhin ca a thc, v hm chnh hnh trn ton mt phng (hm
nguyn) c biu din bi mt chui v hn hi t. Tuy nhin, khc vi l thuyt cc a thc,
trong l thuyt cc hm chnh hnh rt kh khai thc cc kha cnh i s, m ch yu da
vo cc cng c gii tch. Vn phn b khng im ca hm chnh hnh, cng nh vn
phn b nghim ca a thc, l mt trong nhng vn trng tm. V ngay vn ny, ta gp
phi nhng kh khn c bn. Do hm chnh hnh biu din bi chui v hn, v c th c v hn
khng im trn ton mt phng, nn khng th c kt qu n gin nh trong nh l c bn
ca i s. Vn t ra l lm th no c th xt phn b khng im cc hm chnh hnh
tng t nh lm i vi a thc.
T nh l c bn ca i s suy ra rng, a thc no c cp tng cng cao th cng c nhiu
khng im. Mc d cp tng l mt trong nhng c trng quan trng ca cc hm chnh hnh,
c th thy ngay rng, m rng trc tip ca nh l c bn ca i s khng cn ng cho trng
hp cc hm chnh hnh. Tht vy, tn ti cc hm chnh hnh c cp tng rt ln (nh hm e z /;
nhng khng c khng im no. Trong trng hp cc hm phn hnh th vn cng tr nn
phc tp: Hm phn hnh l hm c th nhn gi tr v hn ti mt s im hu hn, v cn phi
c mt quan nim mi v cp tng. L thuyt phn phi gi tr ca Nevanlinna ra i nhm gii
quyt cc vn trn. Trc ht, Nevanlinna nh ngha cc hm m v hm xp x, m ta s
m t mt cch s lc nh sau. Gi s f .z/ l mt hm phn hnh trn ton mt phng, a l
mt gi tr phc tu . Khi , hm m N.f; a; r/ c mc ch o ln ca tp hp cc
im nm trong vng trn bn knh r; tm ti gc, m ti hm nhn gi tr a. Nh vy, nu
f l mt a thc bc n th khi r ln, gi tr ny l mt hng s khng ph thuc a; m ch
ph thuc bc a thc. Hm xp x m.f; a; r/ nhm o ln ca tp hp cc im nm
164

Tp ch Epsilon, S 08, 04/2016


trong vng trn bn knh r; tm ti gc, m ti hm nhn gi tr gn bng a. Hm c trng
Nevanlinna c nh ngha bi:
T .f; a; r/ D m.f; a; r/ C N.f; a; r/:
Nh vy, ni mt cch nm na, hm T .f; a; r/ dng tnh s nghim ca phng trnh
f .z/ D a trong vng trn bn knh r (k c s cc im ti hm nhn gi tr gn vi a/: Khi
nghin cu cc hm phn hnh, hm c trng T .f; a; r/ ng vai tr gn ging nh bc khi
nghin cu cc a thc. iu th hin r trong cc nh l c bn ca Nevanlinna:
nh l c bn th nht. Tn ti hm T .f; r/ sao cho vi mi gi tr a; ta c
T .f; a; r/ D T .f; r/ C 0.1/;
trong 0.1/ l i lng gii ni khi r tin ra v cng.
T nh l trn, c th xem hm T .f; a; r/ khng ph thuc gi tr a; ngha l hm phn hnh
nhn mi gi tr a (k c cc gi tr gn vi n) mt s ln nh nhau. y chnh l mt tng
t ca nh l c bn ca i s cho trng hp cc hm nguyn v hm phn hnh. Tuy nhin,
t c tng t p ni trn, ngoi hm N.f; a; r/ ta phi b sung thm mt hm xp
x m.f; a; r/ m thc cht l dng o cc im ti hm cho nhn gi tr gn vi a:
Nu s hiu chnh ny m qu ln th hin nhin, nh l c bn th nht ca Nevanlinna tr
nn t ngha.
Nevanlinna chng minh nh l c bn th hai, su sc hn nhiu so vi nh l c bn th
nht. Ni nm na, nh l c bn th hai cho thy rng i lng hiu chnh m.f; a; r/ ni
chung rt nh. nh l c bn th hai ca Nevanlinna c pht biu nh sau:
nh l c bn th hai. Vi mi s nguyn dng q v cc s phc phn bit tu ai ; i D
1; 2; : : : ; q (c th bng 1/; ta c
q
X

m.f; ai ; r/ 6 2T .f; r/ C o.log T .f; r//:

i D1

Do q l s tu , m v phi trong bt ng thc ca nh l c bn th hai khng ph thuc q


nn t c th thy rng, cc i lng m.f; a; r/ ni chung rt nh. c th nh lng
c tnh cht , Nevanlinna a ra cc hm khuyt sau y:
m.f; a; r/
;
r!1
T .f; r/
N1 .f; a; r/
.a/ D lim sup
:
r!1
T .f; r/
.a/ D lim inf

trong N1 .f; a; r/ l i lng c tnh nh N.f; a; r/ nhng mi nghim ca phng trnh


f .z/ D a ch c k mt ln (khng tnh bi).
S .a/ c gi l s khuyt ca hm ti gi tr a: Tn gi s khuyt phn nh ngha ca i
lng ny: .a/ o mc m ta phi hiu chnh c tng t ca nh l c bn ca i s,
v chnh l s nghim b thiu (khuyt) m ta phi thm vo bng cch cng thm hm xp
x, tc l thm cc im m ti hm nhn gi tr gn vi a: S .a/ c gi l ch s bi, v
165

Tp ch Epsilon, S 08, 04/2016


r rng n ph thuc vo bi ca cc nghim phng trnh f .z/ D a: Vi nhng nh ngha ,
t nh l c bn th nht, ta c:
0 6 .a/ C .a/ 6 1:
T nh l c bn th hai ta thu c bt ng thc sau y:
X
f.a/ C .a/g 6 2:

.1/

.2/

a2C[1

Bt ng thc .2/ c gi l quan h s khuyt.


T quan h s khuyt, ta suy ra rng, vi hu ht gi tr a; i lng .a/ bng 0; tr ra cng lm
l mt s m c cc gi tr ca a; ng thi tng cc gi tr cng b chn bi 2:
Cc nh l c bn th nht v th hai, cng vi quan h s khuyt lm nn ba hn tng
ca l thuyt Nevanlinna.
T quan h s khuyt, mt cch t nhin phi t ra vn sau y, thng c gi l Bi ton
ngc ca l thuyt Nevanlinna.
Cho dy (hu hn hoc v hn) cc im ak trong mt phng phc C (k c im v cng), v
cc s khng m tng ng .ak /; .ak / tho mn cc iu kin sau:
0 < .ak / C .ak / 6 1; k D 1; 2; : : :
X
f.ak / C .ak /g 6 2:
k

Vn t ra l tm hm phn hnh c s khuyt (tng ng, ch s bi) ti cc im ak l .ak /


(tng ng .ak // v s khuyt (tng ng, ch s bi) bng 0 ti cc im cn li.
Nevanlinna (nm 1932/ cho li gii ca bi ton trn trong trng hp ring vi nhng gi
thuyt cht sau y:
1. Dy fak g l hu hn.
2. .ak / l cc s hu t.
3. .ak / D 0 vi mi k:
Trong khong 15 nm tip theo k t kt qu u tin ca Nevanlinna, bi ton trn khng tin
trin thm c bc no. Cho n nm 1949; L Vn Thim tin mt bc di trong vic
gii bi ton ngc ca l thuyt Nevanlinna. Kt qu chnh m ng thu c l xy dng nghim
ca bi ton ngc vi nhng gi thit sau y:
1. Dy fak g l hu hn.
2. .ak / v .ak / l cc s hu t.
3. Nu .ak / > 0 th .ak / C .ak / < 1:
4. .ak / C .ak / D 2:
166

Tp ch Epsilon, S 08, 04/2016


ng gp quan trng ca L Vn Thim khng ch l vic chng minh s tn ti nghim ca bi
ton ngc trong nhng tnh hung tng qut hn so vi cng trnh ca Nevanlinna, m iu
quan trng l ln u tin, ng a cng c nh x bo gic v khng gian Teichmuler vo
vic gii bi ton ngc. T tng ca ng c nhng nh ton hc ni ting khc s
dng tip tc thu c nhng kt qu mi cho bi ton ngc: Goldberg, Weitsman, Drasin.
Cui cng, nm 1977; Drasin cho li gii trn vn ca bi ton ngc ca l thuyt Nevanlinna,
45 nm sau khi bi ton c t ra. iu ng ni l, trong cng trnh ca mnh, Drasin cng s
dng nhng phng php m L Vn Thim ln u tin p dng.
Cng trnh v bi ton ngc ca l thuyt Nevanlinna t L Vn Thim vo hng ng
nhng tc gia kinh in ca l thuyt ny. Ngay khi cng trnh ra i, ngi gii thiu n trn t
American Mathematical Reviews chnh l Ahlfors, ngi nhn Gii thng Fields nm 1936:
Ahlfors cng gii thiu mt s cng trnh tip theo ca L Vn Thim trn tp ch ny. Cho n
tn ngy hm nay, hu nh cun sch no v L thuyt hm phn hnh, khi ni n l thuyt
Nevanlinna u nhc n cc cng trnh u tin ca L Vn Thim. Khng phi nh khoa hc
no cng c ci vinh d c nhc n kt qu ca mnh 60 nm sau! C th tin rng, cc cng
trnh ca L Vn Thim s cn c nh n nhiu nm, nh l mt trong nhng ct mc ca
l thuyt cc hm phn hnh.
Bi bo Beitrag zum Typenproblem der Riemannschen Flachen (V vn phn loi din
Riemann) ca L Vn Thim ng trn t Commentarii mathematici Helvertici nm 1947 chnh
l cng trnh ton hc u tin ca ngi Vit Nam cng b trn tp ch quc t. C th xem
nm 1947 l nm m u cho Lch s ton hc Vit Nam hin i, v tht ng t ho khi Ton
hc Vit Nam tham gia vi ton hc th gii bng mt cng trnh u tay c ngha lch s!
Tr v Vit Nam nm 1949 theo li ku gi ca Ch tch H Ch Minh, Gio s L Vn Thim
tm dng cc nghin cu khoa hc ca mnh chuyn tm vo cc nhim v quan trng c
Nh nc giao ph. Tuy vy, khi c ch trng thc y phong tro nghin cu khoa hc trong
cc trng i hoc, Gio s li tr v vi l thuyt din Riemann yu thch ca mnh. Theo li k
ca ng, hai cng trnh ng trong tp ch Sibirskii Matematicheski Journal v Acta Scientiarum
Vietnamicarum vo cc nm 1964; 1965 l kt qu ca vic nghin cu mt vn m ng suy
ngh t khi cn Php, nhng cha c dp thc hin. Trong cc cng trnh , L Vn Thim
a ra iu kin mt mt ph l din Riemann thuc kiu hypecblic thng qua vic tn ti
u mt mula. ng cng a ra cc iu kin mt din Riemann thuc lp OHB ; tc l
trn khng tn ti hm iu ho gii ni khc hng s.
T sau hai cng trnh k trn, Gio s L Vn Thim chuyn hn sang nghin cu cc vn
ton hc ng dng, theo ch trng a khoa hc vo phc v thc tin sn xut v chin u.

2.2. Cc cng trnh v ton hc ng dng


Vn l mt chuyn gia ni ting v l thuyt hm phn hnh v din Riemann, nhng vn ca
ton hc l thuyt, Gio s L Vn Thim chuyn sang nghin cu v lnh o cc nhm nghin
cu v ton hc ng dng. iu ng ngc nhin l trong s nhng cng trnh u tin ca ng
v ton ng dng, c cng trnh tr thnh kinh in trong lnh vc ny: Li gii tng minh ca
bi ton thm qua hai lp t.
Bi ton thm l vn c nghi thc tin quan trng, xut hin khi tnh ton s bn vng ca
cc , p nc, tr lng du trong cc ti du, vn ra mn cc rung vng ven bin, ...
167

Tp ch Epsilon, S 08, 04/2016


Trong nhiu bi ton thm, chng hn khi xt nc thm qua mt con di, ta i n m hnh
bi ton thm phng (tc l khng ph thuc mt chiu no ). Vi mt s gi thit chp nhn
c, vic m hnh ho ton hc a bi ton thm qua mt mi trng ng cht v vic xy
dng mt hm chnh hnh thc hin nh x bo gic min thm ln na mt phng. l vic rt
kh khn v mt ton hc, v min thm thng ht sc phc tp. Tuy vy, ngay trong trng
hp , ta phi xt mt m hnh kh xa vi thc tin: Mi trng m nc thm qua l ng
cht, tc l ch c mt lp t vi cng mt h s thm. Trong thc tin, thng c nhiu lp vi
h s thm khc nhau nm di mt cng trnh thu li: Lp t st, lp t ct, ... i vi trng
hp min thm khng ng cht, cho n trc cng trnh ca L Vn Thim, ngi ta ch mi
c cc phng php gii gn ng. Trong cng trnh Sur un problme dinfiltratione travers
un sol deux couches. (V bi ton thm qua hai lp t) ng trn tp ch Acta Sci.Vietnam.
1; 1964; pp. 3 9; L Vn Thim dng Nguyn l i xng trong gii tch phc xy dng
c nghim tng minh cho bi ton thm qua hai lp t vi h s thm khc nhau. y l
cng trnh u tin trong lnh vc l thuyt nc thm cho php xy dng nghim gii tch ca
bi ton thm khng ng cht. iu c khng nh trong cun sch L thuyt chuyn
ng ca nc ngm ca Palubarinova-Kochina xut bn Matxcva nm 1977:
Mt hng nghin cu ng dng m Gio s L Vn Thim cng cc hc tr ca mnh tin hnh
trong nhiu nm l n nh hng. Phng php n nh hng do nh ton hc Nga Lavrenchiep
a ra, da trn nguyn tc sau y: Khi c mt v n ln, di tc ng ca p sut qu cao,
cc vt cht quanh tm ca v n chuyn ng theo quy lut ca cht lng l tng, tc l khng
nht v khng nn c. Chuyn ng ca cht lng l tng c th m t bng mt hm gii
tch. Nu tm c hm gii tch ny, ta c th tnh c p lc quanh tm n, qu o chuyn
ng ca vt cht gn tm n. Nhn thy y l vn c ngha thc tin ln, Gio s L Vn
Thim hng dn cc hc tr ca mnh ti Trng i hc Tng hp H Ni v Vin Ton
hc nghin cu p dng. Nm 1966; mt nhm cc nh ton hc tr ca hai c quan trn (gm
Ng Vn Lc, L Vn Thnh, Nguyn Vn Lm, H Huy Khoi, L Hng Sn v mt s ngi
khc) ln ng vo Ngh An tin hnh trn thc t. a im lm vic l vng Hong Mai
thuc a phn huyn Qunh Lu. Hong Mai l ni gp nhau ca ba tuyn ng vo Nam:
ng b, ng st, ng thu (knh Nh L). V th, y tr thnh mt trong nhng trng
im nh ph ca my bay M. Do ng st v ng b b h hi nghim trng, vic vn
chuyn qua knh Nh L tr nn rt quan trng. Con knh c o t thi L, n nay kh
cn. Vn cp thit t ra l phi no vt lng knh cc thuyn trng ti ln c th i qua.
Cc n v Thanh nin xung phong c giao nhim v ny. Tuy vy, khng th tp trung mt
lc lng ln, v my bay M bn ph ngy m. Gio s L Vn Thim xut dng phng
php n nh hng no vt lng knh. Mc tiu t ra l lm th no sau khi n, hu ht
t vng ln b, ch khng ri li xung lng knh. Cc v n c tin hnh vo lc thu
triu xung thp nht c hiu qu cao nht. V vy, nhiu lc phi n vo nhng gi cao
im, tc l nhng gi m my bay M bn ph c lit nht. Thc t chng t, phng php
n nh hng c tc dng rt thit thc, gp phn tng kh nng vn chuyn qua knh Nh
L, gim nh tn tht v ngi v ca. Phng php n nh hng cng c p dng trong
vic xy dng cc con ng chin lc trong rng. Cc n v Thanh nin xung phong cng
nhm hc tr ni trn ca Gio s L Vn Thim p dng l thuyt n nh hng trong vic ph
, bt ta-luy, ht nhng cy to chn ng xung vc trong qu trnh lm ng. Gio s L
Vn Thim vit mt ti liu hng dn cho Thanh nin xung phong h t lm ly sau khi
nhm nghin cu rt khi hin trng. Tic rng bn ti liu ngy nay khng tm li c.
Sau ngy t nc hon ton gii phng, Gio s L Vn Thim chuyn vo cng tc ti Thnh
168

Tp ch Epsilon, S 08, 04/2016


ph H Ch Minh. ng lp nn Phng Ton hc ng dng, nghin cu cc vn ton hc
t ra trong l thuyt n hi v chuyn ng ca cht lng nht.
Cc vn ton hc ng dng m gio s L Vn Thim quan tm nghin cu u l nhng vn
c t ra trong thc tin Vit Nam: xy dng iu v cc cng trnh thu li, ci to cc
rung nhim mn vng ven bin, tnh ton tr lng du kh, no vt lng knh phc v giao
thng thi chin. Ngay khi gii quyt cc nhim v ng dng trc mt, vi trnh cao v khoa
hc c bn, ng c nhng ng gp quan trng vo s pht trin ca l thuyt.

3. Xy dng nn Ton hc Vit nam


Vi nhng cng trnh khoa hc xut sc, L Vn Thim l ngi vit trang u tin ca lch s
ton hc Vit Nam hin i. ng cng l mt trong nhng ngi u tin t nn mng xy dng
ton hc Vit Nam. Uy tn ca ng tng l nguyn nhn khin nhiu thanh nin ti nng ln
Chin khu Vit Bc nghin cu v ging dy ton hc: Hong Tu, Nguyn Cnh Ton, ... V
khng ch li cun, khuyn khch h bng ting tm ca mnh, Gio s L Vn Thim dn
tm sc o to lp thanh nin y nhit huyt ca nhng ngy u cch mng. Vn ling
ca ng khi tht t i, ch l mt t sch m ng v mt s gio s khc c gng mang
theo mnh sut chng ng t chu u n chin khu. ng lun khuyn khch nhng ti nng
tr i su vo nghin cu khoa hc, v c gng to cho h nhng iu kin tt nht c th.
Ngay c sau khi ho bnh lp li, cc trng i hc Vit Nam hu nh cha c gio trnh i
hc v ton bng ting Vit. Vy m mt trong nhng quyt tm ln ca Nh nc Vit Nam mi
l ging dy ting Vit bc i hc. L Vn Thim dch v vit cc gio trnh, t Hm bin
phc cho n Xc sut thng k. n nm 1964; chng ti vn c Th vin cho mn cc gio
trnh do ng dch, nh my bng ting Vit khng du: C l do thi quen khi cn Php, hoc
l tit kim thi gian khi vit, ting Vit ca Gio s L Vn Thim thng khng c du!
Nhn thc r tm quan trng ca Ton hc trong vic xy dng nn khoa hc nc nh, Gio s
L Vn Thim cng vi cc Gio s T Quang Bu, Hong Tu vch mt chin lc lu di
pht trin Ton hc Vit Nam. S ra i ca Phng Nghin cu Ton nm 1962 (trc thuc U
ban Khoa hc v K thut Nh nc) l mt ct mc quan trng trong qu trnh xy dng nn
ton hc Vit Nam.
Nm 1969; Th tng Phm Vn ng k quyt nh thnh lp Vin Ton hc thuc U ban
khoa hc v K thut Nh nc. Nm 1970; Gio s L Vn Thim, lc ang l Ph Hiu
trng Trng i hc Tng hp H Ni, c chuyn v gi chc v Ph Vin trng, Ph
trch Vin Ton hc. T lc , Vin Ton hc chnh thc i vo hot ng.
Vi s lnh o ca Gio s L Vn Thim, t khi thnh lp, Phng Nghin cu Ton, v sau ny
l Vin Ton hc ch trng pht trin ton din: Nghin cu c bn, nghin cu ng dng v
o to. Nhng sinh vin gii tt nghip i hc Tng hp H Ni v cc i hc nc ngoi
c chnh Gio s L Vn Thim tuyn chn v Vin Ton hc, v c c i tip tc nghin
cu, hc tp nc ngoi. Chnh nh chin lc o to c bn ca Gio s L Vn Thim
m Vin ton hc, t ch ch c hn 20 cn b nm 1970; n nay tr thnh mt Vin nghin
cu hng u c nc vi 100 cn b, trong c 19 Gio s v 22 Ph gio s, 28 Tin s khoa
hc, 38 Tin s, c 7 Tin s khoa hc, 119 Tin s v 200 Thc s c o to ti Vin.
Sau ngy ti thng nht t nc, Gio s L Vn Thim vo cng tc ti Thnh ph H Ch
Minh. Gio s lp nn Phng Ton hc ng dng, vi nhim v nghin cu nhng vn gn
169

Tp ch Epsilon, S 08, 04/2016


vi cc ng dng thc tin, c bit l cc vn t ra ti Min Nam nh thu li ng bng
sng Cu Long, du kh.
Gio s L Vn Thim, cng vi Gio s Hong Tu, l nhng ngi u tin gy dng Khoa
Ton ca Trng i hc tng hp H Ni. ng lun kin tr phng chm gi vng cht lng
o to, ngay c trong nhng nm chin tranh, khi nh trng phi s tn vo vng ni Vit Bc.
ng cng phi tri qua nhiu cuc u tranh gay go trong ni b Khoa Ton v Trng i
hc Tng hp H Ni trong nhng nm 60 gi vng chin lc ng n . Nh th, Khoa
Ton ca i hc Tng hp H Ni (nay l i hc Khoa hc t nhin thuc i hc Quc gia
H Ni) o to nn nhiu nh ton hc hng u trong c nc.
Gio s L Vn Thim cng l Ch tch u tin ca Hi Ton hc Vit Nam. Vi uy tn, ti
nng v c ca mnh, Gio s l ngi lnh o, cng ng thi l ht nhn gn kt cng
ng ton hc Vit Nam.
Sut i ht lng v th h tr, Gio s L Vn Thim l mt trong nhng ngi sng lp t bo
Ton hc v Tui tr, v trc tip vit bi cho bo ngay t nhng s u tin. ng cng trc tip
ra thi chn hc sinh gii ton Min Bc nhng nm 1963 1964: ng khng n h vic g,
d to d nh, min l c li cho vic du dt th h tr. Nhiu hc sinh gii gp kh khn khi xt
tuyn vo i hc do gia nh, h hng b mt s nh kin v l lch tm n ng, v c
gip tn tnh. Nhiu ngi trong s h tr thnh nhng nh ton hc gii, c nhiu ng
gp cho t nc.
Ngay khi c nc ang trong chin tranh, my bay M bn ph d di min Bc, Gio s L Vn
Thim l ngi ng ra sng lp t bo Ton hc v Vt l bng ting nc ngoi u tin ca
Vit Nam: T Acta Scientiarum Vietnamicarum (Sectio Mathematicarum et Physicarum). Phn
ton hc ca t bo ngy nay tr thnh t Acta Mathematica Vietnamica, t bo c uy tn
nht v ton ca vit Nam, c mt th vin ca nhiu trng i hc ln trn th gii. Vic cho
ra i mt t bo nghin cu ton hc (bng ting Anh, Php, Nga, c) trong chin tranh l
iu him c trn th gii. Nhiu nh khoa hc nc ngoi t ngc nhin v khm phc khi
thy Vit Nam, mt t nc ang phi ng u vi cuc chin tranh tn khc nht c hai
min, li ngh n vic ra mt t tp ch nghin cu khoa hc bng ting nc ngoi. Vic lm
chng t tm nhn xa ca cc nh lnh o khoa hc ca Vit Nam, v c s tin tng vo
thng li tt yu ca s nghip cch mng.
S pht trin ca Ton hc Vit Nam, v ca khoa hc c bn Vit Nam ni chung t sau Cch
mng Thng Tm mang m du n ca Gio s L Vn Thim.

4. Thay li kt lun
Kh c th ni ht trong mt bi vit ngn tt c nhng g m Gio s L Vn Thim lm v
s pht trin mt nn Khoa hc Vit nam. Trong tp sch L Vn Thim cc cng trnh khoa
hc, c gi s tm thy nhiu bi vit ca nhng ngi tng hc, tng cng tc vi Gio s
L Vn Thim. Hy vng qua nhng bi vit , c gi hiu r hn v Nh Khoa hc, Nh Gio,
ngi Tr thc, ngi chin s L Vn Thim.
170

Tp ch Epsilon, S 08, 04/2016

Gio s L Vn Thim, 1950:


ng thuc vo s nhng con ngi khng lp li ca Lch s.

171

Tp ch Epsilon, S 08, 04/2016

172

CC VN C IN V HIN I
Trn Nam Dng - i hc Khoa hc T nhin - HQG TP.HCM

LI GII THIU
Chuyn mc ny dnh cho cc vn c in v hin i c trnh by di
dng cc bi ton xu chui. c th l chui cc bi gii bi ton ng chu,
1
2
1
chng minh ng thc Euler k diu 1 C 2 C 2 C    D
; mt chui bi ton
2
3
6
vn tr ... Cch trnh by xut pht t nhng vn n gin, d hiu, nhng khi
nim mi s c nh ngha lun trong bi c th c tng i c lp. V mi
mt chui bi s nu ra nhng vn nht nh, c th l gii quyt mt bi ton
kinh in hay nu ra nhng gi thuyt mi, nhng vn mi. Li gii v tho lun
v cc bi ton s c ng s N C 3:
Trong s ny, chng ti xin gii thiu vi bn c cc bi ton trong thi tt
nghip ca Php do GS Nguyn Tin Dng gii thiu v c hai bn L Phc L,
Hunh Cng Bng dch. Ngoi ra chng ti s ng tm tt li gii cc bi ton
ng s 4:

173

Tp ch Epsilon, S 08, 04/2016

tt nghip THPT mn Ton ca Php


Bi vit c GS Nguyn Tin Dng gii thiu v c hai bn L Phc L, Hunh
Cng Bng dch.

Mt s thng tin v k thi:


1. Thi gian lm bi l 5 gi.
2. Th sinh c s dng my tnh b ti.
3. Th sinh cn trnh by r rng, mch lc cc bn copy ca bi lm d c.
4. thi c tng cng 3 bi v th sinh c th lm theo th t ty .

1. Tng ca nhng con s lp phng


Vi mi s nguyn dng n; ta gi n3 l lp phng ca n: Trong bi ton ny, ta quy c rng:
 S l tp hp cc s nguyn dng c th biu din di dng tng ca cc s lp phng
phn bit.
 S0 l tp hp cc s nguyn dng c th biu din di dng tng ca cc s lp phng
chn phn bit.
 S1 l tp hp cc s nguyn dng c th biu din di dng tng ca cc s lp phng
l phn bit.
V d:
 8; 190 2 S v 8 D 23 v 190 D 13 C 43 C 53 I
 216; 1072 2 S0 v 216 D 63 v 1072 D 23 C 43 C 103 I
 125; 2568 2 S1 v 125 D 53 v 2568 D 13 C 33 C 73 C 133 :
Bng cch tr li cc cu hi sau y, chng ta s chng minh rng mi s nguyn dng ln
u thuc S; tc l tn ti N sao cho vi mi s nguyn dng n  N; ta u c n 2 S:
1) Chng minh rng 2016 thuc S0 :
2) Chng minh rng:
a) Vi mi s thc x  5 th
.2x C 1/3  2.2x
174

1/3 :

Tp ch Epsilon, S 08, 04/2016


b) Vi s nguyn dng k > 5 khi vi mi s nguyn p  k ta lun c
3

.2p C 1/  .2k

1/ C

p
X

.2j

1/3 :

j Dk

3) Chng minh rng tn ti 288 s s1 ; s2 ; : : : ; s288 thuc S1 sao cho si  i .mod 288/ vi
mi i D 1; 2; 3; : : : ; 288:
Tip theo, ta c nh b s .s1 ; s2 ; : : : ; s288 / nu v t m l s ln nht trong chng.
4) Gi n l s nguyn dng tha mn 288n  m v xt cp s cng nguyn dng
u1 ; u2 ; : : : ; un c cng sai l 288: Chng minh rng cc s nguyn thuc on m C u1 ; 288n C u1
u c th biu din c di dng si C uj , trong 1  i  288 v 1  j  n:
5) Cho bit rng vi mi s thc x, ta c ng nht thc sau
.2x C 12/3 C .2x C 4/3 C .2x C 2/3

.2x C 10/3

.2x C 8/3

.2x/3 D 288:

Hy chng minh rng:


a) Tn ti mt s nguyn dng u sao cho u; u C 288 v u C 576 u thuc S0 :
b) Vi mi s nguyn n  2, tn ti n phn t trong S0 to thnh mt cp s cng c
cng sai l 288:
6) Cho k l mt s nguyn dng khng nh hn 5 tha mn .2k C 1/3 > m:
dng N  1 sao cho mi s nguyn dng
a) Chng minhrng lun tn ti s nguyn

3
thuc on N; N C 2.2k 1/ u c th biu din c di dang si C u vi
1  i  288 v u 2 S0 :
b) Chng minh rng mi s nguyn dng ln hn hoc bng N u thuc S:
Gi . Vi mi s nguyn dng p  k, ta xem xt trng hp cc s nguyn dng


nm trong on N; Np , trong
Np D N C .2k

1/ C

p
X

.2j

1/3 :

j Dk

2. Thm him rng rm


Mt nh thm him ang khm ph mt khu rng v pht hin ra rng tt c cc thn cy trong
u c cng bn knh. y qu tht l mt iu rt th v! Tm thi b qua chiu cao ca cc
cy, ta t khu rng vo trong mt phng ta vung gc nh sau:
 Nh thm him ang ng ti gc ta v mun tm mt gc nhn xuyn qua khu rng.
 Cc thn cy c xem nh cc vng trn c bn knh R > 0 v ta tm t ti cc im
nguyn .a; b/ vi a; b l cc s L. Do tnh i xng nn ta c th gi s a > 0; b > 0,
tc l ch xt cc im thuc gc phn t th nht.
175

Tp ch Epsilon, S 08, 04/2016


Ta ni rng nh thm him c th nhn xuyn qua khu rng nu nh c mt tia xut pht t v
tr ng ca anh ta (ti gc ta ) v i qua khu rng m khng ct bt c gc cy no ( y l
cc hnh trn).

v d trong hnh, tia D b vng phi mt gc cy trong khi D0 th khng. V do c mt tia D0


nh vy nn ta ni nh thm him c th nhn xuyn qua khu rng.
Vi m 2 .0I C1/, ta gi Dm l tia gc O c phng trnh y D mx v x > 0:
Tip theo, ta tha nhn tnh cht sau: Vi mi s v t dng m v mi s " > 0 nh ty , lun
tn ti hai s nguyn dng a; b l sao cho
jb

maj < ":

Hy tr li nhng cu hi sau:
1) Cho a; b; m l cc s thc dng. Chng minh rng Dm ct ng trn c bn knh R > 0
c tm t ti .a; b/ khi v ch khi
p
jb maj  R 1 C m2 :
2) T suy ra rng nu m l s v t th Dm s vng phi mt gc cy.
3) Gi s m D

b
vi a; b l cc s nguyn dng v nguyn t cng nhau:
a

a) Gi s a; b cng l. Hi ng thng Dm c vng phi gc cy no khng?


176

Tp ch Epsilon, S 08, 04/2016


b) Gi s a; b khc tnh chn l v Dm b vng vo mt ci cy. Chng minh rng
p
1  R a2 C b 2 :
4) T chng minh rng nu tt c cc ng thng Dm vi m > 0 u vng phi gc cy
1
no th R  p :
5
1
5) Gi s rng R  p . Chng minh rng cc tia Dm vi m > 0 s vng phi cc gc cy
5
ti ta .; 1/ hoc .1; / vi l s nguyn dng l.
Ta gi hng u tin l tt c cc cy c tm nm v tr .1; / hoc .; 1/ vi l s l.
6) Chng minh rng nu ngi quan st c th nhn xuyn qua hng u tin ny th c th
nhn xuyn qua c khu rng.

3. Hnh trnh trong min s phc


Trong bi ton ny, ta quy c k hiu j D e

2i
3

v xc sut ca bin c A l P .A/:

1) a) Chng t rng j 3 D 1 v 1 C j C j 2 D 0:
b) Trong mt phng phc, ba im biu din 1; j; j 2 to thnh tam gic c c im g?
c) Chng minh rng nu a; b; c l cc s thc tha mn a C bj C cj 2 D 0 th a D b D c:
Cho mt con xc sc ng cht c 6 mt (c nh s t 1 n 6). K hiu F l bin
ngu nhin ch s xut hin khi tung con xc sc v Z l bin ngu nhin j F :


2) Chng minh rng ta lun c Z 2 1; j; j 2 v
1
P .Z D 1/ D P .Z D j / D P .Z D j 2 / D :
3
Xt s nguyn n  1 v tung con xc sc n ln c lp vi nhau. K hiu Fk l kt qu ca
ln tung th k v t Zk D j Fk . t Sn D Z1 C Z2 C    C Zn v pn D P .Sn D 0/.
Gi Un ; Vn ; Wn ln lt l cc bin ngu nhin ch s cc s k 2 1I n m
Zk D 1; Zk D j; Zk D j 2 :
3) a) Tnh Un C Vn C Wn .
b) Chng minh rng
Sn D Un C j Vn C j 2 Wn :
c) Chng minh rng nu Sn D 0 th Un D Vn D Wn .
d) T chng minh rng nu n khng chia ht cho 3 th pn D 0:
177

Tp ch Epsilon, S 08, 04/2016


4) Gi s rng n c dng 3m vi m l s nguyn dng.
a) Chng minh rng Un l mt phn phi nh thc. Xc nh cc tham s tng ng ca
phn phi ny.
b) T suy ra rng

3m 22m
:
P .Un D m/ D
m 33m


K hiu PUn Dm .Vn D m/ l xc sut c iu kin ca bin c Vn D m cho bit trc


Un D m:
c) Chng minh rng
PUn Dm .Vn D m/ D 2

2m

 
2m
:
m

d) T suy ra
p3m D 3

3m

  
3m 2m
:
m
m

e) Chng minh rng


p3mC3
.3m C 2/.3m C 1/
D
:
p3m
9.m C 1/2
5) Vi mi s nguyn dng m  1, chng minh rng
m
p3mC3
2

v p3m 
:
mC1
p3m
9m
t Xn l bin ngu nhin ch s cc s nguyn k 2 1; n sao cho Sk D 0:
6) a) Xc nh bin ngu nhin Bernoulli Yk vi 1  k  n tha mn
Xn D Y1 C Y2 C    C Yn :
b) K hiu E.Xn /; E.Y1 /; E.Y2 /; : : : ; E.Yn / l gi tr k vng ca cc bin Xn ; Y1 ;
Y2 ; : : : ; Yn theo th t . Cho bit rng E.Xn / D E.Y1 / C E.Y2 / C    C E.Yn /; hy
chng minh
E.Xn / D p1 C p2 C    C pn :
c) T suy ra
lim E.Xn / D C1:

n!C1

t qn D P .Xn > 0/, cu hi tip theo yu cu chng minh rng qn hi t v 1:


7) a) Chng minh rng dy s .qn / hi t v mt s thc q vi qn  q  1 vi mi n:
b) Vi r; n l cc s t nhin, chng minh rng
P .Xn  r/  q r :
c) T suy ra vi mi s nguyn n  1, ta c
E.Xn /  q C q 2 C    C q n :
d) Rt ra kt lun.
178

Tp ch Epsilon, S 08, 04/2016

Chia on thng
Bi ton c xut bi A.K.Tolpygo, K.K.Kokhas v A.Mogileva cho Hi
ngh ma h, cuc thi gia cc thnh ph nm 2014:
s ny chng ti xin c gii thiu li gii cho cc bi ton c ng
s 4 ca tp ch, tin cho vic theo di ca c gi chng ti xin c ng li
phn bi.

1. Dn nhp
Chn s ; trn on 0; 1 ta nh du cc im fg; f2g; f3g; : : : ; f.n

1/g:

p
;
q
ta s gi s rng p < q: Nh vy, cho d bng bao nhiu, khng c hai im no trng nhau.
Trong sut bi ny, nu nh khng ni iu ngc li, ta s gi s rng v t. Nu nh D

Nh th, an 0; 1 s c chia thnh n phn. Tip theo ta s gi s n > 10 v 0; 3 < fg < 0; 7:


Cc hn ch ny thc ra khng quan trng, ta a ra cc iu kin ny loi b cc hiu ng
hin nhin cho cc s nh. Nhng t iu kin ny c th suy ra l mi phn u nh hn :
Ta cng ch rng nu nh thay bng n C hay n

th cc phn vn nh vy. V vy trong


1
cc bi ton, trong ni v tnh duy nht, ta c thm iu kin 0 < < :
2
Ni dung ca bi ton nghin cu xem ta c nhng phn nh th no v chng sp xp ra sao.

Cc bi ton
 T l gia on di nht v on ngn nht ta k hiu l L D L.; n/:
A1. Gi s D

p
l s hu t. Chng minh rng tn ti n sao cho L.n/ D 1:
q

A2. Vi nhng s nguyn hay hu t k no, k > 1 ta c th khng nh rng vi mi s hu t ;


tn ti n sao cho L.n/ D k
Li gii. L D 2 v L D 3: Cc s khc khng tha mn nh c th d dng a ra cc v d:
5
4
Chng hn nu ly D
v D ; th chng khng c gi tr L no ging nhau ngoi hai
12
13
gi tr ni trn.
 Tip theo ta khng gi s hu t na.
B1. Chng minh rng cho d n bng bao nhiu, trong cc phn c khng qu 3 di khc
nhau (v cng hin nhin l nu v t th c t nht 2 di khc nhau).
179

Tp ch Epsilon, S 08, 04/2016


Li gii. Xt mt on thng m ta thu c bc th n: Gi s u mt ca n l cc im
fkg; flg: Hin nhin l on thng ny cng c di bng vi di cc on gii hn bi
cc im f.k 1/g; f.l 1/g; f.k 2/g v fl 2/g; : : : V chui ny hu hn nn trong
c phn t u tin, chng hn l f.k s/g v f.l s/g: Hin nhin ch c 3 nguyn
nhn ti sao khng c on thng trc :
 hoc k D s;
 hoc l D s;
 hoc, cui cng, on thng f.k s 1/g; f.l s 1/g tn ti nhng b chia ra
thnh 2 phn. Trong trng hp cui n ch c th b chia ra bi im n; nu khng on
thng vi u mt l f.k s/g v f.l s/g cng b chia ra.
Hai chui u lun tn ti, cn chui th ba c th khng tn ti. T y m ta c hoc 2; hoc 3
chui m trong mi chui di cc on thng bng nhau nh yu cu.
T y cng suy ra rng cc on thng ca chui th ba (nu chng tn ti) s c di bng
tng di hai on thng ca hai chui cn li. Nh vy, di cc on thng lun bng
a < b < .a C b/ trong di th ba c th khng c.
T y cng suy ra rng im mi s lun chia on thng di nht c di a C b ra 2 phn
a; b (trong trng hp ngc li ta s c 4 di khc nhau, l iu khng th).
By gi ta nghin cu cu hi s L s thay i nh th no khi chuyn t n ln n C 1: T nhng
iu ni trn r rng l s n vn l s bc 3 nu nh vn cn cc on thng dng .a C b/;
v vo thi im khi on thng cui cng nh th bin mt, s ch cn li a v b; tc l s s tr
thnh bc 3: Khng nh:
(a) Nu n v n C 1 l cc s bc 3 th L khng thay i.
(b) Khi chuyn t s bc 3 sang s bc 2 .n bc 3; .n C 1/ bc 2/L chuyn thnh L

1:

(c) Khng c s chuyn i t s bc 2 sang s bc 2:


(d) Khi chuyn t s bc 2 sang s bc 3 c hai trng hp. Gi s a < b l di cc on
ngn v on di ca n: Khi :
b
th L khng thay i.
2
b
L
(d2 ) Nu a > th L i thnh
:
2
L 1

(d1 ) Nu a <

Tht vy:
(a) Trong trng hp th nht, cc on thng lc u v sau vn th, c di l:
a; b; a C b:
180

Tp ch Epsilon, S 08, 04/2016


(b) Trong trng hp th hai on thng c di a C b bin mt, ch cn li a < b v nu
aCb
b
trc L D
th by gi L D :
a
a
(c) Trong trng hp th 3 cn nhc li gi thit ca chng ta l n > 10 v 0; 3 < fg < 0; 7
(nu khng c iu ny th khng nh khng ng).
(d) C hai trng hp .d1 /; .d2 / u c xt ging nh .a/ v .b/:

 Vi s cho ta s ni s n l bc hai nu ch c 2 di khc nhau v bc ba nu c 3


di khc nhau.
B2. Cho l s v t, chng minh rng khi tn ti v s n bc hai v v s n bc ba.
B3. Cho s v t ; v n chy qua cc gi tr n D 1; 2; : : : ; m: Chng minh rng m ! 1; th
t l s bc hai trong chng dn n 0:
Gi .m/ l s cc s bc 2 trong cc s n D 1; 2; : : : ; m: Hy nh gi tc dn n 0 ca
.m/
t s
khi m ! 1:
m
Li gii. T cc l lun trong bi B1 ta thy rng tip sau mt s bc 2; s bc 3 s ng bng
s cc on thng di nht. V s cc on thng tng v hn, kh hin nhin l s cc on
thng c di ln nht cng dn n v cng, v l iu ta cn.
B4. S phi bng bao nhiu t l ny tin n 0 chm nht c th? Nhanh nht c th ?
(Ch cn a ra mt s v d, nhng phi thuyt phc).
B5. Hy nh gi chn trn v chn di s cc s bc hai trong 1 triu s u tin (cng ng
cng tt, nh gi ln hn 3 khng c chp nhn).
Li gii. nh gi chn trn: Khng qu 500:000 - v nh gi ny khng th lm cht hn.
1
1
thu c s lng cc s bc 2 nh vy, ta cn chn D
 vi  < 6 : Trong trng hp
2
10
ny d thy mt na l s bc 2:
T y cng suy ra rng t l cc s bc 2 c th tin n 0 mt cch rt chm. C th hn, trong
khi chng ta c nhiu s bc 2 1 triu s u tin th sau chng xut hin t hn. Nhng nu
1
1
1
nh ta ly tip theo l D
 vi  < 18 th chng s i rt lu vi tn xut mt
6
2 10
10
1
phn 6 , tc l n mt lc no th t l ny l n nh. R rng cu trc nh vy c th
10
1
1
1
tip tc, ly v d D
 : : : y mi bc chuyn t l s bc hai, s
6
2 10
10106
tht l, s gim mt cch ng k nhng sau th RT, RT lu tr thnh khng i. V chnh
iu ny c ngha l dy s dn n 0 rt chm.
i vi vic tin n 0 nhanh nht th c nhng s ; m i vi nhng s t l ny gim
theo logarit (v d s c cho di y), v c l y l tc ln nht. y l tc gim ln
v iu ny c ngha l trong 1 triu s u tin ch c khong ln 106 s bc 2:
181

Tp ch Epsilon, S 08, 04/2016


C1. Tn ti hay khng s ; sao cho L > 10 vi mi n, bt u t n D 10
Li gii. Khng tn ti, v t cc cng thc bin i s L ta thy rng n lun nhn gi tr nh
hn 2:


1
C2. Vi n D 2000000: C th xy ra trng hp trn on a; a C
no c hn 1100000
2
im ? (Nhc li l theo gi thit, ta c 0; 3 < < 0; 7/:
1
Li gii. C, v y l v d: Gi s D C  trong  rt nh. Khi tt c 2 triu im
3
1
2
nm gn 3 im gm im 0; im v im : Tng ng trn on 0:3; 0:8 s cha khong
3
3
2
6
tt c cc im ln hn 10 rt nhiu.
3
C3. Tn ti hay khng ; sao cho L nhn:
 V hn.
 Hu hn cc gi tr khc nhau khi n chy qua cc gi tr t 10 n 1: Nu tn ti, hy
a ra cc v d.
p
Li gii. C, tn ti c loi ny v loi kia. V d, nu D 2 th L nhn ch 3 gi tr khc
nhau (v d ny s c xt chi tit bn di). Mt khc, nu L nhn mt gi tr siu vit no
L
(v d L D ) th L c th bin i khi th thnh L 1; khi th thnh
v ta thy lun
L 1
nhn c nhng s khc, tc l chng s c v hn gi tr.
C4. Hy a ra cc iu kin no L nhn hu hn (v hn) cc gi tr (nu c th, hy
tm iu kin cn v , nhng c th gii hn cc iu kin no ).
Li gii. Nu nh L ch nhn hu hn gi tr th n mt lc no cc gi tr ca L s vo
vng lp: Cc gi tr xut hin trc s lp li.
Nhng nu sau mt s ln bin i (m chng ta bit rng c dng hoc L ! L 1 hoc
L
L!
ta li thu c gi tr L gp trc th d thy L phi tha mn mt phng
L 1
trnh bc 2 vi h s nguyn.
Nh vy y l iu kin cn v tng ng, nu L khng phi l nghim ca phng trnh bc
hai (v d nh nu L siu vit) th iu ny L nhn v s gi tr.
iu kin L nhn hu hn gi tr, v d c th l iu kin: L2 .n C 2/L C n D 0:
p
V d s 3 v s  m chng ta xt cc bi ton di y tha mn iu kin ny.
D1. Chng minh rng vi mi s v t cho trc tn ti gi tr m L nhn hn 1000 ln (vi
nhng gi tr n khc nhau).
182

Tp ch Epsilon, S 08, 04/2016


Li gii. L s lin tip nhn mt gi tr cho n khi ta gp s bc 3; ni cch khc s ln s
bng s on thng di xut hin nc i hin ti. Nhng s nhng on thng nh th dn n
v cng (nh ni li gii bi B3).
Hin nhin l khi s on thng dn n v cng th s on thng di cng dn n v
cng.
D2. Mt cch logic, c th xy ra ba trng hp:
.1/ D bng bao nhiu, tn ti gi tr sao cho L nhn v s ln.
.2/ D bng bao nhiu, L nhn mi gi tr ch hu hn ln.
.3/ Vi mt s iu ny ng, cn vi nhng khc iu kia ng.
iu ny trn y ng ? Nu nh iu th .3/ ng th vi iu .1/ ng, vi iu .2/
ng?
Li gii. Xy ra iu th ba.
D3. Gi s rng vi no L nhn mi gi tr A v B t nht mt ln (vi n > 10/: Liu hai
khng nh sau c tng ng:
.a) L nhn gi tr A hu hn ln.
.b/ L nhn gi tr B hu hn ln ?
p
100 C 2
Li gii. Khng, khng ng. V d nu nh ban u L nhn gi tr
p : V s ny nh
99 C 2
L
hn 2 nn gi tr tip theo ca L phi c tnh theo cng thc
; v c ngha l gi tr tip
L 1 p
p
p
p
theo ca L l 100 C 2: Tipp
theop
L s lnp
lt nhn cc gi tr 99 C 2; 98 C 2; : : : ; 2:
Sau L ch nhn cc gi tr 2; 2 C 1; 2 C 2 (v khng cn gi tr no khc), v cc gi
tr ny u c nhn v hn ln.

2. Cc bi ton lin quan


Phn ny, ta ch yu xt cc bi ton lin quan n cc gi tr c th ca :
p
1C 5
 K hiu  l t s vng:  D
 1; 618 : : :
2
E1. Ta a ra 3 phng php vit dy cc k t AABAABABAAB : : : Hy chng minh rng c
ba phng php cho ra cc kt qu ging nhau (mt cch chnh xc hn mnh s c pht
biu bn di y).
Cc phng php nh sau:
183

Tp ch Epsilon, S 08, 04/2016

.1/ Trn li vung k mt tia, n bt u t mt nt li v to mt gc bng  vi ng


nm ngang ca li.
Ti im nt u tin ta vit k t A; sau vit cc k t vo cc im giao ca tia vi
cc ng thng ca li: K t A nh du giao im vi cc ng thng ng, k t B
nh du cc giao im vi ng nm ngang (xem hnh).
.2/ u tin ta vit k t A; sau thc hin mt s bc, mi bc, k t A c thay bi
AAB; cn k t B c thay bi AB: V d sau 3 bc ta thu c
u tin l AAB;
sau l AABAABAB;
v bc th ba AABAABABAABAABABAABAB:
Chng minh rng mi mt dy s thu c bng cch ni trn l on u ca dy s c
nh ngha .1/:
.3/ on 0; 1 c chia bng phng php m t phn dn nhn vi, trong D :
Dy s c xc nh nh sau: Vi mi n l s bc 2; ta vit theo th t di cc on
thng bt u t cui (tc l t 1/; on di k hiu l A; on ngn l B:
Chng minh rng trong s cc dy s hu hn, c v s cc dy s l on u ca dy s .1/:
Cc dy s ny vi n1 D 3 < n2 D 8 <    l nhng dy s no ?
Li gii. Ta s a cc nh ngha 1 v 3 v 2:
nh ngha 1. Ta nng im A ang nm ng thng ng th k ln thm k n v. Hin
nhin tt c cc im ny s tri vo mt ng thng to vi trc honh gc arctg (ch khng
phi arcctg). Mt khc, nu sau ta t tt c cc im B; nh yu cu, th gia hai ch A s
c ng mt im B: Ta thc hin php bin i th hai: Tt c cc ch A thay bng B; tt c
cc ch B thay bng A:
Nh vy, dy ca chng ta s bin i theo quy lut: Thay A bng BA v B bng A: Kt qu
hin nhin dy ca chng ta s khng thay i bi v nu ta ly i xng hnh qua ng cho
(ng thng y D x/ th tia ca chng ta tr v v tr c. By gi ch cn ch rng nu ta thc
184

Tp ch Epsilon, S 08, 04/2016


hin php i hnh hai ln th A s c thay bng AAB v B thay bng AB m dy mt ln
na li khng i.
Ghi ch. V bn cht chng ta chng t rng bin i A ! AAB; B ! AB c th biu din
di dng tch: u tin ta thc hin bin i A ! BA; B ! A v sau A ! AB; B ! A
ta thu c bin i cho.
nh ngha 3. Ta l lun tng t. Gi s N l s bc 2: iu ny c ngha l c r on thng
a
di di a v s on thng ngn di ; r C s D N: Khi s bc 2 tip theo M D N C r

v khi di chuyn v s ny, ta chia mi on thng di di ra on ngn v ngn nht c di
thm  ln nh hn.
Khi vng lp ny kt thc, ta bt buc phi i on thng ngn c thnh on thng di. Trong
on thng di c s c tc ra thnh di ngn, hoc ngn - di. y chnh l cc
bin i c m t trong phn nhn xt ca l lun phn trc. T y ta cng thy r
rng l ti sao khng phi s bc 2 no cng c: Ta phi ly nhy cch 1 thu c bnh
phng ca php bin i ny.
E2. Dy s c xc nh theo quy tc trn. Gi s ta ly hai khc, mi khc c n k t: T k
t th k C 1 n k t th k C n v t k t th m C 1 n k t th m C n: Chng minh rng
s cc k t A trong cc khc ny gn bng nhau, c th l: S cc k t A mi khc cch
nhau khng qu 1:
Li gii. Ta s dng nh ngha 1: Gi s rng khng nh khng ng. Khi trn tia s tn ti
hai on thng c s ch ci bng nhau, nhng on ny c s ch ci B nhiu hn hay bng 2
so vi on kia (v s ch ci A tng ng s t hn).
Nhng nu trn mt on c nhiu hn 2 ch ci B th hnh chiu ca n ln trc tung ln hn.
Tng ng, nu trn c t hn hai ch ci A th hnh chiu ca n ln trc hong nh hn.
Mu thun l hin nhin.
E3. Hy tm thm mt dy no m c th c xy dng bng ba phng php tng t (hay
t nht l hai).
Li gii. im mu cht trong li gii bi ton 1 l s kin khi nng cc giao im ca tia vi
cc ng thng ng ln mt s n v tng ng ta s thu c tia i xng vi tia ban u.
Nhng ta c th nng im nm ng thng ng th k khng phi ln k n v, m 2k n
v th d thy tia nhn c s i xng vi tia ban u nu tha mn phng trnh
2 C 2

1 D 0:

Tt nhin l c nhiu cch xy dng khc na.


 Trong chui bi ton sau ta ly D :
P1. Tm tt c cc gi tr c th ca L (vi cc n khc nhau).
P2. L c th nhn gi tr no u n l s bc 2
185

Tp ch Epsilon, S 08, 04/2016


Li gii. Nu n D 2; th ta c hai on thng v t s hai on thng l t l vng. Tip theo, theo
L
cng thc a ra u (trong li gii bi B1), L ch c th thay bng
D L2 D L C 1:
L 1
Gi tr tip theo ca n l .L C 1/ 1 D L; : : : T y ta thy rng L ch nhn 2 gi tr, v nu
nh cn c thm n l s bc 2 th L nhn gi tr duy nht L D :
P3. Tm tt c cc s bc 2:
P4. Tm s cc s bc 2 trong 1 triu s u tin nu D : Ch cn p s chnh xc n 10%:
Li gii. Cc s bc hai i vi cho l cc s Fibonacci, tc l cc s 2; 3; 5; 8; 13; : : :
(mi s tip theo bng tng hai s trc ).
S Fibonacci th k xp x bng k; v s lng ca chng trong 1 triu s u tin gn bng
log 106 theo c s ; tc l khong 28:
Tt nhin l ta c th vit r ra 30 s Fibonnaci u tin. S th 28 bng 832040; cn s th 29
ln hn 1 triu.
p
 Trong chui bi ton sau ta ly D 2  1; 4142:::
T1. Tm tt c cc gi tr c th ca L (vi cc n khc nhau).
T2. L c th nhn gi tr no u n l s bc 2
Li gii.
p Bi ton ny tng t vi bi ton trc. V gi tr ban u ca L (khi n D 2/ bng
p
p
p
2
2 p
D 2; v cc gi tr tip theo L s bng hoc 2; hoc 2 C 1 hoc 2 C 2 (vi cc
p
2 1
s n bc 2 th ch c hai gi tr u).
p
T3. Tm tt c cc gi tr c th ca L cho mt s no khc (ngoi D  v D 2/:
K1. Ta khng bit s : Nhng ta bit rng d n bng bao nhiu, s L ch nhn mt trong hai
gi tr. Vi nhng no iu ny c th?
Li gii. T cc l lun trc y ta thy rng L cn tha mn iu kin
L l php chia vng. T y d dng tm c :

L
L

1 D L; t y

K2. Bi ton nghin cu. Ta bit s nhng ta bit rng vi n ln ty , s im trn mi


1
n
on c di khc vi khng ln hn 10: Ta c th ni g v s C th:
2
2
 Hy nu v d mt vi s nh th.
 Hy a ra mt du hiu no iu ny khng xy ra: Nu c tnh cht ny v
tnh cht ny th khng nh bi ton khng ng.
1
 Hy a ra du hiu no c th khng nh s im trn mi on di nm
2
1
trong phm vi t a n b vi cc s a; b no .a < < b v ta mun cc s ny cng
2
1
gn cng tt).
2
186

Tp ch Epsilon, S 08, 04/2016


113
v n nhn cc gi tr n D 1; 2; 3; : : : ; 246: Trong cc s ny c
248
bao nhiu s bc 2; bao nhiu s bc 3
K3. Cho s hu t D

Li gii. Khai trin s cho thnh lin phn s (vi s


1

2C

113
th khai trin ny c dng
248

5C

7C

1
3

Khi s bc 2 s xut hin cch k; trong k l mu s ca phn s thnh phn ca lin phn
s, tc l cc phn s
1
36
1
5
1
1; ;
D ; :::
D ;
1
1
2 2C 5
11 2 C 1
79
5C
7

Nh vy, cc s bc hai u tin s cch 1 (tc l tt c, vi n D 2 v n D 3/; sau l dy s


bc 2 cch 2; sau l dy cch 11 v cch 79: Trong s cc s bc 2 trong dy s bng mu
s tng ng ca phn s ban u, tc l dy u c 2 s, sau l 5 s v 7 s.
Nu nh s v t th iu ny s tip tc cho n v cng, nhng i vi s hu t th cui s
c quy lut khc: Tt c cc s, bt u t 169 s c bc 2:
p s:
 S 2:
 S 3; 5; 7; 9; 11:
 S 13; 24; 35; : : : cho n 90:
 Cui cng, tt c cc s t 169 n 248:

K4. Hy nu ra phng php cho php vi s hu t cho trc


D

p
; p < q < 1000000;
q

tm c s s bc 2 v s s bc 3; khi n chy qua cc gi tr t 1 n q


php (bng tay).

1; trong thi gian cho

K5. Cho s L D L.100; /:


Lm sao c th xc nh vi mi gi tr c th ca n v cho trc tn ti hu hn hay v hn
cc gi tr khc nhau ca L
Ni ring, nghin cu cu hi ny cc trng hp sau:
187

Tp ch Epsilon, S 08, 04/2016


.1/ Nu L l nghim ca phng trnh bc 2 vi h s nguyn
L2 C nL C m D 0:
.2/ Nu L l nghim ca phng trnh bc 3 vi h s nguyn
L3 C nL2 C mL C q D 0:
K6. Bit rng s c vit thnh lin phn s vi cc mu s u tin l 3; 5; 12 tc l
1

1C
3C

1
1
C 
5C
12

Hy tm tt cc cc s bc 2 nm gia 1 v 100:

188

V K THI VIT NAM TST 2016 V


DANH SCH I TUYN VIT NAM
Ban Bin tp Epsilon

K thi chn i tuyn Vit Nam tham d Olympic Ton quc t (vit tt l IMO) 2016 din ra
trong 2 ngy 24, 25 thng 3. thi gm 6 bi ton, mi ngy 3 bi lm trong 4 gi 30 pht. Cc
bi ton thuc 4 phn mn i s, Hnh hc, S hc v T hp vi mc kh d khc nhau. C
th phn b cc bi ton nh sau.
 Bi 1: S hc.
 Bi 2: Nm gia t hp v i s.
 Bi 3: Hnh hc.
 Bi 4: Hnh hc.
 Bi 5: T hp v i s.
 Bi 6: i s.
Nm nay, c tng s 50 th sinh tham gia k thi ny, gm V Xun Trung, hc sinh lp 12 trng
THPT chuyn Thi Bnh, tnh Thi Bnh, l th sinh c HCV ti IMO 2015 cng 49 hc sinh
c t 27,5 im tr ln ti k thi HSG quc gia din ra vo u nm nay. Di y l thi
y .

1. thi
Bi 1.1. Tm a; n nguyn dng vi a > 2 mi c nguyn t ca an
2016
nguyn t ca a3
1:

1 cng u l c

Bi 1.2. Gi A l tp hp 2000 s nguyn phn bit v B l tp hp 2016 s nguyn phn bit


v K l s cp .m; n/ c th t vi m thuc A v n thuc B m
jm

nj  1000:

Tm gi tr ln nht ca K.
Bi 1.3. Cho tam gic ABC ni tip ng trn .O/ c B; C c nh v A chuyn ng trn
cung BC ca .O/. Cc phn gic AD; BE; CF giao nhau ti I . ng trn qua D tip xc
vi OA ti A ct .O/ ti G. GE; GF giao .O/ ln th hai ti M; N . BM giao CN ti H:
189

Tp ch Epsilon, S 08, 04/2016


1. Chng minh rng AH i qua mt im c nh.
2. Gi s BE; CF giao .O/ ln lt ti K; L. ng thng AH ct KL ti P . Gi s Q l
mt im trn EF sao cho QP D QI: Gi J l im nm trn .BIC / sao cho IJ ? IQ.
Chng minh rng trung im IJ chuyn ng trn mt ng trn c nh.
BAC
Bi 1.4. Cho tam gic ABC nhn c ACB < ABC < ACB C
. Ly im D
2
BAC
thuc cnh BC sao cho ADC D ACB C
. Tip tuyn vi ng trn ngoi tip tam
2
gic ABC ti A ct BC ti E. Phn gic AEB ct AD v ct .ADE/ ti G v F , DF giao
AE ti H:
1. Chng minh rng cc ng trn ng knh AE; DF; GH c mt im chung.
2. Trn phn gic ngoi BAC v trn tia AC ln lt ly cc im K v M sao cho
KB D KD D KM , trn phn gic ngoi BAC v trn tia AB ln lt ly cc im L
v N sao cho LC D LD D LN: ng trn i qua M; N v trung im I ca BC ct
BC ti P (P I ). Chng minh rng BM; CN; AP ng quy.
Bi 1.5. Cho a1 ; a2 ; : : : ; an 1 ; an (n  3), trong mi s ai nhn gi tr l 0 hoc 1. Xt n b
s nh sau:
S1 D .a1 ; a2 ; : : : ; an 1 ; an /;
S2 D .a2 ; a3 ; : : : ; an ; a1 /;
:::
Sn D .an ; a1 ; : : : ; an 2 ; an 1 /
Vi mi b s r D .b1 ; b2 ; : : : ; bn /, t
!.r/ D b1  2n

C b 2  2n

C : : : C bn  20 :

Gi s cc s !.S1 /I !.S2 /; : : : ; !.Sn / nhn ng k gi tr phn bit.


1. Chng minh rng k j n v

2n
2k

1
j !.Si / vi mi i D 1; n:
1

2. K hiu M v m ln lt l max v min ca !.S1 /; !.S2 /; : : : ; !.Sn /. Chng minh rng


.2n 1/.2k 1 1/
M m
:
2k 1
Bi 1.6. Cho cc s thc phn bit l 1 ; 2 ; : : : ; 16 . Vi mi a thc h s thc P .x/, ta t
V .P / D P .1 / C P .2 / C : : : C P .16 /:
Chng minh rng tn ti duy nht a thc Q.x/ bc 8 c h s x 8 bng 1 tha mn ng thi
hai iu kin:
1. V .QP / D 0 vi mi a thc P c bc b hn 8:
2. Q.x/ c 8 nghim thc (tnh c bi).
190

Tp ch Epsilon, S 08, 04/2016

2. Nhn xt chung.
V kh th bi 6 l kh nht do l mt bi ton ly t tng ca ton cao cp em xung (c
th l i s tuyn tnh). gii bi ton ny, hc sinh phi va bt c hng i quy np, t
tng trc giao ha Gram-Schmidt (hay t tng tng t) va phi c k nng x l k thut
rt tt. Bi th hai l bi 3b vi mt cu hnh rt ri theo dng hnh chng hnh.
Cc bi ton cn li v c bn c kh ngang nhau, lm c hay khng l ty thuc vo gu
ca cc th sinh. Bi s hc l mt bi ton c tng kh c m li gii ch yu da vo b
.am 1; an 1/ D a.m;n/ 1. l cha k nu dng n nh l Zsigmondy th bi ton gn
nh hin nhin.
Bi t hp s 2 cng l mt bi m kt qu c th on c d dng. Bi hnh s 4 l mt bi
ton kh p v va sc, mang tnh phn loi cao. Bi 5 li tip tc l mt bi ton thun ty k
thut. Ch l bi s 2 v bi s 5 tuy khng kh nhng trnh by cht ch th kh hn bi s
1 nhiu.
C l chnh v l do nn mc d v l thuyt th s hc sinh lm c 4 bi, 4 bi ri l kh
nhiu nhng thc t th ch c 3 bn c s im 27 tr ln (tc l coi nh lm c 4 bi tr ln)
v im chun vo i tuyn ch l 23.5, tc l 3 bi ++. y cng l iu cc th sinh phi
ht sc rt kinh nghim. Lm c bi th lun phi trnh by cho tht chc.

3. Kt qu i tuyn
Thng tin v i tuyn c cng b vo tun va qua nh sau:
H tn

Trng, lp

im

o V Quang
V Xun Trung
Hong Anh Dng
Phm Nguyn Mnh
L Nht Hong
V c Ti

Lp 12 THPT chuyn H Ni Amsterdam


Lp 12 THPT chuyn Thi Bnh
Lp 12 THPT chuyn Lam Sn, Thanh Ho
Lp 11, PTNK - HQG TP. HCM
Lp 12, THPT chuyn L Qu n, Bnh nh
Lp 12, THPT chuyn L Hng Phong, Nam nh

30.5
29
27
24,5
23,5
23,5

K thi IMO nm nay s c t chc ti Hong Kong, t ngy 06/7 n ngy 16/7/2016. Xin chc
cc th sinh c thi gian n tp, chun b tht tt v c kt qu cao nht ti k thi ny.
Ghi ch: Ban bin tp c tham kho t thng tin t trang c nhn trn facebook.com ca
thy Nguyn Khc Minh v thi t bi vit ca bn Nguyn Vn Linh trn mathscope.org.

191

You might also like